Vous êtes sur la page 1sur 215

Natalie Ceballos

Texas State Universitv, San Marcos




STUDENT STUDY GUIDE
FOR

Psychology
Third Edition







!"#$%&" () *+,,"&-..+
Gulf Coast Communitv College

/) 01."$% 23+4-
Georgia College and State Universitv










Prentice Hall
Boston Columbus Indianapolis New York San Francisco Upper Saddle River
Amsterdam Cape Town Dubai London Madrid Milan Munich Paris Montreal Toronto
Delhi Mexico City Sao Paulo Sydney Hong Kong Seoul Singapore Taipei Tokyo


Copyright 2012, 2009, 2006 Pearson Education, Inc., publishing as Prentice Hall, 1 Lake St.,
Upper Saddle River, NJ 07458. All rights reserved. ManuIactured in the United States oI
America. No part oI the material protected by this copyright notice may be reproduced or utilized
in any Iorm or by any means, electronic or mechanical, including photocopying, recording, or by
any inIormation storage and retrieval system, without written permission Irom the copyright
owner. To obtain permission(s) to use material Irom this work, please submit a written request to
Pearson Higher Education, Rights and Contracts Department, 501 Boylston Street, Suite 900,
Boston, MA 02116, or Iax your request to 617-671-3447.
10 9 8 7 6 5 4 3 2 1 14 13 12 11 10







www.pearsonhighered.com
ISBN-10: 0-205-15346-1
ISBN-13: 978-0-205-15346-6

Contents


1. The Science oI Psychology 1
2. The Biological Perspective 13
3. Sensation and Perception 27
4. Consciousness: Sleep, Dreams, Hypnosis, and Drugs 43
5. Learning 57
6. Memory 75
7. Cognition: Thinking, Intelligence, and Language 87
8. Development Across the LiIe Span 99
9. Motivation and Emotion 115
10. Sexuality and Gender 131
11. Stress and Health 143
12. Social Psychology 155
13. Theories oI Personality 169
14. Psychological Disorders 183
15. Psychological Therapies 197

CHAPTER 1 - THE SCIENCE OF PSYCHOLOGY

YOU KNOW YOU ARE READY FOR THE TEST IF YOU ARE ABLE TO.
DeIine psychology and describe the goals that psychologists hope to achieve.
Describe the history oI psychology.
Discuss the current state oI psychology, including the most common perspectives and major
proIessions in the Iield.
Describe the scientiIic method and discuss its strengths and weaknesses.
Explain the basic guidelines and ethical concerns oI psychological research.
Introduce the criteria Ior critical thinking and its application in psychology.

RAPID REVIEW
Psychology is deIined as the scientiIic study oI behavior and mental processes. The goals oI
psychology are to describe, explain, predict, and control the behaviors and mental process oI both humans
and animals. The goals oI psychology can be thought oI in terms oI what, why, when, and how behaviors
and mental processes occur.
The Iield oI psychology is relatively new (about 130 years old) but has its origins in the much
older Iields oI physiology and philosophy. Wilhelm Wundt Iormed the Iirst psychology laboratory in
Germany in 1879. Wundt used the method oI objective introspection in an attempt to study human
thought processes. Because oI his innovative eIIorts to bring objectivity and measurement to the concept
oI psychology, Wundt is oIten reIerred to as the Iather oI psychology. The reality, however, is that
multiple people in multiple locations began studying psychology and promoting their particular
perspective around the same time. Five historical perspectives are discussed in the text.
Edward Titchener, a student oI Wundt`s, expanded on Wundt`s ideas and brought the method oI
introspection to the United States. Titchener believed that introspection could be used on thoughts as well as
physical sensations. He called his approach structuralism because his ultimate goal was to describe the precise
structure oI our mental processes. At the same time in the United States, William James was Iocused on
discovering how our mental processes help us to Iunction in our daily lives and began to promote his viewpoint
known as functionalism. The terms structuralism and functionalism are no longer used to describe speciIic
viewpoints in the Iield oI psychology. Meanwhile, back in Germany, the Gestalt psychologists were studying
how sensation and perception create a whole pattern that is greater than the sum oI the individual components.
Max Wertheimer was a major proponent oI Gestalt psychology. In neighboring Austria, Sigmund Freud
developed his theory oI psychoanalysis based on the concept oI the unconscious. Freud believed the
unconscious played an important role in controlling our day-to-day behaviors and thoughts. Freud`s theory is
also reIerred to as the psychodynamic perspective. On the opposite end oI the spectrum, and back in the United
States, was John Watson. Watson expanded the Iindings oI Russian physiologist Ivan Pavlov, to promote the
perspective oI behaviorism. The behaviorists believed that psychology should Iocus on concepts that could be
studied scientiIically, and they Ielt that the only area oI psychology that could be approached scientiIically was
observable behavior.
Today seven major perspectives make up the Iield oI psychology. The psychodynamic
perspective Iocuses on the role oI the unconscious. Behaviorism attempts to study psychology by
Iocusing on observable actions and events. The humanistic perspective emphasizes human potential and
Iree will; in other words, it Iocuses on people`s abilities to direct their own lives. Biopsychology Iocuses
on the biology underlying our behavior and thoughts, while the cognitive perspective Iocuses on the
thoughts or 'cognitions themselves. Cognitive neuroscience is a speciIic area oI the cognitive
perspective that Iocuses on the physical changes in the brain that occur when we think, remember, or
engage in other mental processes. The sociocultural perspective explores the role oI social and cultural
Iactors on our behaviors and thoughts, while evolutionary psychologists attempt to explain behavior and
thoughts in terms oI their adaptive or 'survival qualities.
The Iield oI psychology oIIers many proIessional opportunities. Psychiatrists receive a medical
degree (M.D.), treat serious psychological disorders, and can prescribe medication Ior their patients. A
The Science of Psychology CHAPTER 1 -1-
psychologist attends graduate school to obtain a doctorate degree (either a Ph.D., Ed.D. or Psy.D.) and
can select one oI many career options Irom research to counseling to consulting Ior a business. A
psychoanalyst is a psychiatrist or psychologist who has received special training in Freud`s method oI
psychoanalysis. A psychiatric social worker receives a Master oI Social Work degree (M.S.W.) and
provides counseling to patients or possibly conducts research.
Psychologists use the scientific method to reduce bias and error in their observations. The steps
oI the scientiIic method include asking a question, turning that question into a hypothesisa statement
about what you believe the actual answer istesting your hypothesis, drawing a conclusion, and
reporting your Iindings. Your Iindings can then be Iurther strengthened iI other researchers conduct a
study and draw the same conclusions as you did, or in other words iI other researchers replicate your
Iindings. The method you use to test your hypothesis depends on which oI the Iour goals oI psychology
you are attempting to achieve. II you would like to answer the question oI 'what (goal describe), you
would use a descriptive method. Naturalistic observation provides a realistic picture oI behavior but can
become biased through the observer effect (research participants behave diIIerently when they know they
are being watched) and observer bias (the researcher only sees what he or she wants to see). Laboratory
observation is similar to naturalistic observation but the participants are observed in a laboratory setting
instead oI 'out in nature. Sometimes a researcher will disguise himselI or herselI as an actual participant
in order to reduce the observer eIIect. This approach is called participant observation. A case study is a
detailed investigation oI one individual, or case, and can provide a great deal oI inIormation about that
one person. However, case studies are hard to generalize to a larger population. For a survey, researchers
ask a group oI participants a series oI questions. Surveys allow researchers to gather a lot oI inIormation
quickly. However, a survey oIIers no guarantee that the participants will answer the questions truthIully.
Also, researchers must be sure to take a representative sample oI the population in which they are
interested. A researcher interested in discovering the relationship between two variables would use the
correlational method. A correlation coefficient tells the researcher the direction and strength oI the
relationship. The coeIIicient will always be a number between 1.00 and 1.00. A correlation shows that
a relationship between two variables exists, but cannot explain the cause oI the relationship. In other
words, correlation does not prove causation. In order to answer the question oI 'why, a researcher must
conduct an experiment. A true experiment diIIers Irom a quasi-experiment in that true experiments use
random assignment to sort participants into groups. In a quasi-experimental design, the groups are already
in place when the participants are recruited Ior the study (Ior example, smokers vs. nonsmokers). In an
experiment, the researcher manipulates a variable (the independent variable) and measures some
response Irom the participants (the dependent variable). In order to measure the dependent variable, the
researcher must come up with an operational definition Ior the variable. An operational deIinition is a
set oI instructions that explains exactly how to measure the variable. For example, aggressive behavior
could be operationally deIined as the number oI times a participant swings a toy sword in a Iive-minute
observation period. The overall goal oI the experiment is to keep everything the same except Ior the
independent variable. In order to accomplish this, the researcher usually observes two groups: an
experimental group and a control group. The researcher will most likely use random assignment to
determine which participants will go into which group. OIten, the control group receives a Iake treatment
in order to control Ior the placebo effect in which the participant`s expectations actually inIluence the
results oI the experiment. Normally, the participants are not told which group they are in (single-blind
study). In order to control Ior any expectations the experimenter might have (the experimenter effect)
the study is oIten designed so that neither the participants nor the experimenter know who is in which
group (double-blind study). All psychological research must Iollow the ethical guidelines speciIied by
the American Psychological Association.
Understanding the scientiIic method can help you in your daily liIe as you apply the Iour
principles oI critical thinking to problems you Iace on a day-to-day basis. The Iour criteria are as
Iollows: (1) most truths need to be tested, (2) all evidence is not equal, (3) authorities are not always right,
and (4) an open mind is still important.

The Science of Psychology CHAPTER 1 -2-
STUDY HINTS
1.

Be careIul not to conIuse the independent variable (i.v.) with the dependent variable (d.v.). The
independent variable is the variable the researcher manipulates her or himselI. II you think about
it as iI you were the researcher conducting the experiment, the independent variable is the one
that I control. Another way to make sure you have correctly labeled the variables in an
experiment is to insert the variable names into the Iollowing phrase and make sure it still makes
sense. The test phrase is:
How aIIects .
(i.v.) (d.v.)

Here is an example Ior you to practice using the test phrase.
A researcher conducts a studv looking at the color of different rooms and aggressiveness.
She takes a group of 40 college students and randomlv assigns 20 to the red room and 20
to the blue room. After the students have been in the rooms for 30 minutes, she measures
each persons aggressiveness level on a scale of 1 to 10. In this experiment, which
variable is the independent variable and which is the dependent? Trv inserting the
variable names into the phrase above.

You can see that 'How aggressiveness aIIects room color does not make sense and is not what
the researcher is interested in. However, 'How room color aIIects aggressiveness does
correspond to the researchers` goals. So in this case, the room color is the independent variable
and aggressiveness is the dependent variable.

Try one more example.
A researcher conducts an experiment to studv memorv skills and caffeine intake.
The researcher has a total of 20 volunteer subfects. He gives 10 subfects a can of
caffeinated soda and the other 10 subfects receive a can of decaffeinated soda.
He then has all the subfects complete a memorv task. What are his independent
and dependent variables? Trv inserting the variable names into the phrase
above.

Again, you can see that 'How memory skills aIIect caIIeine intake does not make sense and is
not what the researcher is interested in. However, 'How caIIeine intake aIIects memory skills
does correspond to the researcher`s goals. So in this case, the caIIeine intake is the independent
variable and memory skill is the dependent variable.


2.


The concept oI operational deIinitions is introduced in this chapter. An operational deIinition can
be thought oI as a recipe telling a researcher precisely how to make her observations. In other
words, they deIine the operations or procedures the researcher should go through in order to
record her data. Operational deIinitions are based on behaviors and actions that can be observed
and they are much diIIerent Irom the deIinitions given in a standard dictionary. For example, the
dictionary might deIine Iear as Ieeling anxious or apprehensive about a possible situation.
However, that deIinition does not tell the researcher how to measure one individual`s level oI
Iear. On the other hand, the researcher might operationally deIine Iear as the percent increase in
heart rate Irom a baseline level during a two-minute observation period.

Try this example yourselI.

Dictionarv definition of anger.


The Science of Psychology CHAPTER 1 -3-
Operational definition of anger.


The dictionary might deIine anger as a strong Ieeling oI displeasure. However, an operational
deIinition oI anger might be something like the number oI times an adult slams his or her Iists
on the table.

Now, try to Iigure out what variable is being operationally deIined below.

The number of times a person laughs within a five-minute period.

Operational Definition of .


The score an individual receives on an IQ test.

Operational Definition of .


The Iirst example is operationally deIining the variable oI happiness and the second example
gives an operational deIinition Ior intelligence.


LEARNING OBJECTIVES
1.1 What defines psvchologv as a field of
studv, and what are psvchologvs four
primarv goals?
1.2 How did structuralism and functionalism
differ, and who were the important people
in those earlv fields?
1.3 What were the basic ideas and who were
the important people behind the earlv
approaches known as Gestalt,
psvchoanalvsis, and behaviorism?
1.4 What are the basic ideas behind the seven
modern perspectives, as well as the
important contributions of Skinner,
Maslow, and Rogers?
1.5 How does a psvchiatrist differ from a
psvchologist, and what are the other tvpes
of professionals who work in the various
areas of psvchologv?
1.6 Whv is psvchologv considered a science,
and what are the steps in using the
scientific method?
1.7 How are naturalistic and laboratorv
settings used to describe behavior, and
what are some of the advantages and
disadvantages associated with these
settings?


1.8 How are case studies and survevs used to
describe behavior, and what are some
drawbacks to each of these methods?
1.9 What is the correlational technique, and
what does it tell researchers about
relationships?
1.10 How are operational definitions,
independent and dependent variables,
experimental and control groups, and
random assignment used in designing an
experiment?
1.11 Whv are the placebo and the experimenter
effects problems for an experiment, and
how can single-blind and double-blind
studies control for these effects?
1.12 What are the basic elements of a real-
world experiment?
1.13 What are some ethical concerns that can
occur when conducting research with
people and animals?
1.14 What are the basic principles of critical
thinking, and how can critical thinking be
useful in evervdav life?




The Science of Psychology CHAPTER 1 -4-
PRACTICE EXAM
For the Iollowing multiple choice questions, select the option you Ieel best answers the question or
completes the sentence.

1. How is psychology diIIerent Irom philosophy?
a) Psychology uses the scientiIic method to answer questions.
b) Psychology is interested in questions related to human behavior.
c) There is no diIIerence between philosophy and psychology.
d) The Iield oI psychology is much older than the Iield oI philosophy.

2. The Iirst psychology laboratory was opened in in order to study .
a) 1865; psychological disorders
b) 1946; learning
c) 1879; introspection
d) 1809; biopsychology

3. The psychological perspective oI structuralism Iocused on
a) how the whole structure is bigger than the individual parts.
b) understanding each individual structure oI human thought.
c) how mental thought helps us structure our daily activities.
d) the structure oI society at large.

4. The school oI psychology called structuralism used a technique called , which involved
reporting the contents oI consciousness to study a person's experiences.
a) intervention
b) introspection
c) insight inventory
d) induction

5. William James believed that mental processes could not be studied as isolated, static events but
instead needed to be viewed in terms oI how they helped people perIorm in their daily lives. James
was a strong proponent Ior
a) structuralism.
b) Iunctionalism.
c) behaviorism.
d) the humanistic perspective.

6. Gestalt psychologists are associated with which oI the Iollowing sayings?
a) The pineal gland is the seat oI the human soul.
b) Psychology should reach into the soul oI mankind.
c) Behavior should be broken down into its individual components.
d) The whole is greater than the sum oI its parts.

7. Freud said phobias were whereas Watson said phobias were .
a) learned; inherited
b) repressed conIlicts; learned
c) sexual; unconscious
d) conditioned; unconditioned

The Science of Psychology CHAPTER 1 -5-

8. A researcher who studies the chemical changes in the brains oI patients with depression would be
approaching psychology Irom which perspective?
a) behaviorist
b) psychodynamic
c) cognitive
d) biopsychological

9. One oI the reasons psychodynamic theories have persisted over the years is that they are
a) supported by signiIicant scientiIic research.
b) based on Iacts.
c) diIIicult to scientiIically test and, thus, diIIicult to disprove.
d) used by the majority oI psychologists.

10. A humanistic psychologist would be interested in which oI the Iollowing research studies?
a) describing a group oI people who claim to have reached their Iull potential
b) understanding the role oI the unconscious in a child`s decision to disobey her parents
c) investigating the role oI hormones in the mating behavior oI birds
d) Iiguring out visual illusions are possible

11. Taylor received her degree Irom a medical school and now meets with patients on a daily basis. Most oI her
patients have a serious psychological disorder and oIten Taylor will prescribe medication to treat the
disorder. Taylor is a
a) psychologist.
b) psychiatrist.
c) psychiatric social worker.
d) school nurse.

12. Vido has an M.S.W. and is interested in working on the causes oI poverty. What type oI
proIessional is Vido most likely to become?
a) educational psychologist
b) psychiatrist
c) school psychologist
d) psychiatric social worker

13. Why do psychologists use the scientiIic method?
a) It is easier to use than other methods.
b) All academic Iields must use the scientiIic method.
c) It is the only method available to answer questions.
d) It reduces bias and error in measurement.

14. Deb spent the entire day at the park observing children with their parents to see whether Iathers or
mothers spent more time playing with their kids. Deb used the method oI
a) naturalistic observation.
b) laboratory observation.
c) survey.
d) case study.

The Science of Psychology CHAPTER 1 -6-

15. Which oI the Iollowing topics would be best studied using the case study method?
a) the reaction times oI adults in a stressIul situation
b) the sleep pattern oI adolescents
c) the impact oI club sports involvement on Iemale adolescent selI-esteem
d) the personality characteristics oI a man accused oI killing Iive people

16. A group oI randomly selected subjects that matches the population on important characteristics such
as age and sex is called
a) volunteer bias.
b) a representative sample.
c) the experimental group.
d) the control group.
17. Which oI the Iollowing correlation coeIIicients represents the strongest relationship between two
variables?
a) 0.62
b) -0.98
c) 0.01
d) 1.24

18. A researcher Iinds that as the number oI classes missed increases, the students` grades decrease. This is an
example oI a
a) positive correlation.
b) negative correlation.
c) zero correlation.
d) case study.

19. Marcy is trying to deIine anxietv in a way that can be empirically tested. She is attempting to Iind an
appropriate
a) hypothesis.
b) operational deIinition.
c) double-blind study.
d) theory.

20. A researcher is investigating the eIIects oI exercise on weight. What are the independent and dependent
variables in this experiment?
a) The dependent variable is weight; the independent variable is exercise.
b) The independent variable is calories consumed; the dependent variable is diet.
c) The independent variable is weight; the dependent variable is calories consumed.
d) The dependent variable is amount oI exercise; the independent variable is calories consumed.

21. In a laboratory, smokers are asked to "drive" using a computerized driving simulator equipped with a stick
shiIt and a gas pedal. The object is to maximize the distance covered by driving as Iast as possible on a
winding road while avoiding rear-end collisions. Some oI the participants smoke a real cigarette
immediately beIore climbing into the driver's seat. Others smoke a Iake cigarette without nicotine. You are
interested in comparing how many collisions the two groups have. In this study, the group that smokes the
cigarette without nicotine is
a) the control group.
b) the driving simulator.
c) the experimental group.
d) the no-control group.

The Science of Psychology CHAPTER 1 -7-

22. A psychology proIessor Ieels that her students will do better on her exams iI music is playing while they
take their exams. To test her hypothesis she divides her class in halI. One halI takes the exam in a room with
music playing and the other halI takes the exam in a similar room but without the music playing. In this
case, the independent variable is
a) the time oI day when the exam is taken.
b) the absence or presence oI music playing.
c) the exam.
d) the students` scores on the exam.

23.

For the experiment described in Question 22, the dependent variable is
a) the room the exam is taken in.
b) the absence or presence oI music playing.
c) the exam.
d) the students` scores on the exam.

24. Twenty volunteers are brought into a sleep laboratory in the evening. Ten are allowed eight hours oI sleep
while the other ten are only allowed two hours oI sleep. In the morning, all 20 subjects are tested Ior their
reaction time in a driving simulation program. For this experiment, the reaction time in the simulation
program is the
a) independent variable.
b) dependent variable.
c) conIounding variable.
d) random variable.

25. For the experiment described in Question 24, the amount oI sleep allowed is the
a) independent variable.
b) dependent variable.
c) conIounding variable.
d) random variable.

26. Which oI the Iollowing situations best illustrates the placebo eIIect?
a) You sleep because you are tired.
b) You throw up aIter eating bad meat.
c) You have surgery to repair a deIective heart valve.
d) You drink a nonalcoholic drink and become "intoxicated" because you think it contains
alcohol.

27. is an experiment in which neither the participants nor the individuals running
the experiment know whether participants are in the experimental or the control group until aIter the results
are tallied.
a) The double-blind study
b) Field research
c) The single-blind study
d) Correlational research

The Science of Psychology CHAPTER 1 -8-
PRACTICE EXAM ANSWERS
1. a Psychology bases its answers on observations, while philosophy answers its questions using
logic and reasoning. Both Iields are interested in human behavior. The Iield oI psychology
is only 125 years old, while philosophy is much older.
2. c Wilhelm Wundt opened his laboratory in Germany in 1879 and used the method oI
introspection to study the basic elements oI mental processes.
3.

b Structuralists Ielt that mental processes had to be broken down into their most basic or
elemental Iorm in order to be understood.
4. b Introspection was used in an attempt to selI-examine the structure oI the mind. Although
the word intervention looks similar, it has a completely diIIerent meaning.
5. b James believed we need to understand the Iunction oI mental processes.
6. d Gestalt psychologists believed that you had to look at the whole picture in order to
understand the larger processes oI perception and sensation and that it could not be broken
down into its smaller components without losing its essence.
7. b Freud studied repressed (unconscious) conIlict and Watson studied observable behavior.
Watson did not believe that the unconscious could be studied scientiIically.
8. d The biopsychological perspective Iocuses on studying the biological changes that underlie
behavior and mental processes.
9. c Since it is very hard to scientiIically test the psychodynamic theories there is little scientiIic
data to support the theories.
10. a The humanistic perspective Iocuses on the uniqueness and potential oI human beings and
tries to suggest ways Ior humans to maximize their potential.
11. b Psychiatrists have M.D.s, counsel patients with serious disorders, and can prescribe
medications.
12. d Psychiatric social workers typically have their Masters oI Social Work (M.S.W.) and
counsel patients with less severe disorders or Iocus on social issues such as poverty.
13. d The scientiIic method is based on observations so that the inIluence oI the researcher`s bias
is minimized.
14. a Naturalistic observation consists oI recording behaviors as they occur in their normal
settings.
15. d A case study Iocuses on one individual (or 'case) and provides a detailed description oI
that individual.
16. b A representative sample is a randomly selected group that matches the population on
important characteristics. An experimental group is not necessarily representative oI the
population.
17. b The correlation coeIIicient must be between 1.00 and -1.00. The sign oI the coeIIicient
indicates the direction oI the relationship and the absolute value oI the coeIIicient indicates
the strength; thereIore, 0.98 is the largest absolute value listed between 0 and 1.
18. b For a negative correlation, the variables move in the opposite direction. As one variable
increases the other one decreases. In this case, as the number oI absences increase the grade
in class decreases.
19. b An operational deIinition deIines responses in terms that allow them to be measured, while
a hypothesis is an educated guess, not a deIinition.
20. a The exercise is controlled by the experimenter and is, thereIore, independent oI anything
the participants do, while the participants` weight is expected to depend on the amount oI
exercise.
21. a A control group gets either no treatment or treatment that has no eIIect (in this case,
experimenters are controlling Ior the possibility that the cigarette itselI, and not the
nicotine, might cause people to get into collisions).
22. b The independent variable is the variable the researcher manipulates. In this case, the
instructor manipulated whether music was playing.
The Science of Psychology CHAPTER 1 -9-

23. d Recall the test phrase, 'How (i.v.) aIIects (d.v.). The proIessor is
testing 'How music aIIects student test scores. The dependent variable is the subjects`
responses. The room the test is taken in and the test itselI should be the same Ior both
groups.
24. b The reaction time is the response observed in the subject. It is not manipulated by the
experimenter.
25. a Recall the test phrase, 'How hours slept aIIect driving reaction time.
26. d The placebo eIIect is brought on by expectations, and in this case you Ielt drunk only
because you believed you were drinking alcohol.
27. a The double-blind study is an experiment in which neither the participants nor the
individuals running the experiment know whether the participants are in the experimental
or control group. In a single-blind study, only the participants are 'blind.

CHAPTER GLOSSARY
behaviorism the science oI behavior that Iocuses on observable behavior only.
biopsychological
perspective
perspective that attributes human and animal behavior to biological events
occurring in the body, such as genetic inIluences, hormones, and the activity
oI the nervous system.
case study study oI one individual in great detail.
cognitive neuroscience study oI the physical changes in the brain and nervous system during
thinking.
cognitive perspective modern perspective that Iocuses on memory, intelligence, perception,
problem solving, and learning.
control group subjects in an experiment who are not subjected to the independent variable
and who may receive a placebo treatment.
correlation a measure oI the relationship between two variables.
correlation coefficient a number derived Irom the Iormula Ior measuring a correlation and
indicating the strength and the direction oI a correlation.
critical thinking making reasoned judgments about claims.
dependent variable variable in an experiment that represents the measurable response or
behavior oI the subjects in the experiment.
double-blind study study in which neither the experimenter nor the subjects know iI the subjects
are in the experimental or control group.
evolutionary perspective perspective that Iocuses on the biological bases oI universal mental
characteristics that all humans share.
experiment a deliberate manipulation oI a variable to see iI corresponding changes in
behavior result, allowing the determination oI cause-and-eIIect relationships.
experimental group subjects in an experiment who are subjected to the independent variable.
experimenter effect tendency oI the experimenter`s expectations Ior a study to unintentionally
inIluence the results oI the study.
functionalism early perspective in psychology associated with William James, in which the
Iocus oI study is how the mind allows people to adapt, live, work, and play.
Gestalt psychology early perspective in psychology Iocusing on perception and sensation,
particularly the perception oI patterns and whole Iigures.
humanistic perspective perspective that emphasizes human potential and the idea that people have
the Ireedom to choose their own destiny.
hypothesis tentative explanation oI a phenomenon based on observations.
independent variable variable in an experiment that is manipulated by the experimenter.
The Science of Psychology CHAPTER 1 -10-

naturalistic observation study in which the researcher observes people or animals in their normal
environment.
objective introspection the process oI examining and measuring one`s own thoughts and mental
activities.
observer bias tendency oI observers to see what they expect to see.
observer effect tendency oI people or animals to behave diIIerently Irom normal when they
know they are being observed.
operational definition deIinition oI a variable oI interest that allows it to be directly measured.
participant observation a naturalistic observation in which the observer becomes a participant in the
group being observed.
placebo effect the phenomenon in which the expectations oI the participants in a study can
inIluence their behavior.
population the entire group oI people or animals in which the researcher is interested.
psychiatric social worker a social worker with some training in therapy methods who Iocuses on the
environmental conditions that can have an impact on mental disorders, such
as poverty, overcrowding, stress, and drug abuse.
psychiatrist a medical doctor who has specialized in the diagnosis and treatment oI
psychological disorders.
psychoanalysis the theory and therapy based on the work oI Sigmund Freud.
psychoanalyst either a psychiatrist or a psychologist who has special training in the theories
oI Sigmund Freud and his method oI psychoanalysis.
psychodynamic
perspective
modern version oI psychoanalysis that is more Iocused on the development
oI a sense oI selI and the discovery oI other motivations behind a person`s
behavior than sexual motivations.
psychologist a proIessional with an academic degree and specialized training in one or
more areas oI psychology.
psychology the scientiIic study oI behavior and mental processes.
random assignment process oI assigning subjects to the experimental or control groups
randomly, so that each subject has an equal chance oI being in either group.
replicate in research, repeating a study or experiment to see iI the same results will be
obtained in an eIIort to demonstrate reliability oI results.
representative sample randomly selected sample oI subjects Irom a larger population oI subjects.
scientific method system oI gathering data so that bias and error in measurement are reduced.
single-blind study study in which the subjects do not know iI they are in the experimental or the
control group.
sociocultural perspective perspective that Iocuses on the relationship between social behavior and culture.
structuralism early perspective in psychology associated with Wilhelm Wundt and Edward
Titchener, in which the Iocus oI study is the structure or basic elements oI
the mind.
survey study conducted by asking a series oI questions to a group oI people.

The Science of Psychology CHAPTER 1 -11-
The Science of Psychology CHAPTER 1 -12-
CHAPTER 2 - THE BIOLOGICAL PERSPECTIVE

YOU KNOW YOU ARE READY FOR THE TEST IF YOU ARE ABLE TO.
Explain what neurons are and how they work to transIer and process inIormation.
Introduce the peripheral nervous system and describe its role in the body.
Discuss the role oI the endocrine system.
Describe the methods used to observe the structure and Iunction oI the brain.
IdentiIy the basic structures oI the brain and explain their Iunctions.

RAPID REVIEW
The nervous system is made up oI a complex network oI cells throughout your body. Because
psychology is the study oI behavior and mental processes, understanding how the nervous system works
provides Iundamental inIormation about what is going on inside your body when you engage in a speciIic
behavior, Ieel a particular emotion, or have an abstract thought. The Iield oI study that deals with these
types oI questions is called biological psychology or behavioral neuroscience. The role oI the nervous
system is to carry inIormation. Without your nervous system, you would not be able to think, Ieel, or act.
The cells in the nervous system that carry inIormation are called neurons. InIormation enters a neuron at
the dendrites, Ilows through the cell body (or soma) and down the axon in order to pass the inIormation
on to the next cell. Although neurons are the cells that carry the inIormation, most oI the nervous system
(about 90) consists oI glial cells. Glial cells provide Iood, support, and insulation to the neurons. The
insulation around the neuron is called myelin and works in a way similar to the plastic coating oI an
electrical wire. Bundles oI myelin-coated axons are wrapped together in cable like structures called
nerves.
Neurons use an electrical signal to send inIormation Irom one end oI its cell to the other. At rest, a
neuron has a negative charge inside and a positive charge outside. When a signal arrives, gates in the cell
wall next to the signal open and the positive charge moves inside. The positive charge inside the cell
causes the next set oI gates to open and those positive charges move inside. In this way, the electrical
signal makes its way down the length oI the cell. The movement oI the electrical signal is called an action
potential. AIter the action potential is over, the positive charges get pumped back out oI the cell and the
neuron returns to its negatively charged state. This condition is called the resting potential. A neuron
acts in an all-or-none manner, which means the neuron either has an action potential or it does not. The
neuron indicates the strength oI the signal by how many action potentials are produced or 'Iired within a
certain amount oI time.
Neurons pass inIormation on to target cells using a chemical signal. When the electrical signal travels
down the axon and reaches the other end oI the neuron called the axon terminal, it enters the very tip oI
the terminal called the synaptic knob. At this point, the electrical signal triggers a cascade oI events that
cause the neurotransmitters in the synaptic vesicles to be released into the Iluid-Iilled space between
the two cells. This Iluid-Iilled space is called the synapse or the synaptic gap. The neurotransmitters are
the chemical signals the neuron uses to communicate with its target cell. The neurotransmitters Iit into the
receptor sites oI the target cell and create a new electrical signal that then can be transmitted down the
length oI the target cell.
Neurotransmitters can have two diIIerent eIIects on the target cell. II the neurotransmitter increases
the likelihood oI an action potential in the target cell, the connection is called an excitatory synapse. II
the neurotransmitter decreases the likelihood oI an action potential, the connection is called an inhibitory
synapse. There are at least 50100 diIIerent types oI neurotransmitters in the human body. Acetylcholine
was the Iirst to be discovered; it is an excitatory neurotransmitter that causes your muscles to contract and
has a role in cognition, particularly memory. Gamma amino butyric acid (GABA) is an inhibitory
neurotransmitter that decreases the activity level oI neurons in your brain. Serotonin can Iunction as both
an excitatory and inhibitory neurotransmitter and has been linked with sleep, mood, and appetite. Low
levels oI the neurotransmitter dopamine have been Iound to cause Parkinson`s disease, and increased
levels oI dopamine have been linked to the psychological disorder known as schizophrenia. Endorphins
The Biological Perspective 13 CHAPTER 2
are a special neurotransmitter called a neural regulator that controls the release oI other neurotransmitters.
When endorphins are released in the body, the neurons transmitting inIormation about pain are not able to
Iire action potentials. The diIIerent types oI neurotransmitters are cleared out oI the synaptic gap through
the processes oI reuptake, diIIusion, or enzymatic degradation.
Agonists and antagonists are chemicals that are not naturally Iound in your body. However, when
they get into your nervous system, these chemicals can Iit into the receptor sites oI target cells. Agonists
lead to a similar response in the target cell as the neurotransmitter itselI, while antagonists block or reduce
the action oI the neurotransmitter on the target cell. Valium, a benzodiazepine, is an example oI an
agonist Ior the GABA system. It binds to GABA receptors and causes a calming eIIect in the central
nervous system. Drugs may also interact with the mechanisms Ior clearing the synaptic gap. For instance,
selective serotonin reuptake inhibitors (SSRIs), which are sometimes used to treat depression, block the
reuptake oI serotonin at the synapse, making more serotonin available to bind to post-synaptic receptors.
The central nervous system (CNS) is made up oI the brain and the spinal cord. The spinal cord is a
long bundle oI neurons that transmits messages between the brain and the body. The cell bodies or somas
oI the neurons are located along the inside oI the spinal cord and the cell axons run along the outside oI
the spinal cord. Afferent (sensory) neurons send inIormation Irom your senses to the spinal cord. For
example, sensory neurons would relay inIormation about a sharp pain in your Iinger. Efferent (motor)
neurons send commands Irom the spinal cord to your muscles, such as a command to pull your Iinger
back Irom a painIul stimulus. Interneurons connect sensory and motor neurons and help to coordinate
the signals. All three oI these neurons act together in the spinal cord to Iorm a reflex arc. The ability oI
the brain and spinal cord to change both in structure and Iunction is reIerred to as neuroplasticity. Stem
cells are one type oI cell that Iacilitates these changes.
The peripheral nervous system (PNS) is made up oI all the nerves and neurons that are NOT in the
brain or spinal cord. It includes all the nerves that connect to your eyes, ears, skin, mouth, and muscles.
The PNS is divided into two parts, the somatic nervous system and the autonomic nervous system. The
somatic nervous system consists oI all the nerves coming Irom your sensory systems, called the sensory
pathway, and all the nerves going to the skeletal muscles that control your voluntary movements, called
the motor pathway. The autonomic nervous system is made up oI the nerves going to and Irom your
organs, glands, and involuntary muscles and is divided into two parts: the sympathetic division and the
parasympathetic division. The sympathetic division turns on the body`s Iight-or-Ilight reactions, which
include responses such as increased heart rate, increased breathing, and dilation oI your pupils. The
parasympathetic division controls your body when you are in a state oI rest to keep the heart beating
regularly, to control normal breathing, and to coordinate digestion. The parasympathetic division is active
most oI the time.
The endocrine glands represent a second communication system in the body. The endocrine glands
lack ducts and secrete chemicals called hormones directly into the bloodstream. Compared to neuronal
communication, the hormonal system generally results in slower, more widespread eIIects on the body
and/or behavior. The pituitary gland is located in the brain and secretes the hormones that control milk
production, salt levels, and the activity oI other glands. The pineal gland is also located in the brain and
secretes melatonin. This hormone helps to track day length and contributes to the regulation oI the sleep
cycle in humans. The thyroid gland is located in the neck and releases a hormone that regulates
metabolism. The pancreas controls the level oI blood sugar in the body, while the gonad sex glands
called the ovaries in Iemales and the testes in malesregulate sexual behavior and reproduction. The
adrenal glands are divided into two sections that play a critical role in regulating the body`s response to
stress. The adrenal medulla releases epinephrine and norepinephrine, whereas the adrenal cortex releases
stress hormones such as cortisol.
Researchers have used animal models to learn a great deal about the human brain. Two oI the most
common techniques used in animals involve either destroying a speciIic area oI the brain (deep lesioning)
or stimulating a speciIic brain area (electrical stimulation of the brain or ESB) to see the eIIect.
Researchers have applied both invasive and noninvasive Iorms oI ESB to treat human conditions. For
instance, deep brain stimulation (DBS) has been used in the treatment oI Parkinson`s disease.
Transcranial magnetic stimulation (TMS) has been applied to the treatment oI posttraumatic stress
The Biological Perspective 14 CHAPTER 2
disorder (PSTD). In work with humans, researchers have also developed several imaging methods to
observe the structure and Iunction oI a living brain. II a researcher wants a picture oI the structure oI the
brain, he or she might choose a CT scan or an MRI. Computed tomography (CT) scans use X-rays to
create images oI the structures within the brain. Magnetic resonance images (MRIs) use a magnetic Iield
to 'take a picture oI the brain. MRIs provide much greater detail than CT scans. On the other hand, iI a
researcher wanted to record the activity oI the brain, he or she might select an EEG, fMRI, PET scan, or
SPECT scan. An electroencephalogram (EEG) provides a record oI the electrical activity oI groups oI
neurons just below the surIace oI the skull. A Iunctional magnetic resonance image (IMRI) uses magnetic
Iields in the same way as an MRI, but goes a step Iurther and pieces the pictures together to show changes
over a short period oI time. A positron emission tomography (PET) scan involves injecting a person with
a low dose oI a radioactive substance and then recording the activity oI that substance in the person`s
brain. The single photon emission computed tomography (SPECT) scan Iunctions similarly to the PET
scan but uses a somewhat diIIerent radiotracer technique.
The brain can be roughly divided into three sections: the brainstem, the cortex, and the structures
under the cortex. The brainstem is the lowest part oI the brain that connects to the spinal cord. The outer
wrinkled covering oI the brain is the cortex, and the structures under the cortex are essentially everything
between the brainstem and the cortex. The brainstem contains Iour important structures. The medulla
controls liIe-sustaining Iunctions such as heart beat, breathing, and swallowing. The pons inIluences
sleep, dreaming, and coordination oI movements. The reticular formation plays a crucial role in
attention and arousal, and the cerebellum controls all oI the movements you make without really
'thinking about it.
One main group oI structures under the cortex is the limbic system. The limbic system includes the
thalamus, hypothalamus, hippocampus, amygdala, and cingulate cortex. The thalamus receives input
Irom your sensory systems, processes it, and then passes it on to the appropriate area oI the cortex. The
hypothalamus interacts with the endocrine system to regulate body temperature, thirst, hunger, sleeping,
sexual activity, and mood. It appears that the hippocampus is critical Ior the Iormation oI long-term
memories and Ior memories oI the locations oI objects. The amygdala is a small almond-shaped structure
that is involved in your response to Iear. The cingulate cortex plays an important role in both emotion and
cognition.
In a process reIerred to as 'corticalization, the outer part oI the brain, or cortex, is wrinkled to
increase its surIace area within the bony skull. The cortex is divided into right and leIt sections called
cerebral hemispheres. The two hemispheres communicate with each other through a thick band oI
neurons called the corpus callosum, which is located immediately below the cingulate cortex. Each
cerebral hemisphere can be roughly divided into Iour sections. These sections are called lobes. The
occipital lobes are at the back oI the brain and process visual inIormation. The parietal lobes are located
at the top and back halI oI the brain and deal with inIormation regarding touch, temperature, body
position, and possibly taste. The temporal lobes are just behind your temples and process auditory
inIormation. The frontal lobes are located at the Iront oI your head and are responsible Ior higher mental
Iunctions such as planning, personality, and decision making, as well as language and motor movements.
Motor movements are controlled by a band oI neurons called the motor cortex, which is located at the
back oI the Irontal lobe. Mirror neurons, neurons that Iire when you perIorm an action and also when
you see someone else perIorm that action, may explain a great deal oI the social learning that takes place
in humans Irom inIancy on. Recent studies suggest that humans have mirror neurons in areas oI the brain
associated with movement, vision, and memory.
Association areas are the areas within each oI the lobes that are responsible Ior 'making sense oI all
the incoming inIormation. Broca`s area is located in the leIt Irontal lobe in most people and is
responsible Ior language production. A person with damage to this area would have trouble producing the
words that he or she wants to speak. This condition is reIerred to as Broca`s aphasia. The comprehension
oI language takes place in Wernicke`s area located in the leIt temporal lobe. II this area oI the brain is
damaged, individuals are oIten still able to speak Iluently, but their words do not make sense. This type oI
language disorder is reIerred to as Wernicke`s aphasia. Damage to the right parietal and occipital lobes
The Biological Perspective 15 CHAPTER 2
can cause a condition known as spatial neglect in which the individual ignores objects or body parts in
their leIt visual Iield.
The cerebrum is made up oI the two cerebral hemispheres and the structures connecting them. The
split-brain research studies oI Roger Sperry helped scientists to Iigure out that the two cerebral
hemispheres are not identical. The leIt hemisphere is typically more active when a person is using
language, math, and other analytical skills, while the right hemisphere shows more activity during tasks
oI perception, recognition, and expression oI emotions. This split in the tasks oI the brain is reIerred to as
lateralization.

STUDY HINTS
3.

You will need to know the diIIerent Iunctions oI the peripheral nervous system (PNS). Recall
that the PNS is divided into two main sections: the somatic nervous system and the autonomic
nervous system. The somatic nervous system deals with the senses and the skeletal muscles (all
'S`s) and is Iairly straightIorward to understand. The autonomic nervous system is slightly
more complicated. First, understand that the autonomic nervous system deals with all the
automatic Iunctions oI your body. What are some Iunctions that are controlled automatically in
your body? List them here:

, , , ,

You probablv mentioned functions such as digestion, heart rate, pupil dilation, breathing, salivation,
or perspiration, to name a few. These are the functions controlled bv the autonomic svstem.

The two components oI the autonomic system balance each other out. The two divisions are the
sympathetic and parasympathetic divisions. Most oI the time, the parasympathetic division is in
control. Some people have called the parasympathetic division the rest-and-digest system because it
controls the digestive processes, maintains a resting heart and breathing rate, and in general keeps your
body in its normal relaxed state. The sympathetic division goes into action when your body needs to
react to some type oI threat. It might be helpIul to associate sympathetic with surprise, since the
sympathetic division is the part oI your nervous system that responds when you are surprised. This
system is oIten reIerred to as the Iight-or Ilight system. What happens to your body when you are
surprised? List some oI the responses here:

, , , ,

You probablv mentioned responses such as vour heart rate increases, vou breathe faster, vour
pupils dilate, vou begin to sweat, to name a few. All of these responses are 'turned on` bv the
svmpathetic division of vour autonomic nervous svstem and aid in vour survival bv allowing vou
to respond quicklv to a threat.


4.

Two oI the brain structures most commonly conIused with each other are the hippocampus and the
hypothalamus. Both oI the structures are located in the limbic system in the area oI your brain above
your brainstem and below the outer surIace. The hippocampus has been Iound to be important in
helping us Iorm memories that last more than just a Iew seconds. Patients with damage to the
hippocampus oIten cannot remember inIormation Ior longer than a Iew seconds. The hippocampus
is also important in storing memories oI where things are located, a spatial map. On the other hand,
the hypothalamus is important in controlling many oI your basic bodily Iunctions such as sleeping,
drinking, eating, and sexual activities. The structures are oIten conIused because the two words
sound so similar to each other. Can you think oI any memory device or 'trick to help you keep
these two brain structures separate? List your idea in the Iollowing space:

The Biological Perspective 16 CHAPTER 2
hippocampus:


hypothalamus:


One suggestion might be as follows. If vou look at the word hippocampus vou can think of the
last part of the wordcampus. In order to get around on vour college campus, vou need to
keep in mind where certain buildings and areas are located. This is exactlv what vour
hippocampus is involved in. Without vour hippo-campus, vou would have a verv hard time
finding vour wav around vour college campus.

To remember the hypothalamus, Iirst it might help to understand how the name came about.
'Hypo means under or below. For example, iI someone has 'hypothermia their body
temperature is under the normal amount and the person is probably Ieeling very cold. II
someone has 'hypoglycemia they have under or lower than the normal amount oI blood sugar
(glycemia is reIerring to the sugar Iound in your blood). What do you think 'hypothalamus
means?



If vou wrote 'under the thalamus,` then vou are correct. The hvpothalamus is located directlv
underneath the thalamus. You might also look at the name to trv to remember some of the
activities the hvpothalamus regulates. Recall that we said the hvpothalamus plavs a role in
hunger, sleep, thirst, and sex. If vou look at the 'hvpo` of hvpothalamus vou might memori:e
'h` hunger, 'v` vawning, 'p` parched (or verv, verv thirstv), and 'o` overlv excited.


LEARNING OBJECTIVES
2.1 What are the nervous svstem, neurons, and
nerves, and how do thev relate to one
another?
2.2 How do neurons use neurotransmitters to
communicate with each other and with the
bodv?
2.3 How do the brain and spinal cord interact?
2.4 How do the somatic and autonomic nervous
svstems allow people and animals to
interact with their surroundings and control
the bodvs automatic functions?
2.5 How do the hormones released bv glands
interact with the nervous svstem and affect
behavior?



2.6 How do psvchologists studv the structure
and function of the brain?
2.7 What are the different structures of the
bottom part of the brain and what do thev
do?
2.8 What are the structures of the brain that
control emotion, learning, memorv, and
motivation?
2.9 What parts of the cortex control the different
senses and the movement of the bodv?
2.10 What parts of the cortex are responsible for
higher forms of thought, such as language?
2.11 How does the left side of the brain differ
from the right side?

The Biological Perspective 17 CHAPTER 2

PRACTICE EXAM
For the Iollowing multiple choice questions, select the option you Ieel best answers the question or
completes the sentence.

1. Which oI the Iollowing terms reIers to a group oI specialized cells that carry inIormation to and Irom
all parts oI the body?
a) Soma
b) Synapse
c) nervous system
d) Endorphins

2. The central nervous system is made oI which two components?
a) the somatic and autonomic systems
b) the brain and the spinal cord
c) the sympathetic and parasympathetic divisions
d) Neurotransmitters and hormones

3. Located within the nervous system, the Iunction oI the is to send and receive messages.
a) glial cell
b) neuron
c) Schwann cell
d) oligodendrocyte

4. What type oI signal is used to relay a message Irom one end oI a neuron to the other end?
a) chemical
b) hormonal
c) biochemical
d) electrical

5. When , a chemical Iound in the synaptic vesicles, is released, it aIIects the next cell.
a) glial cell
b) neurotransmitter
c) precursor cell
d) synapse

6. Which specific event causes the release oI chemicals into the synaptic gap?
a) an agonist binding to the dendrites
b) an action potential reaching the axon terminal
c) the reuptake oI neurotransmitters
d) excitation oI the glial cells

7. Sara has been experiencing a serious memory problem. An interdisciplinary team has ruled out a
range oI causes and believes that a neurotransmitter is involved. Based on the inIormation presented
in Chapter 2, which neurotransmitter is most likely involved in this problem?
a) GABA
b) dopamine
c) serotonin
d) acetylcholine

The Biological Perspective 18 CHAPTER 2

8. Neuron A releases a neurotransmitter into the synaptic gap. As a result, the Irequency oI action
potentials in Neuron B (the receptor cell) is reduced. Which oI the Iollowing neurotransmitters is
most likely to have been released by Neuron A?
a) an inhibitory neurotransmitter
b) an excitatory neurotransmitter
c) glutamate
d) an agonist

9. Which part oI the nervous system takes the inIormation received Irom the senses, makes sense out oI
it, makes decisions, and sends commands out to the muscles and the rest oI the body?
a) spinal cord
b) brain
c) reIlexes
d) interneurons

10. Every deliberate action you make, such as pedaling a bike, walking, scratching, or smelling a
Ilower, involves neurons in the nervous system.
a) sympathetic
b) somatic
c) parasympathetic
d) autonomic

11. The heart and the intestines are composed oI muscles and are controlled by .
a) involuntary; the somatic nervous system
b) involuntary; the autonomic nervous system
c) voluntary; the sympathetic nervous system
d) voluntary; the parasympathetic nervous system

12. Which oI the Iollowing responses would occur iI your sympathetic nervous system has been
activated?
a) increased heart rate
b) pupil constriction
c) slowed breathing
d) increased digestion

13. Small metal disks are pasted onto Miranda's scalp and they are connected by wire to a machine that
translates the electrical energy Irom her brain into wavy lines on a moving piece oI paper. From this
description, it is evident that Miranda's brain is being studied through the use oI
a) a CT scan.
b) Iunctional magnetic resonance imaging (IMRI).
c) a microelectrode.
d) an electroencephalograph.

14. Which method would a researcher select iI she wanted to determine whether her patient`s right
hemisphere was the same size as his leIt hemisphere?
a) EEG
b) deep lesioning
c) CT scan
d) PET scan

The Biological Perspective 19 CHAPTER 2

15. Which oI the Iollowing is responsible Ior the ability to selectively attend to certain kinds oI
inIormation in one's surroundings and become alert to changes in inIormation?
a) reticular Iormation
b) pons
c) medulla
d) cerebellum

16. When a proIessional baseball player swings a bat and hits a home run, he is relying on his
to coordinate the practiced movements oI his body.
a) pons
b) medulla
c) cerebellum
d) reticular Iormation

17. Eating, drinking, sexual behavior, sleeping, and temperature control are most strongly inIluenced by
the
a) hippocampus.
b) thalamus.
c) hypothalamus.
d) amygdala.

18. AIter a brain operation, a laboratory rat no longer displays any Iear when placed into a cage with a
snake. Which part oI the rat's brain was most likely damaged during the operation?
a) amygdala
b) hypothalamus
c) cerebellum
d) hippocampus

19. Darla was in an automobile accident that resulted in an injury to her brain. Her sense oI touch has
been aIIected. Which part oI the brain is the most likely site oI the damage?
a) Irontal lobes
b) temporal lobes
c) occipital lobes
d) parietal lobes

20. II an individual damages his occipital lobes, which would be the most likely problem he would
report to his doctor?
a) trouble hearing
b) problems with his vision
c) decreased sense oI taste
d) numbness on the right side oI his body

21. Damage to what area oI the brain would result in an inability to comprehend language?
a) occipital lobes
b) Broca's area
c) Wernicke's area
d) parietal lobe

The Biological Perspective 20 CHAPTER 2

22. II Darren's brain is like that oI most people, then language will be handled by his
a) corpus callosum.
b) occipital lobe.
c) right hemisphere.
d) leIt hemisphere.

23. The two hemispheres oI the brain are identical copies oI each other.
a) true
b) Ialse

24. Which oI the Iollowing hormones is released by the pineal gland and prepares you Ior sleep?
a) melatonin
b) DHEA
c) parathormone
d) thyroxin

25. Which endocrine gland regulates your body's response to stress?
a) pancreas
b) thyroid gland
c) pineal gland
d) adrenal gland

PRACTICE EXAM ANSWERS
1. c The nervous system is the correct answer because it is composed oI a group oI specialized
cells that send inIormation to and Irom all parts oI the body. The soma and the synapse are
both parts oI an individual neuron (not a group oI cells), and endorphins are one type oI
neurotransmitter Iound in the body.
2. b The central nervous system is composed oI the nerves and neurons in the center oI your
body. The somatic and autonomic systems, as well as the sympathetic and parasympathetic
divisions, are components oI the peripheral nervous system. Hormones are the chemical
messengers Ior the endocrine system.
3. b Neurons are specialized cells within the nervous system that receive and send messages.
Glial cells serve as a structure Ior neurons. Schwann cells and oligodendrocytes are special
types oI glial cells that generate myelin.
4. d Neurons use electrical signals to communicate within their own cell. The electrical signal is
called an action potential.
5. b Neurotransmitters are stored in the synaptic vesicles. The synapse is the space between the
synaptic knob oI one cell and the dendrites. Glial cells provide structure Ior neurons, and
precursor cells are not relevant to this question.
6. b When the electrical signal (called an action potential) reaches the axon terminal, the
synaptic vesicles release their contents into the synaptic gap. Agonists binding to
dendrites and the reuptake oI neurotransmitters are activities that can occur in a neuron,
but do not directlv result in neurotransmitter release. Glial cells Iunction as a support
structure.
7. d Acetylcholine is Iound in a part oI the brain responsible Ior Iorming new memories.
8. a Inhibitory neurotransmitters Irom Neuron A inhibit the electrical activity oI Neuron B
(receptor cell). The other answer choices would result in the opposite eIIect; Ior
example, glutamate is a major excitatory neurotransmitter in the central nervous
system.
The Biological Perspective 21 CHAPTER 2

9. b The spinal cord carries messages to and Irom the body to the brain, but it is the job oI
the brain to make sense oI all the inIormation. ReIlexes are an action rather than a
structure, and interneurons were only discussed in terms oI their role in spinal cord
reIlexes.
10. b The somatic nervous system controls voluntary muscle movement, whereas the
autonomic nervous system consists oI nerves that control all oI the involuntary muscles,
organs, and glands. The sympathetic and parasympathetic systems are merely divisions
oI the autonomic nervous system.
11. b The autonomic nervous system controls involuntary muscles like the heart and
intestines. The somatic nervous system controls voluntary muscles. The heart and
intestines are involuntary rather than voluntary muscles.
12. a The sympathetic division is responsible Ior controlling your body's Iight-or-Ilight
response, which prepares your body to deal with a potential threat. The responses
include increased heart rate and breathing, pupil dilation, decreased digestion, among
others.
13. d An electroencephalograph or EEG records brain wave patterns. CT scans take
computer-controlled X-rays oI the brain. Microelectrodes were not mentioned in this
context in Chapter 2. IMRI does not record electrical activity oI the brain.
14. c A CT scan is the only selection among these options that would allow the researcher to
take a picture oI the structure oI the brain. The other options listed provide inIormation
about the function oI the brain.
15. b The reticular Iormation plays a role in selective attention.
16. c The cerebellum is responsible Ior controlling the movements that you have practiced
repeatedly, the movements that you don't have to really 'think about.
17. c The hypothalamus regulates sleep, hunger, thirst, and sex.
18. a The amygdalalocated within the limbic system, a part oI your brain responsible Ior
regulating emotions and memorieshas been Iound to regulate the Iear response.
19. d The parietal lobes contain the centers Ior touch, taste, and temperature.
20. b The occipital lobes are responsible Ior processing visual inIormation.
21. c Wernicke's area is located in the temporal lobe and is important in the comprehension
oI language. Broca's area is located in the Irontal lobe and plays a role in the production
oI language. The occipital and parietal lobes are not specialized language areas.
22. d For most people the leIt hemisphere specializes in language.
23. b The leIt hemisphere is more active during language and math problems, while the right
hemisphere appears to play a larger role in nonverbal and perception based tasks.
24. a The pineal gland secretes melatonin.
25. d The adrenal glands secrete several hormones in response to stress.

CHAPTER GLOSSARY
agonists chemical substances that mimic or enhance the eIIects oI a neurotransmitter on the
receptor sites oI the next cell, increasing or decreasing the activity oI that cell.
acetylcholine the Iirst neurotransmitter to be discovered. Found to regulate memories in the CNS
and the action oI skeletal and smooth muscles in the PNS.
action potential the release oI the neural impulse consisting oI a reversal oI the electrical charge
within the axon.
adrenal glands endocrine glands located on top oI each kidney that secrete over 30 diIIerent
hormones to deal with stress, regulate salt intake, and provide a secondary source oI
sex hormones aIIecting the sexual changes that occur during adolescence.
The Biological Perspective 22 CHAPTER 2

afferent (sensory)
neuron
a neuron that carries inIormation Irom the senses to the central nervous system.
all-or-none reIerring to the Iact that a neuron either Iires completely or does not Iire at all.
amygdala brain structure located near the hippocampus, responsible Ior Iear responses and
memory oI Iear.
antagonists chemical substances that block or reduce a cell`s response to the action oI other
chemicals or neurotransmitters.
association areas areas within each lobe oI the cortex responsible Ior the coordination and
interpretation oI inIormation, as well as higher mental processing.
autonomic nervous
system
division oI the PNS consisting oI nerves that control all oI the involuntary muscles,
organs, and glands.
axon tubelike structure that carries the neural message to other cells.
axon terminals branches at the end oI the axon.
biological psychology
or behavioral
neuroscience
branch oI neuroscience that Iocuses on the biological bases oI psychological
processes, behavior, and learning.
brainstem section oI the brain that connects directly to the spinal cord and regulates vital
Iunctions such as breathing, the heart, reIlexes, and level oI alertness.
Broca`s aphasia condition resulting Irom damage to Broca`s area, causing the aIIected person to be
unable to speak Iluently, to mispronounce words, and to speak haltingly.
central nervous
system (CNS)
part oI the nervous system consisting oI the brain and spinal cord.
cerebellum part oI the lower brain located behind the pons that controls and coordinates
involuntary, rapid, Iine motor movement.
cerebral hemispheres the two sections oI the cortex on the leIt and right sides oI the brain.
cerebrum the upper part oI the brain consisting oI two hemispheres and the structures that
connect them.
cingulate cortex primary cortical component oI the limbic system, involved in emotional and
cognitive processing.
computed tomography
(CT)
brain-imaging method using computer-controlled X-rays oI the brain.
corpus callosum thick band oI neurons that connects the right and leIt cerebral hemispheres.
cortex outermost covering oI the brain consisting oI densely packed neurons, responsible
Ior higher thought processes and interpretation oI sensory input.
deep lesioning insertion oI a thin, insulated wire into the brain through which an electrical current is
sent that destroys the brain cells at the tip oI the wire.
dendrites branchlike structures that receive messages Irom other neurons.
diffusion process oI molecules moving Irom areas oI high concentration to areas oI low
concentration.
dopamine neurotransmitter that regulates movement, balance, and walking and is involved in
the disorders oI schizophrenia and Parkinson`s disease.
efferent (motor)
neuron
a neuron that carries messages Irom the central nervous system to the muscles oI the
body.
electroencephalograph machine designed to record the electroencephalogram.
electroencephalogram
(EEG)
a recording oI the electrical activity oI large groups oI cortical neurons just below the
skull, most oIten using scalp electrodes.
endocrine glands glands that secrete chemicals called hormones directly into the bloodstream.
The Biological Perspective 23 CHAPTER 2
endorphin neurotransmitter that is Iound naturally in the body and works to block pain and
elevate mood. It is chemically similar to morphine and its name is short Ior
'endogenous morphine.
enzymatic
degradation
process by which structure oI neurotransmitters is altered so it can no longer act on a
receptor.
excitatory synapse synapse at which a neurotransmitter causes the receiving cell to Iire.
frontal lobes areas oI the cortex located in the Iront and top oI the brain, responsible Ior higher
mental processes and decision making as well as the production oI Iluent speech.
functional magnetic
resonance imaging
(fMRI)
MRI-based brain-imaging method that allows Ior a Iunctional examination oI brain
areas through changes in brain oxygenation.
GABA abbreviation Ior gamma-aminobutyric acid, the major inhibitory neurotransmitter in
the brain.
glial cells cells that provide support Ior the neurons to grow on and around, deliver nutrients to
neurons, produce myelin to coat axons, clean up waste products and dead neurons,
inIluence inIormation processing, and during prenatal development, inIluence the
generation oI new neurons.
gonads sex glands; secrete hormones that regulate sexual development and behavior as well
as reproduction.
hippocampus curved structure located within each temporal lobe, responsible Ior the Iormation oI
long-term memories and the storage oI memory Ior location oI objects.
hormones chemicals released into the bloodstream by endocrine glands.
hypothalamus small structure in the brain located below the thalamus and directly above the
pituitary gland, responsible Ior motivational behavior such as sleep, hunger, thirst,
and sex.
inhibitory synapse synapse at which a neurotransmitter causes the receiving cell to stop Iiring.
interneuron a neuron Iound in the center oI the spinal cord that receives inIormation Irom the
aIIerent neurons and sends commands to the muscles through the eIIerent neurons.
Interneurons also make up the bulk oI the neurons in the brain.
limbic system a group oI several brain structures located under the cortex and involved in learning,
emotion, memory, and motivation.
magnetic resonance
imaging (MRI)
brain-imaging method using radio waves and magnetic Iields oI the body to produce
detailed images oI the brain.
medulla the Iirst large swelling at the top oI the spinal cord, Iorming the lowest part oI the
brain, which is responsible Ior liIe-sustaining Iunctions such as breathing,
swallowing, and heart rate.
mirror neurons neurons that Iire when an animal or person perIorms an action and also when an
animal or person observes that same action being perIormed by another.
motor cortex section oI the Irontal lobe located at the back, responsible Ior sending motor
commands to the muscles oI the somatic nervous system.
motor neuron a neuron that carries messages Irom the central nervous system to the muscles oI the
body. Also called eIIerent neuron.
motor pathway nerves coming Irom the CNS to the voluntary muscles, consisting oI eIIerent
neurons.
myelin Iatty substances produced by certain glial cells that coat the axons oI neurons to
insulate, protect, and speed up the neural impulse.
nerves bundles oI axons coated in myelin that travel together through the body.
nervous system an extensive network oI specialized cells that carry inIormation to and Irom all parts
oI the body.
The Biological Perspective 24 CHAPTER 2
neuron the basic cell that makes up the nervous system and which receives and sends
messages within that system.
neuroplasticity the ability within the brain to constantly change both the structure and Iunction oI
many cells in response to experience or trauma.
neuroscience a branch oI the liIe sciences that deals with the structure and Iunction oI neurons,
nerves and nervous tissue.
neurotransmitter chemical Iound in the synaptic vesicles that, when released, has an eIIect on the next
cell.
occipital lobes sections oI the brain located at the rear and bottom oI each cerebral hemisphere
containing the visual centers oI the brain.
olfactory bulbs two bulb-like projections just under the Iront oI the brain that receives inIormation
Irom the receptors in the nose.
ovaries the Iemale gonads.
pancreas endocrine gland; controls the levels oI sugar in the blood.
parasympathetic
division
part oI the ANS that restores the body to normal Iunctioning aIter arousal and is
responsible Ior the day-to-day Iunctioning oI the organs and glands.
parietal lobes sections oI the brain located at the top and back oI each cerebral hemisphere
containing the centers Ior touch, taste, and temperature sensations.
peripheral nervous
system (PNS)
all nerves and neurons that are not contained in the brain and spinal cord but that run
through the body itselI.
pineal gland endocrine gland located near the base oI the cerebrum; secretes melatonin.
pituitary gland gland located in the brain that secretes human growth hormone and inIluences all
other hormone-secreting glands (also known as the master gland).
pons the larger swelling above the medulla that connects the top oI the brain to the bottom
and that plays a part in sleep, dreaming, leItright body coordination, and arousal.
positron emission
tomography (PET)
brain-imaging method in which a radioactive sugar is injected into the subject and a
computer compiles a color-coded image oI the activity oI the brain.
receptor sites 3-dimensional proteins on the surIace oI the dendrites or certain cells oI the muscles
and glands, which are shaped to Iit only certain neurotransmitters.
reflex arc the connection oI the aIIerent neurons to the interneurons to the eIIerent neurons,
resulting in a reIlex action.
resting potential the state oI the neuron when not Iiring a neural impulse.
reticular formation
(RF)
an area oI neurons running through the middle oI the medulla and the pons and
slightly beyond that is responsible Ior general attention, alertness, and arousal.
reuptake process by which neurotransmitters are taken back into the synaptic vesicles.
sensory neuron a neuron that carries inIormation Irom the senses to the central nervous system. Also
called aIIerent neuron.
sensory pathway nerves coming Irom the sensory organs to the CNS consisting oI aIIerent neurons.
serotonin neurotransmitter involved in pain disorders and emotional perceptions. Is also known
as 5-hydroxytryptamine (5-HT).
single photon emission
tomography (SPECT)
neuroimaging method that is similar to PET but uses a diIIerent radioactive tracer
and can be used to examine brain blood Ilow.
soma the cell body oI the neuron responsible Ior maintaining the liIe oI the cell.
somatic nervous
system
division oI the PNS consisting oI nerves that carry inIormation Irom the senses to the
CNS and Irom the CNS to the voluntary muscles oI the body.
somatosensory cortex area oI neurons running down the Iront oI the parietal lobes responsible Ior
processing inIormation Irom the skin and internal body receptors Ior touch,
temperature, body position, and possibly taste.
The Biological Perspective 25 CHAPTER 2
The Biological Perspective 26 CHAPTER 2
spatial neglect condition produced by damage to the association areas oI the right hemisphere
resulting in an inability to recognize objects or body parts in the leIt visual Iield.
spinal cord a long bundle oI neurons that carries messages between the body and the brain and is
responsible Ior very Iast, liIesaving reIlexes.
stem cells special cells Iound in all the tissues oI the body that are capable oI becoming other
cell types when those cells need to be replaced due to damage or wear and tear.
sympathetic division
(fight-or-flight system)
part oI the ANS that is responsible Ior reacting to stressIul events and bodily arousal.
synapse (synaptic gap) microscopic Iluid-Iilled space between the synaptic knob oI one cell and the
dendrites or surIace oI the next cell.
synaptic knob rounded areas on the end oI the axon terminal.
synaptic vesicles saclike structures Iound inside the synaptic knob containing chemicals.
temporal lobes areas oI the cortex located just behind the temples containing the neurons responsible
Ior the sense oI hearing and meaningIul speech.
testes the male gonads.
thalamus part oI the limbic system located in the center oI the brain, this structure relays
sensory inIormation Irom the lower part oI the brain to the proper areas oI the cortex
and processes some sensory inIormation beIore sending it to its proper area.
thyroid gland endocrine gland Iound in the neck; regulates metabolism.
Wernicke`s aphasia condition resulting Irom damage to Wernicke`s area, causing the aIIected person to
be unable to understand or produce meaningIul language.

CHAPTER 3 - SENSATION AND PERCEPTION

YOU KNOW YOU ARE READY FOR THE TEST IF YOU ARE ABLE TO.
DeIine sensation and introduce some oI the key concepts developed by researchers in the study oI
sensation.
Explain in detail how our sense oI sight and our sense oI hearing work and discuss some causes
Ior impairments in these senses.
Discuss the chemical senses oI taste and smell and the lesser known somesthetic senses oI touch,
body position, and balance.
Describe our experience oI perception, especially in relation to visual stimuli.
Understand depth perception and the cues that Iacilitate this Iunction.

RAPID REVIEW
Sensation allows us to receive inIormation Irom the world around us. Synesthesia is the rare
condition in which a person experiences more than one sensation Irom a single stimulus, Ior example the
person who can hear and see a sound. The process oI converting an outside stimulus into the electrical-
chemical signal oI the nervous system is called sensory transduction. Outside stimuli (such as the sound
oI your mother`s voice) activate sensory receptors, which convert the outside stimulus into a message
that our nervous system can understandelectrical and chemical signals. The sensory receptors are
specialized Iorms oI neurons and make up part oI our somatic nervous system. Ernst Weber and Gustav
Fechner were two pioneers in the study oI sensory thresholds. Weber studied the smallest diIIerence
between two stimuli that a person could detect 50 percent oI the time. He called this diIIerence a just
noticeable difference (jnd), and he discovered that the jnd is always a constant. For instance, iI a person
needs to add 5 percent more weight to notice the diIIerence in the heaviness oI a package, then this
person`s jnd is 5 percent. II the initial weight oI the package is 10 lbs., then 0.5 lb. would need to be
added to detect a diIIerence (5 percent oI 10 lbs. 0.5 lb). II the initial weight is 100 lbs., then 5 lbs.
would need to be added in order Ior the person to detect a diIIerence in weight (5 percent oI 100 lbs. 5
lbs.). The Iact that the jnd is always a constant is known as Weber`s law. Fechner investigated the lowest
level oI a stimulus that a person could detect 50 percent oI the time. He called this level the absolute
threshold. The ability to see a candle Ilame Irom 30 miles away on a clear dark night is an example oI
the absolute threshold Ior human sight. In general, the body is interested in detecting changes in
environmental inIormation; constant stimuli tend to be ignored. Habituation and sensory adaptation are
two methods our body uses to ignore unchanging inIormation. Habituation takes place when the lower
centers oI the brain prevent conscious attention to a constant stimulus, such as the humming oI a desktop
computer. Sensory adaptation occurs in the sensory receptors themselves when the receptors stop
responding to a constant stimulus, such as the Ieeling oI your shirt on your skin. The visual system does
not adapt in this way, but rather, uses saccadic eye movements, tiny vibrations below the level oI our
conscious awareness, to allow the viewing oI unchanging stimuli.
The visual sensory system is activated by photons oI light that have speciIic wavelengths associated
with them. The three psychological aspects to our experience oI light are brightness, determined by the
height, or amplitude, oI the light wave; color, or hue, determined by the length oI the light wave; and
saturation, or purity, determined by the mixture oI wavelengths oI varying heights and lengths that make
up light. Light enters your eye through the cornea that protects your eye and helps to Iocus the light, and
then travels through a hole in your iris, called your pupil. The iris is a group oI muscles that control the
size oI the pupil; this is also the structure that gives us eye color. The light then passes through the lens,
which Iocuses the light and allows you to Iocus on objects that are close or Iar away. This process is
known as visual accommodation, the eIIiciency oI which typically decreases with age. The light then
travels through the vitreous humor in the middle oI your eyeball to reach the retina at the very back oI
your eye. The retina is approximately the size oI a postage stamp and contains the sensory receptor
neurons that convert the incoming light waves in to an electrical-chemical signal that the nervous system
can understand. Your eye contains two types oI sensory receptors, rods and cones. About 70 percent oI
Sensation and Perception CHAPTER 3 -27-
the sensory receptors in your eyes are rods. Rods detect the brightness oI light and send inIormation about
the levels oI black, white, and shades oI gray. The rods are located in the periphery oI the retina; that is,
almost everywhere except the very center. Rods are extremely sensitive to light but produce images with
low acuity, or sharpness. Our eyes` ability to adapt to a dark room and eventually see objects is mediated
by the rods in our eyes and is called dark adaptation. Cones make up the remaining 30 percent oI the
sensory receptors in your eyes and are located mainly in the center oI the retina. Cones transmit
inIormation about color and produce images with very high acuity. Our ability to quickly adapt when we
enter a bright room is called light adaptation and is accomplished by the cones. The place where the
inIormation Irom the rods and cones leaves the eye is called the blind spot because it contains no visual
receptors to receive inIormation.
AIter light is converted to an electrical-chemical signal by the rods and cones, the message passes
through the bipolar cells, to the ganglion cells, the axons oI which travel out oI the eye as the optic nerve.
At the optic chiasm, about 50 percent oI these axons, those associated with the halI oI the eye closest to
the nose, cross over and travel toward the visual cortex on the opposite side oI the brain. The remaining
50 percent oI the axons, those associated with the halI oI the eye closest to the ear, project to the visual
cortex on the same side oI the brain. From that point, the axons Iorming the optic nerve synapse at the
lateral geniculate nucleus oI the thalamus. From the thalamus the signal is sent to the occipital lobes,
which, iI you recall Irom the previous chapter, are responsible Ior processing visual inIormation.
The exact method the cones use to transmit inIormation about color is still unknown. Two theories
are currently proposed. The trichromatic theory was originally proposed by Thomas Young and later
modiIied by Hermann Helmholtz. The theory suggests the three types oI conesred, green, and blue
combine to produce sensation oI color much like three spotlights would combine to produce the Iull
spectrum oI colors. The trichromatic theory most likely is an accurate description oI the cones but cannot
explain certain visual phenomena such as the afterimage. The aIterimage is the image you see aIter
staring at something and then looking away. For example, stare at something red, then look away and you
see a green aIterimage. A diIIerent theory oI color perception known as the opponent-process theory
was developed to explain phenomena such as the aIterimage. The theory states that cones are arranged in
pairs with a red-green pair and a blue-yellow pair. II one member oI the pair is Iiring then the other
member cannot. When you stare at something red, the red member sends inIormation and the green
member is inhibited. When you look away, the green member is no longer inhibited and sends
inIormation even though you are not looking at anything green. Both the trichromatic theory and the
opponent-process theory are probably correct. The trichromatic theory most likely explains the actions oI
cones in the retina, while the opponent-process theory explains the actions higher up in the visual system
in the lateral geniculate nucleus oI the thalamus.
Color blindness is caused by deIective cones in the retina. Monochromatic color blindness occurs
when a person has no cones or no Iunctional cones; whereas, dichromatic color blindness exists when
only one cone is not working properly. Two types oI red-green colorblindness are common. A lack oI
Iunctioning red cones is called protanopia, and deIicient green cones result in deuteranopia. Blue-
yellow color deIiciency, tritanopia, is less common and is seen in individuals with nonIunctional blue
cones. The gene Ior color-deIicient vision is sex-linked and thus, more prevalent among males.
Our sense oI hearing, the auditory system, is activated by the vibrations oI molecules in the air that
surrounds us. These vibrations are called sound waves, and like light waves, we respond to three Ieatures
oI sound waves. Pitch corresponds to the Irequency oI the wave, volume is determined by the amplitude
oI the wave, and timbre relates to the purity oI the wavelengths. Humans can only respond to
wavelengths oI a certain Irequency. The average range Ior humans is between 20 and 20,000 Hertz (Hz)
or waves per second. Sound waves enter our auditory system through the pinna, travel down the ear
canalalso known as the auditory canaland then vibrate the eardrum, which causes the hammer,
anvil, and stirrup to vibrate. The vibrations oI the stirrup cause the oval window to move back and Iorth,
which causes the Iluid in the cochlea to vibrate. The Iluid causes the basilar membrane to vibrate, which
causes the organ oI Corti to brush against the membrane above it, and this causes the hair cells to bend.
The hair cells are the sensory receptors oI the auditory system, and the movement oI the hair cells triggers
Sensation and Perception CHAPTER 3 -28-
an action potential in axons, which travel to the brain in a bundle called the auditory nerve. A louder
noise causes the hair cells to Iire more action potentials.
Three theories explain how the brain receives inIormation about pitch. DiIIerent areas oI the basilar
membrane vibrate in response to diIIerent Irequencies oI sound waves. Place theory states that pitch is
determined by the place on the organ oI Corti that is stimulated. The frequency theory suggests that the
speed oI vibrations oI the basilar membrane determine the pitch heard by the person. The volley principle
suggests that hair cells take turns Iiring in a process called volleying. All three theories are correct.
Frequency theory holds true Ior wavelengths oI 100 Hz or less, volley theory covers the wavelengths
Irom 100 to 1000 Hz, and place theory seems to account Ior the wavelengths Iaster than 1000 Hz. Hearing
impairment is the term used to describe diIIiculties in hearing. Conduction hearing impairment occurs
Irom damage to the eardrum or the bones oI the middle ear. Nerve hearing impairment is caused by
problems in the inner ear or in the auditory pathways and cortical areas oI the brain. Ordinary hearing aids
are designed to assist with conduction hearing impairment, whereas cochlear implants can be used to
restore some hearing Ior people with nerve hearing impairment.
The sense oI taste, or gustation, is activated by chemicals that dissolve in the mouth. The sensory
receptors are receptor cells Iound within the taste buds that are located on the little bumps on the tongue,
cheek, and rooI oI your mouth. The little bumps that you can actually see with your eye are called
papillae. The average person has about 500 taste buds on his or her tongue. Five basic tastes have been
proposed; they are sweet, sour, salty, bitter, and umami. Umami is the newest taste and corresponds to a
'brothy taste like the taste Irom chicken soup. Some people, called 'supertasters, have 20 times the
typical number oI taste buds and are more sensitive to various Ilavors. Other research suggests that obese
individuals, as compared to those individuals who are not obese, might be less sensitive to sweet Ilavors.
For this reason, obese people might be particularly drawn to Ioods that are very sweet and high in Iat.
The sense oI smell, or olfaction, is also a chemical sense. Humans have about 10 million olIactory
receptor cells located in a 1 square inch area at the top oI the nasal passage. OlIactory receptor cells send
their axons directly to the olfactory bulbs, located right under the Irontal lobes. This pathway is unique in
that it bypasses the thalamus and instead goes directly to the olIactory bulbs, which are a part oI the brain.
From the olIactory bulbs, inIormation is sent to the primary olIactory cortex (also called the piriIorm
cortex), as well as brain areas associated with emotion (the orbitoIrontal cortex and amygdala).
The sense oI touch is actually composed oI several sensations and is more accurately reIerred to as
somesthetic senses. The three somesthetic senses are skin, kinesthetic, and vestibular. The skin contains
at least six diIIerent types oI sensory receptors and transmits inIormation about touch, pressure,
temperature, and pain. The currently accepted theory about pain is called gate-control theory and
suggests that pain inIormation is regulated by a number oI Iactors in the brain and spinal cord. Two
chemicals involved with pain messages are substance P and endorphins. Substance P transmits
inIormation about pain to the brain and spinal cord, while endorphins inhibit the transmission oI signals
oI pain. The kinesthetic sense relays inIormation about your body`s sense oI position in space. The
inIormation comes Irom sensory receptors called proprioceptive receptors located in your skin, joints,
muscles, and tendons. Our sense oI balance, or vestibular sense, is regulated by receptor cells in the
otolith organs and the semicircular canals. Both structures are located near the cochlea oI the inner ear.
The otolith organs contain small crystals suspended in Iluid. Movement oI the body causes the crystals to
move and activates the sensory receptors. Similarly, the semicircular canals are three Iluid-Iilled cavities
located in three diIIerent planes (x, y and z) through which the body can rotate. When the body moves, it
sets oII hair-like receptors in these canals. Sensory conflict theory describes a phenomenon by which
inIormation Irom the eyes may conIlict with inIormation Irom the vestibular system, causing dizziness.
Riding in a moving vehicle causes this type oI response in some people. Research suggests that these
symptoms may decrease Iollowing repeated exposure to the environment that causes motion sickness.
Perception is the interpretation oI sensation and seems to Iollow some basic principles, although
individual and cultural diIIerences in perception have been recorded. One principle is that oI perceptual
constancy. We tend to view objects as the same size, shape, and brightness even iI the sensations we are
receiving Irom our sensory systems are not constant in size, shape, or brightness. An example oI
perceptual constancy is our perception oI the size and shape oI a door as it is opened and closed. Gestalt
Sensation and Perception CHAPTER 3 -29-
psychologists believe that when people are presented with visual inIormation, they interpret the
inIormation according to certain expected patterns or rules. The patterns are called the Gestalt principles
oI perception, and they include the Iollowing seven rules: figure-ground relationships, closure,
similarity, continuity, contiguity, proximity, and common region. The principle oI Iigure-ground
relationships can be illustrated by looking at reversible figures, which are visual illusions in which the
Iigure and ground seem to switch back and Iorth.
Visual perception oI depth, called depth perception, appears to be present at a very early age. Visual
cues Ior depth that require the use oI one eye are reIerred to as monocular cues and include linear
perspective, relative size, overlap or interposition, aerial perspective, texture gradient, motion
parallax, and accommodation. Visual cues that use two eyes are called binocular cues and include
convergence and binocular disparity. An illusion is a perception that does not correspond to reality.
Illusions diIIer Irom hallucinations in that a hallucination`s origin is in the brain itselI; whereas, an
illusion is a distorted perception oI what`s really there. Some Iamous visual illusions include the
Hermann Grid, the Mller-Lyer illusion, the moon illusion, and illusions oI motion. The Rotating
Snakes illusion and the Enigma are two popular motion illusions. Research suggests that eye movements
called microsaccades might partially explain how motion illusions work.
In addition to cultural and individual diIIerences, perceptions can be inIluenced by perceptual sets or
expectancies. One example oI perceptual expectancy is top-down processing and occurs when a person
uses preexisting knowledge to Iit individual Ieatures into an organized whole. II there is no expectancy to
help organize inIormation, a person might use bottom-up processing to build a complete perception by
making sense oI the smaller Ieatures piece by piece. Magic acts are one Iamiliar application oI perceptual
principles, in which magicians use visual and cognitive illusions to 'Iool their audiences.
Sensation and Perception CHAPTER 3 -30-
STUDY HINTS
5.

Chapter 3 presented inIormation about seven diIIerent sensory systems. A chart can be
extremely helpIul in organizing these various components. See how much oI the inIormation you
can Iill in below and go to the textbook to Iind the remaining answers. The Iirst row is Iilled in
Ior you. A complete table can be Iound at the end oI the Study Hints section.

Sensory
System
External
Stimulus
Sensory
Organ
Sensory
Receptor
Proposed
Theories
visual svstem light waves eves rods and cones
trichromatic theorv
opponent-process
theorv







Sensation and Perception CHAPTER 3 -31-


6.


Many students conIuse the Gestalt principles oI perception with the monocular cues Ior depth
perception. The two lists are given here. The principles oI perception deal with the rules we
use to decide which objects should be grouped together, while the monocular depth cues are
used to determine how Iar away objects are.

Gestalt principles oI perception Monocular depth cues
closure linear perspective
similarity texture gradient
contiguity aerial position
continuity interposition
Iigure-ground relationship motion parallax
proximity relative size

In order to help clariIy the diIIerence, use these cues to draw two separate pictures.

Use one or more oI the Gestalt principles to
create a picture with at least two separate
groups oI objects.

Use one or more oI the monocular depth
cues to draw a picture oI a tree, house and a
person. Make sure the tree is the Iarthest
object and the person is the closest object.



Sensation and Perception CHAPTER 3 -32-

Completed Sensorv Svstem Chart
Sensory
System
External
Stimulus
Sensory
Organ
Sensory
Receptor
Proposed
Theories
visual svstem light waves eves rods and cones
trichromatic theorv
opponent-process
theorv
auditorv svstem sound waves ears
hair cells in the
organ of Corti
place theorv
frequencv theorv
vollev theorv
gustatorv svstem
(taste)
soluble
chemicals
tongue,
cheeks,
mouth
taste cells in the
taste buds

olfactorv svstem
(smell)
air-borne
chemicals
nose
olfactorv
receptors

skin senses
pressure,
temperature,
pain
skin
six different
tvpes including
free nerve
endings and
pacinian
corpuscles
gate-control theorv
of pain
kinesthetic bodv position
skin, foints,
muscles, and
tendons
proprioceptive
receptors

vestibular
acceleration and
tilt
semicircular
canals and
otolith
organs
hair cells


LEARNING OBJECTIVES
3.1 How does sensation travel through the
central nervous svstem, and whv are some
sensations ignored?
3.2 What is light, and how does it travel
through the various parts of the eve?
3.3 How do the eves see, and how do the eves
see different colors?
3.4 What is sound, and how does it travel
through the various parts of the ear?
3.5 Whv are some people unable to hear, and
how can their hearing be improved?
3.6 How do the senses of taste and smell work,
and how are thev alike?


3.7 What allows people to experience the sense
of touch, pain, motion, and balance?
3.8 What are perception and perceptual
constancies?
3.9 What are the Gestalt principles of
perception?
3.10 How do infants develop perceptual
abilities, including the perception of depth
and its cues?
3.11 What are visual illusions, and how can thev
and other factors influence and alter
perception?


Sensation and Perception CHAPTER 3 -33-
PRACTICE EXAM
For the Iollowing multiple choice questions, select the option you Ieel best answers the question or
completes the sentence.

1. The most important role oI sensory receptors is to
a) coordinate communications within the body.
b) regulate the body`s response to pain.
c) control skeletal muscle contractions.
d) convert an external stimulus into an electrical-chemical message the nervous system can use.

2. The point at which a person can detect a stimulus 50 percent oI the time it is presented is called the
a) absolute threshold.
b) range threshold.
c) diIIerential threshold.
d) noticeable threshold.

3. An automobile manuIacturer has decided to add a little bit oI horsepower to its cars. They have a
device that alters horsepower one unit at a time. Suppose drivers Iirst notice the increase on a 200
horsepower car when it reaches 220 horsepower. How much horsepower must be added to a 150
horsepower car Ior drivers to notice the diIIerence?
a) 5
b) 10
c) 15
d) 25

4. II you stared at a picture Ior a long period oI time, you might think the image oI the picture would Iade
due to sensory adaptation. This would be the case except Ior the tiny vibrations oI your eye called
a) glissades.
b) saccades.
c) habituation movements.
d) light wave responses.

5. Light is said to have a dual nature, meaning it can be thought oI in two diIIerent ways. These two
ways are
a) particles and photons.
b) waves and Irequencies.
c) photons and waves.
d) dark light and daylight.

6. When light waves enter the eye, they Iirst pass through the
a) iris.
b) lens.
c) pupil.
d) cornea.

7. Which oI the Iollowing is true about cones?
a) They are more sensitive to light than rods.
b) They are Iound mainly in the center oI the eye.
c) They operate mainly at night.
d) They respond only to black and white.

Sensation and Perception CHAPTER 3 -34-
8. The existence oI aIterimages in complementary colors best supports the theory oI color vision.
a) opponent-process
b) place
c) vibrational
d) Hering trichromatic

9. Which oI the Iollowing properties oI sound would be the most similar to the color or hue oI light?
a) pitch
b) loudness
c) purity
d) timbre

10. Vibrating molecules in the air are called
a) light waves.
b) sound waves.
c) odor waves.
d) taste sensations.

11. The membrane stretched over the opening to the middle ear is the
a) pinna.
b) oval window.
c) tympanic membrane.
d) cochlea.

12. Which is the correct order oI the three bones oI the middle ear, Irom the outside in?
a) anvil, hammer, stirrup
b) hammer, anvil, stirrup
c) stirrup, anvil, hammer
d) stirrup, hammer, anvil

13. Which theory proposes that above 100 Hz but below 1000Hz, auditory neurons do not Iire all at
once but in rotation?
a) place theory
b) volley theory
c) Irequency theory
d) rotational theory

14. The theory explains how we hear sounds above 1,000 Hz.
a) place
b) Irequency
c) volley
d) adaptive

15. In the auditory system, transduction occurs in the
a) hair cells oI the cochlea.
b) hair cells oI the semicircular canals.
c) hair cells oI the olIactory epithelium
d) hair cells oI the papillae.

Sensation and Perception CHAPTER 3 -35-
16. Which oI the Iollowing items describes a Iunction oI the bones oI the middle ear?
a) They transIorm sound waves to into an electrical signal.
b) They ampliIy the vibrations oI the ear drum.
c) They Iunnel sound waves Irom the outside world into the ear.
d) They provide inIormation about movement in three planes (x, y and z).

17. II a severe ear inIection damages the bones oI the middle ear, you may develop hearing
impairment.
a) nerve
b) stimulation
c) brain pathway
d) conduction

18. Cochlear implants bypass the
a) outer ear.
b) outer and middle ear.
c) outer, middle, and inner ear.
d) auditory nerve.

19. The 'bumps on the tongue that are visible to the eye are the
a) olIactory receptors.
b) taste buds.
c) papillae.
d) taste receptors.

20. An olIactory stimulus travels Irom receptor to
a) olIactory bulb.
b) thalamus.
c) amygdala.
d) pons.

21. According to your textbook, what is the best current explanation Ior how the sensation oI pain
works?
a) the sensory conIlict theory
b) the gate control theory
c) the volley principle
d) congenital analgesia

22. Which is the best description oI the vestibular senses?
a) having to do with touch, pressure, temperature, and pain
b) having to do with the location oI body parts in relation to each other
c) having to do with movement and body position
d) having to do with your location as compared to the position oI the sun

23. We know when we are moving up and down in an elevator because oI the movement oI tiny
crystals in the
a) outer ear.
b) inner ear.
c) otolith organs.
d) middle ear.

Sensation and Perception CHAPTER 3 -36-
24. Proprioceptors aid in the process oI
a) perception oI visceral pain.
b) perception oI the spatial location oI body parts in relation to one another.
c) perception oI odor molecules in the air.
d) perception oI somatic pain.

25. The tendency to interpret an object as always being the same size, regardless oI its distance Irom
the viewer, is known as
a) size constancy.
b) shape constancy.
c) brightness constancy.
d) color constancy.

26. Closure is the tendency
a) to perceive objects, or Iigures, on some background.
b) to complete Iigures that are incomplete.
c) to perceive objects that are close to each other as part oI the same grouping.
d) to perceive things with a continuous pattern rather than with a complex, broken-up
pattern.

27. Which Gestalt principle is at work in the old phrase, 'birds oI a Ieather Ilock together?
a) closure
b) similarity
c) expectancy
d) continuity

28. Visual distance and depth cues that require the use oI both eyes are called
a) monocular cues.
b) diocular cues.
c) binocular cues.
d) dichromatic cues.

29. The Mller-Lyer illusion exists in cultures with
a) more men than women.
b) more women than men.
c) Iew buildings.
d) buildings with lots oI corners.

30. People`s tendency to perceive things a certain way because their previous experiences or
expectations inIluence them is called
a) a perceptual set.
b) binocular disparity.
c) motion parallax.
d) accommodation.

31. When you look at a Ilowing river, the width oI the river seems to converge into a point in the
distance. This phenomenon is called
a) a monocular cue.
b) a pictorial depth cue.
c) linear perspective.
d) all oI the above are correct.
PRACTICE EXAM ANSWERS
Sensation and Perception CHAPTER 3 -37-
1. d Sensory receptors are the body`s 'antennae to the outside world. Each sensory
receptor type is specially designed to receive a speciIic external signal and convert
it to an electrical-chemical signal that the brain can understand.
2. a Gustav Fechner investigated the sensitivity oI the human sensory systems and
called the lowest level oI a stimulus that a person could detect halI oI the time the
absolute threshold.
3. c According to Weber`s law, the just noticeable diIIerence (jnd) is a constant
proportion. A change Irom 200 to 220 represents an increase oI 20 units and a jnd oI
20/200 or 0.10, which is 10 percent. II the company starts with 150 horsepower,
they will need to increase it by 10 percent in order Ior the driver to notice a
diIIerence. Ten percent oI 150 is 15.
4. b Saccades are the small quick movements your eye makes in order to keep the visual
stimuli changing. When our sensory receptors receive unchanging, constant stimuli,
they eventually stop responding to the stimulus. This process is known as sensory
adaptation.
5. c Light can be thought oI as a wave and as particles. Photons are the speciIic type oI
particles oI which light is composed.
6. d The cornea is the outermost coating oI the eye. It is transparent and serves to protect
the eye and to help Iocus the light coming in to the eye.
7. b Cones are the sensory receptors that respond to color and send visual inIormation oI
high acuity or visual sharpness. The cones are located primarily in the center oI the
retina. Rods are more sensitive than cones to light, and they respond only to black
and white.
8. a The opponent process theory oI color vision was introduced, in part, to explain the
phenomenon oI the aIterimage.
9. a Both light and sound travel in waves. Pitch is determined by the length oI the sound
wave just as color is determined by the length oI the light wave. Both brightness
and loudness are determined by the height oI their respective waves.
10. b The outer and middle parts oI the ear are designed to Iunnel the vibrating air
molecules to the inner ear where they are translated into an electrical signal and sent
to the brain.
11. c The tympanic membrane is also known as the eardrum. Sound waves cause the
tympanic membrane to vibrate, which then causes the bones oI the middle ear to
move back and Iorth.
12. b The order oI the bones is hammer, anvil, stirrup which spells 'has.
13. b Volley theory describes the perception oI pitch Ior the middle Irequencies (100
1000 Hz). Frequency theory describes the low Irequencies (100 Hz and less), and
place theory describes the Iastest Irequencies (1000 Hz and higher).
14. a The idea is that at very high sound Irequencies, the action potential Irequency can`t
keep up, so pitch has to be coded by the place on the basilar membrane that is
activated.
15. a Sound waves result in the vibration oI the basilar membrane, which causes the hair
cells oI the cochlea to brush against the membrane above them. In this way, sound
waves are transIormed into an electrical signal (action potential) that the brain can
understand. Although hair cells exist in the vestibular system and hair-like cilia
exist in the olIactory system, neither are involved in auditory transduction. The
papillae are visible components oI the gustatory system and contain the taste buds
rather than hair cells.
Sensation and Perception CHAPTER 3 -38-

16. b The vibration oI the three bones oI the middle ear, the hammer, anvil and stirrup,
ampliIy the vibrations Irom the ear drum. The hair cells oI the cochlear transIorm
sound waves into an electrical signal. The pinna Iunnels sound waves Irom the
outside world into the inner ear. The semicircular canals provide inIormation about
movement in the x, y and z planes.
17. d Conduction hearing impairment is caused by damage to the outer or middle ear.
18. b Cochlear implants use an electronic device instead oI the movements oI the bones in
the middle ear to convert the sound wave into a signal that is then sent to the
auditory nerve in the inner ear.
19. c The bumps you can see with your eye are the papillae. The taste buds are located
along the sides oI the papillae. Each taste bud contains 1020 taste receptors.
20. a The olIactory system is the only system in which the receptors send their signal
directly to the higher brain and bypass the thalamus, which normally acts as a Iilter
Ior the processes oI the lower brain.
21. b According to your textbook, the gate-control theory oI pain is the best current
explanation Ior how pain works. The sensory conIlict theory applies to motion
sickness; the volley principle explains the perception oI sound rather than pain.
Although congenital analgesia is related to pain, it reIers to a condition in which
people cannot Ieel pain.
22. c The vestibular sense provides you with a sense oI balance and sends your brain
inIormation about acceleration and tilt.
23. d Although the otolith organs are located in the inner ear, the inner ear is a more
precise answer.
24. b Proprioceptors aid in the perception oI the spatial location oI body parts in relation
to one another. The term reIerring to the perception oI smell is olIaction.
25. a Size constancy reIers to the Iact that our perception oI the size oI an object tends to
remain constant.
26. b Closure is one oI the Gestalt principles oI perception and reIers to our tendency to
'close objects to Iorm a complete picture.
27. b The saying is emphasizing that objects with similar characteristics ('birds oI a
Ieather) tend to be grouped together ('Ilock together). This is the principle oI
similarity.
28. c The phrase 'ocular means having to do with the eyes. 'Mono reIers to one and
'bi reIers to two. ThereIore, the term binocular means seeing depth with two eyes.
29. d The carpentered-world theory states that the Mller-Lyer illusion does not exist in
certain 'primitive cultures because they are not surrounded by straight lines and
corners.
30. a An individual`s expectations or perceptual set oIten inIluence perception oI objects.
31. d All oI the answers are correct. Linear perspective is a type oI monocular cue that
can be used to add depth to pictures; thus, it is also a pictorial depth cue.

CHAPTER GLOSSARY
absolute threshold the lowest level oI stimulation that a person can consciously detect 50
percent oI the time the stimulation is present.
accommodation as a monocular clue, the brain`s use oI inIormation about the changing
thickness oI the lens oI the eye in response to looking at objects that are
close or Iar away.
aerial (atmospheric)
perspective
the haziness that surrounds objects that are Iarther away Irom the viewer,
causing the distance to be perceived as greater.
afterimages images that occur when a visual sensation persists Ior a brieI time even
aIter the original stimulus is removed.
Sensation and Perception CHAPTER 3 -39-
auditory canal short tunnel that runs Irom the pinna to the eardrum.
auditory nerve bundle oI axons Irom the hair cells in the inner ear.
binocular cues cues Ior perceiving depth based on both eyes.
binocular disparity the diIIerence in images between the two eyes, which is greater Ior objects
that are close and smaller Ior distant objects.
blind spot area in the retina where the axons oI three layers oI retinal cells exit the eye
to Iorm the optic nerve.
bottom-up processing the analysis oI the smaller Ieatures to build up to a complete perception.
brightness constancy the tendency to perceive the apparent brightness oI an object as the same
even when the light conditions change.
closure the tendency to complete Iigures that are incomplete.
cochlea snail-shaped structure oI the inner ear that is Iilled with Iluid.
cones visual sensory receptor Iound at the back oI the retina, responsible Ior color
vision and sharpness oI vision.
contiguity the tendency to perceive two things that happen close together in time as
being related.
continuity the tendency to perceive things as simply as possible with a continuous
pattern rather than with a complex, broken-up pattern.
convergence the rotation oI the two eyes in their sockets to Iocus on a single object,
resulting in greater convergence Ior closer objects and less convergence iI
objects are distant.
dark adaptation the recovery oI the eye`s sensitivity to visual stimuli in darkness aIter
exposure to bright lights.
depth perception the ability to perceive the world in three dimensions.
figure-ground the tendency to perceive objects, or Iigures, as existing on a background.
frequency theory theory oI pitch that states that pitch is related to the speed oI vibrations in
the basilar membrane.
gustation (gustatory system) the sensation oI taste.
habituation tendency oI the brain to stop attending to constant, unchanging inIormation.
hertz (Hz) cycles or waves per second, a measurement oI Irequency.
just noticeable difference
(difference threshold)
the smallest diIIerence between two stimuli that is detectable 50 percent oI
the time.
kinesthetic senses sense oI the location oI body parts in relation to the ground and each other.
light adaptation the recovery oI the eye`s sensitivity to visual stimuli in light aIter exposure
to darkness.
linear perspective the tendency Ior parallel lines to appear to converge on each other.
monocular cues (pictorial
depth cues)
cues Ior perceiving depth based on one eye only.
motion parallax the perception oI motion oI objects in which close objects appear to move
more quickly than objects Iarther away.
Mller-Lyer illusion illusion oI line length that is distorted by inward-turning or outward-turning
corners on the ends oI the lines, causing lines oI equal length to appear to
be diIIerent.
olfaction (olfactory system) the sensation oI smell.
olfactory bulbs areas oI the brain located just above the sinus cavity and just below the
Irontal lobes that receive inIormation Irom the olIactory receptor cells.
opponent-process theory theory oI color vision that proposes visual neurons (or groups oI neurons)
are stimulated by light oI one color and inhibited by light oI another color.
overlap (interposition) the assumption that an object that appears to be blocking part oI another
object is in Iront oI the second object and closer to the viewer.
Sensation and Perception CHAPTER 3 -40-

perception the method by which the sensations experienced at any given moment are
interpreted and organized in some meaningIul Iashion.
perceptual set (perceptual
expectancy)
the tendency to perceive things a certain way because previous experiences
or expectations inIluence those perceptions.
pinna the visible part oI the ear.
pitch psychological experience oI sound that corresponds to the Irequency oI the
sound waves; higher Irequencies are perceived as higher pitches.
place theory theory oI pitch that states that diIIerent pitches are experienced by the
stimulation oI hair cells in diIIerent locations on the organ oI Corti.
proximity the tendency to perceive objects that are close to each other as part oI the
same grouping.
relative size perception that occurs when objects that a person expects to be oI a certain
size appear to be small and are, thereIore, assumed to be much Iarther
away.
reversible figures visual illusions in which the Iigure and ground can be reversed.
rods visual sensory receptor Iound at the back oI the retina, responsible Ior
noncolor sensitivity to low levels oI light.
sensation the process that occurs when special receptors in the sense organs are
activated, allowing various Iorms oI outside stimuli to become neural
signals in the brain.
sensory adaptation tendency oI sensory receptor cells to become less responsive to a stimulus
that is unchanging.
sensory conflict theory an explanation oI motion sickness in which the inIormation Irom the eyes
conIlicts with the inIormation Irom the vestibular senses, resulting in
dizziness, nausea, and other physical discomIort.
shape constancy the tendency to interpret the shape oI an object as being constant, even
when its shape changes on the retina.
similarity the tendency to perceive things that look similar to each other as being part
oI the same group.
size constancy the tendency to interpret an object as always being the same actual size,
regardless oI its distance.
skin senses the sensations oI touch, pressure, temperature, and pain.
somesthetic senses the body senses consisting oI the skin senses, the kinesthetic sense, and the
vestibular senses.
synesthesia disorder in which the signals Irom various sensory organs are processed in
the wrong cortical areas, resulting in the sense inIormation being
interpreted as more than one sensation.
texture gradient the tendency Ior textured surIaces to appear to become smaller and Iiner as
distance Irom the viewer increases.
top-down processing the use oI preexisting knowledge to organize individual Ieatures into a
uniIied whole.
transduction the process oI converting outside stimuli, such as light, into neural activity.
trichromatic theory theory oI color vision that proposes three types oI cones: red, blue, and
green.
vestibular senses the sensations oI movement, balance, and body position.
visual accommodation the change in the thickness oI the lens as the eye Iocuses on objects that are
Iar away or close.
volley principle theory oI pitch that states that Irequencies about 400 Hz to 4000 Hz cause
the hair cells (auditory neurons) to Iire in a volley pattern, or taking turns in
Iiring.
Sensation and Perception CHAPTER 3 -41-
Sensation and Perception CHAPTER 3 -42-

CHAPTER 4 - CONSCIOUSNESS: SLEEP, DREAMS, HYPNOSIS, AND DRUGS

YOU KNOW YOU ARE READY FOR THE TEST IF YOU ARE ABLE TO.
DeIine consciousness and discuss the diIIerent levels oI consciousness.
Explain the Iactors that control sleep, theories on the purpose oI sleep, the stages oI sleep, and
disorders oI sleep.
Discuss dreams and three theories that attempt to explain the purpose oI dreams.
Introduce the phenomenon oI hypnosis and theories suggesting the underlying mechanism.
Describe properties and potential dangers oI psychoactive drugs including stimulants,
depressants, narcotics, and hallucinogens.
Understand how hypnogogic and hypnopompic hallucinations might be misinterpreted as
supernatural events.

RAPID REVIEW
Consciousness is deIined as a person`s awareness oI the world around him or her; this awareness
may be used to organize behavior. Waking consciousness is deIined as the state oI awareness where our
thoughts and Ieelings are clear and organized. Altered states of consciousness describe a shiIt in the
quality or pattern oI a person`s awareness. Examples oI altered states oI consciousness include using
drugs, daydreaming, being hypnotized, or simply sleeping.
The sleepwake cycle is a circadian rhythm, meaning one cycle takes about a day to complete. The
cycle is regulated by the suprachiasmatic nucleus (SCN) located in the hypothalamus. The SCN
responds to changes in daylight and regulates body temperature and the release oI melatonin Irom the
pineal gland. By the end oI the day, lower body temperature and higher melatonin levels cause people to
Ieel sleepy. In addition, high levels oI serotonin are believed to produce Ieelings oI sleepiness. The sleep
wake cycle tends to shiIt to a 25-hour cycle when subjects do not have access to the sun or clocks. Sleep
deprivation, or loss oI sleep, results in an increase in microsleeps (brieI periods oI sleep lasting only a
Iew seconds), concentration problems, and an inability to perIorm simple tasks. Participants in a number
oI sleep deprivation studies reported that they were unaware oI their impaired Iunctioning. Two theories
are currently proposed Ior why we sleep. The adaptive theory suggests that we sleep to avoid predators,
while the restorative theory states that sleep is needed to replenish chemicals and repair cellular damage.
Both theories are probably partially correct.
Based on brain wave activity recorded with the use oI an EEG, sleep has been divided into two
diIIerent types, rapid eye movement (REM) sleep and non-REM sleep. Non-REM sleep is a deep,
restIul sleep and consists oI Iour stages. Stage 1 sleep is also called light sleep and occurs when brain
activity begins to shiIt Irom alpha to theta wave activity. Many people experience a hypnic jerk in this
stage when their body jerks suddenly and wakes them up. As body temperature continues to drop and
heart rate slows, sleep spindles begin to appear on the EEG recording, signaling Stage 2 oI non-REM
sleep. Stage 3 occurs when the slow, large delta waves Iirst appear. When delta waves account Ior more
than 50 percent oI the total brain activity, the person is said to be in Stage 4, the deepest stage oI sleep.
AIter a person cycles through Stages 14, instead oI entering Stage 1, people experience REM sleep.
During this type oI sleep, the brain is active and displays beta wave activity, the eye exhibits rapid
movements, and the skeletal muscles oI the body are temporarily paralyzed. This paralysis is reIerred to
as REM paralysis. When a person is wakened Irom this type oI sleep they oIten report being in a dream
state. Most likely, around 90 percent oI dreams take place in REM sleep, although dreams also do occur
in non-REM sleep. Contrary to popular belieI, people do not go crazy when deprived oI REM sleep;
however, they do spend longer amounts oI time in REM sleep when allowed to sleep normally again. This
phenomenon is known as REM rebound. Nightmares are bad dreams and typically occur in REM sleep.
REM behavior disorder is a rare disorder in which a person`s muscles are not paralyzed during REM
sleep, allowing them to thrash about and even get up and act out their dreams.
A large number oI disorders are associated with sleep. Sleepwalking, or somnambulism, occurs in
Stage 4, as does the rare disorder oI night terrors. Most people state that they are not aware oI the actions
Consciousness CHAPTER 4 -43-
they committed during a sleepwalking episode. The explanation oI 'sleepwalking has been used as a
successIul deIense in several trials Ior murder, but in these cases, the term sleepwalking is more likely
reIerring to the condition known as REM behavior disorder. Insomnia is the inability to Iall asleep, stay
asleep, or get a good night oI quality sleep. Sleep apnea is a disorder in which a person actually stops
breathing Ior brieI periods throughout the night. Narcolepsy is a genetic disorder in which a person
suddenly enters REM sleep during the day. Such attacks can occur many times throughout the day and
without warning. The attacks oIten occur with cataplexy, or a sudden loss oI muscle tone.
Several theories have been proposed to explain why dreams occur. Sigmund Freud believed that dreams
represented our unconscious thoughts and desires. He called the actual content oI a dream the manifest content
and the real meaning oI the dream the latent content. The activation-synthesis hypothesis was originally
proposed by Hobson and McCarley and suggests that dreams occur when the cerebral cortex attempts to Iit
together (or synthesize) random neural input Irom the lower brain structures. The activation-information mode
model (AIM) expands on the activation-synthesis model in an attempt to explain the meaningIul, realistic, and
consistent nature oI many dreams. AIM proposes that the cortex uses inIormation Irom the previous days as it
pieces together the input coming Irom the lower brain structures. A considerable amount oI inIormation is
known about the content oI dreams. Most dreams tend to reIlect events in everyday liIe as well as the
'personality oI the dreamer`s culture. Men tend to dream about weapons, tools, cars, roads, and other males.
Further, their dreams tend to occur in outdoor or unIamiliar settings and contain more physical aggression as
compared to women`s dreams. Men also report more sexual dreams. Women tend to dream about men and
women equally, and they also are more likely to report dreams about people they know, Iamily, home, concerns
about their appearances, and dreams in which they are the victims oI aggressive acts. Dreams oI being naked in
public appear to be common in many cultures.
Hypnosis is a state oI consciousness in which a person is especially susceptive to suggestion.
Hypnosis can reduce the sensation oI pain, create temporary states oI amnesia, and aIIect sensory
perception. However, it cannot increase physical strength, enhance memory, or regress a person back to
their childhood. One theory oI hypnosis proposed by Ernst Hilgard suggests that the hypnotized person is
in a state oI dissociation with one part oI the brain unaware oI the activities happening under hypnosis
and another part aware and simply watching what is happening. Hilgard called the part oI the
consciousness that was aware oI the activities the hidden observer. The social-cognitive theory oI
hypnosis states that people who are hypnotized are not in an altered state but are simply playing the role
they Ieel is expected oI them in the situation.
A psychoactive drug is any drug that alters a person`s thinking, perception, or memory. Physical
dependence on a drug occurs when the user`s body does not Iunction normally without the drug. Two
signs oI physical dependence are drug tolerance and symptoms oI withdrawal when deprived oI the drug.
Psychological dependence occurs when a drug is needed to maintain a Ieeling oI emotional or
psychological well-being. Psychoactive drugs can be classiIied into major categories including stimulants,
depressants, narcotics, and psychogenic drugs.
Stimulants are a class oI drugs that increase the activity oI the nervous system and the organs
connected to it. SpeciIically, stimulants activate the Iight-or-Ilight response oI the sympathetic nervous
system. Amphetamines are human-manuIactured stimulants and include drugs such as benzedrine,
methedrine, and dexedrine. Large doses oI amphetamines can lead to a severe mental disturbance and
paranoia called amphetamine psychosis. Cocaine is a naturally occurring stimulant Iound in the leaves oI
the coca plant. Cocaine produces Ieelings oI happiness, energy, power, and pleasure and also reduces pain
and suppresses appetite. Cocaine is highly addictive and can cause convulsions and death even in Iirst-
time users. Signs oI cocaine abuse include compulsive use, loss oI control, and disregard Ior the
consequences oI use. Nicotine is a mild yet toxic, naturally occurring stimulant that raises blood pressure,
accelerates the heart, and provides a rush oI sugar into the bloodstream. Nicotine has been Iound to be
more addictive than heroin or alcohol and is linked to nearly 430,000 deaths in the United States each
year. Caffeine is a third naturally occurring stimulant that increases alertness and can enhance the
eIIectiveness oI certain pain relievers.
Depressants are drugs that slow down the central nervous system and include barbiturates,
benzodiazepines, and alcohol. Barbiturates have a strong sedative, or sleep-inducing, eIIect and are
Consciousness CHAPTER 4 -44-
known as the major tranquilizers. The minor tranquilizers, or benzodiazepines, have a relatively minor
depressant eIIect and are used to lower anxiety and reduce stress. Some common benzodiazepines include
Valium, Xanax, Halcion, Ativan, Librium, and Rohypnol (also known as the date rape drug). The most
commonly used and abused depressant is alcohol.
Narcotics reduce the sensation oI pain by binding to and activating the receptor sites Ior endorphins. All
narcotics are at least partially derived Irom the plant-based substance oI opium. Opium itselI is made Irom
the opium poppy and reduces pain as well as increases Ieelings oI well-being. Morphine is made Irom opium
and is used Ior the short-term relieI oI severe pain. Due to its highly addictive nature, the use oI morphine is
careIully controlled. Heroin is also partially derived Irom opium but is not used as a medicine due to the Iact
that it is more addictive than morphine or opium. Narcotics are thought to be so addictive because they mimic
the action oI endorphins and subsequently cause the body to stop producing its own endorphins so that
without the drug, there is no protection Irom pain. Methadone is a synthetic opioid that does not produce the
Ieelings oI euphoria produced by morphine and heroin. Methadone can be used to attempt to control heroin
dependency. In addition to methadone treatment, heroin addiction is treated with behavioral therapies such as
contingency management therapies and cognitive approaches such as cognitive-behavioral interventions.
Hallucinogens are psychogenic drugs that create Ialse sensory perceptions, also known as
hallucinations. Lysergic acid diethylamide (LSD) is synthesized Irom a grain Iungus and is one oI the
most potent hallucinogens. Phenyl cyclohexyl piperdine or PCP is a synthesized drug that can act as a
hallucinogen, stimulant, depressant, or analgesic depending on the dosage. PCP has also been shown to
lead to acts oI violence against others or suicide. MDMA or Ecstasy shares some chemical characteristics
with amphetamines and also produces hallucinations. Because oI their stimulant and hallucinogenic
properties, PCP and MDMA are now classiIied as stimulatory hallucinogenics. Naturally occurring
hallucinogenics include mescaline, psilocybin, and marijuana. The physiological eIIects oI marijuana
may be milder than other hallucinogens, yet marijuana use can lead to a powerIul psychological
dependency.
Hvpnogogic hallucinations may occur during Stage 1 sleep; whereas, hvpnopompic hallucinations tend
to occur upon awakening Irom REM sleep. The vivid nature oI these hallucinations may cause some people
to misinterpret such experiences as ghostly visitations or alien abductions.

STUDY HINTS
7.

Use the space below to create a visual summary oI the brain wave and physiological changes
that occur as your body moves Irom an awake state through the stages oI sleep typical Ior one
night oI sleep. Use arrows to indicate the progression through the stages throughout the course oI
a night.
Stage Brain wave activity Other descriptions
Awake

non-REM Stage 1

non-REM Stage 2

non-REM Stage 3

non-REM Stage 4

REM


Consciousness CHAPTER 4 -45-

8.


The textbook introduces six diIIerent sleep disorders. Pretend that you have each oI the sleep
disorders and write a brieI description oI a particular episode you experienced due to the
disorder.

sleepwalking I dont remember anvthing that happened but in the morning mv mother
told me that about 50 minutes after I had fallen asleep (right when I
would be in the deepest stage of sleep, Stage 4) I walked past her in the
kitchen and I was carrving a bath towel. I put the towel in the
refrigerator, looked right at her, and then went back to bed in mv
bedroom. Supposedlv I do this tvpe of thing quite often.

night terrors








REM behavior
disorder








insomnia








apnea








narcolepsy







Consciousness CHAPTER 4 -46-

Suggested solutions for Question 1

Stage Brain wave activity Other descriptions
Awake beta
non-REM Stage 1 alpha hypnic jerk occurs here
non-REM Stage 2 theta sleep spindles are seen in this stage
non-REM Stage 3 delta waves initial appearance oI delta waves,
they make up minority oI brain wave
activity
non-REM Stage 4 more than 50 percent
delta waves
deepest stage oI sleep, hardest to
wake the person up, sleepwalking and
night terrors occur in this stage
REM beta skeletal muscles are paralyzed
(except Ior people with REM
behavior disorder), eyes dart back and
Iorth rapidly below the eyelids

LEARNING OBJECTIVES
4.1 What does it mean to be conscious, and are
there different levels of consciousness?
4.2 Whv do people need to sleep, and how does
sleep work?
4.3 What are the different stages of sleep,
including the stage of dreaming and its
importance?
4.4 How do sleep disorders interfere with
normal sleep?
4.5 Whv do people dream, and what do thev
dream about?
4.6 How does hvpnosis affect consciousness?

4.7 What is the difference between a phvsical
dependence and a psvchological dependence
on a drug?
4.8 How do stimulants and depressants affect
consciousness, and what are the dangers
associated with taking them, particularlv
alcohol?
4.9 What are some of the effects and dangers of
using narcotics and hallucinogens, including
marifuana?
4.10 What are hvpnogogic and hvpnopompic
hallucinations?

PRACTICE EXAM
For the Iollowing multiple choice questions, select the answer you Ieel best answers the question.

1.

What term do psychologists use to designate our personal awareness oI Ieelings, sensations, and
thoughts?
a) thinking
b) cognition
c) conscience
d) consciousness

2. A biological cycle, or rhythm, that is approximately 24 hours long is a(n) cycle.
a) inIradian
b) circadian
c) diurnal
d) ultradian

Consciousness CHAPTER 4 -47-
3. The hormone melatonin reaches peak levels in the body during the
a) morning.
b) early evening.
c) aIternoon.
d) night.

4. Sid is taking part in research on the eIIects oI sleep deprivation; he has been without sleep
Ior 75 hours. Right now researchers have asked him to sit in Iront oI a computer screen and hit
a button each time he sees the letter "S" on the screen. A Iew days ago, Sid was a whiz at this
task; however, he is doing very poorly today. How are sleep researchers likely to explain Sid's
poor perIormance?
a) Due to the sleep deprivation, Sid does not understand the task.
b) Microsleeps are occurring due to the sleep deprivation, and he is asleep Ior brieI periods
oI time.
c) He is determined to ruin the research because oI the suIIering he is enduring at the hands
oI the researchers.
d) He is probably dreaming that he is somewhere else and has no interest in responding to the
"here and now."

5. According to this theory, sleep is a product oI evolution.
a) restorative theory
b) adaptive theory
c) psychoanalytic theory
d) dream theory

6. II the EEG record reveals evidence oI very small and very Iast waves, you are likely to conclude
that the sleeping person is
a) really not sleeping and is awake.
b) in Stage 2.
c) in Stage 3.
d) in Stage 4.

7. Each oI the Iollowing is true oI sleepwalking EXCEPT
a) more boys than girls sleepwalk.
b) sleepwalking is more common among children than adults.
c) waking a sleepwalker is diIIicult.
d) waking a sleepwalker is dangerous.

8. For several months, Ted has been taking increasingly larger doses oI barbiturate sleeping pills to
treat insomnia. He just decided to quit taking any barbiturate sleeping pills. What is likely to happen
to Ted when he stops taking the barbiturate sleeping pills?
a) He will become depressed.
b) He will experience the REM rebound.
c) He will increase his intake oI caIIeine.
d) He will suIIer the symptoms oI narcolepsy.

9. REM paralysis
a) is a myth.
b) only occurs in the elderly.
c) prevents the acting out oI dreams.
d) may become permanent.

Consciousness CHAPTER 4 -48-
10. REM behavior disorder results Irom
a) too much sleep.
b) not enough sleep.
c) Iailure oI the pons to block brain signals to the muscles.
d) deterioration oI the medial hypothalamus.

11. What is the rationale Ior the use oI 'sleepwalking as a deIense Ior committing a crime?
a) It was too dangerous to awaken the sleepwalking criminal.
b) The suspect actually suIIers Irom REM behavior disorder and was unknowingly acting out
a dream.
c) High levels oI anxiety and stress were created by the sleep deprivation caused by the
sleepwalking episodes.
d) The suspect was highly susceptible to suggestion at the time oI the crime.

12. Mary is having insomnia. Which advice would you give to help her deal with it?
a) Take sleeping pills.
b) Have a cup oI hot tea beIore going to bed.
c) Study in bed and then go immediately to sleep.
d) Don`t do anything but sleep in your bed.

13. Sleep apnea is a disorder characterized by
a) diIIiculty Ialling or remaining asleep.
b) episodes oI unexplained cataplexy during the day.
c) diIIiculty breathing while asleep.
d) experiencing temporary paralysis immediately aIter waking up Irom sleep.

14. What two categories oI dream content did Sigmund Freud describe?
a) poetic and realistic
b) literal and symbolic
c) latent and maniIest
d) delusional and hallucinatory

15. The activation-inIormation-mode model (AIM) suggests
a) events that occur during waking hours may inIluence dreams.
b) nothing inIluences dreams.
c) dreams are a neurological side eIIect oI indigestion.
d) dreams have more latent content than once thought.

16. According to the textbook, girls and women tend to dream about
a) animals.
b) cars.
c) people they know.
d) strangers.

17. is a state oI consciousness in which a person may be especially susceptible to
suggestion.
a) Hypnosis
b) Meditation
c) Truth induction
d) Extrasensory perception

Consciousness CHAPTER 4 -49-

18. Tests oI 'hypnotic susceptibility have been Iound to
a) be similar Ior almost everyone.
b) make use oI a series oI suggestions.
c) be almost completely inherited.
d) use deception.

19. Hypnosis can
a) give people superhuman strength.
b) reliably enhance accuracy oI memory.
c) regress people back to childhood.
d) induce amnesia.

20. The idea oI 'hidden observer was suggested by
a) Freud.
b) Watson.
c) Hilgard.
d) Kirsch.

21. Psychoactive drugs
a) speed up activity in the central nervous system.
b) are capable oI inIluencing thinking, perception, and memory.
c) slow down activity in the central nervous system.
d) are derived Irom the opium poppy and relieve pain and produce euphoria.

22. Psychological dependence is best described as
a) a desire to take a drug.
b) drug tolerance and signs oI withdrawal when deprived oI the drug.
c) Ieelings oI sadness when the drug is not available.
d) Ieelings oI euphoria Iollowing the ingestion oI a drug.

23. Drugs that speed up the Iunctioning oI the nervous system are called
a) stimulants.
b) depressants.
c) narcotics.
d) psychogenics.

24. The most addictive and dangerous (as deIined by the number oI deaths caused by the drug)
stimulant in use today is
a) alcohol.
b) amphetamine.
c) nicotine.
d) cocaine.

25. Cathy has just taken a drug that has caused her heart rate and breathing to slow down considerably.
Most likely, Cathy has taken
a) an amphetamine.
b) a barbiturate.
c) LSD.
d) MDMA.

Consciousness CHAPTER 4 -50-

26. Your doctor has decided to give you a prescription Ior a drug to reduce your anxiety levels. Most
likely your doctor will prescribe a
a) narcotic.
b) hallucinogen.
c) depressant.
d) stimulant.

27. Which oI the Iollowing is classiIied as a depressant?
a) cocaine
b) alcohol
c) heroin
d) marijuana

28. Jane has a loss oI equilibrium, decreased sensory and motor capabilities, and double vision.
According to the table in the text, how many drinks has Jane had?
a) 12
b) 35
c) 67
d) 810

29. Morphine, heroin, and methadone
a) are stimulants.
b) are derived Irom opium.
c) are oIten used with ADHD.
d) increase the action oI the central nervous system.

30. LSD is similar to which oI the Iollowing drugs?
a) cocaine
b) methadone
c) PCP
d) CHT

31. Bill is taken to the emergency room oI the hospital aIter he reports hearing dogs screaming and
seeing Iire shooting across his shirt and pants. Assuming his condition is due to a drug overdose,
which type oI drug did Bill most likely consume?
a) a depressant
b) a barbiturate
c) a narcotic
d) a hallucinogen

32. One oI the greatest risks oI using marijuana is
a) physical dependency.
b) psychological dependency.
c) weight gain.
d) heart attack.

Consciousness CHAPTER 4 -51-
PRACTICE EXAM ANSWERS
1. D Consciousness is deIined as personal awareness oI Ieelings, sensation, and thoughts.
Your conscience is your sense oI morality or right and wrong.
2. B II you break down the word, 'circa means about or around (such as circa 1960)
and 'dia means day. So circa-dia means about one day long.
3. D High melatonin level is one oI the signals Ior our body that it is time to sleep. The
release oI melatonin is controlled by signals coming Irom the suprachiasmatic
nucleus, which is light sensitive. In this way, the release oI melatonin Iollows the
light-dark patterns oI the day.
4. b Sleep deprivation oIten leads to decreased perIormance in simple tasks. Microsleeps
are a likely reason Ior a decrease in perIormance. These are brieI episodes oI sleep
that we enter and exit rapidly.
5. b Adaptive theory states that a species sleeps during the time when its predators are
most likely to be out hunting, thus increasing the likelihood oI survival Ior that
species.
6. a The Iaster the brain wave activity, the more alert and awake the person is. Another
option would have been that the person was in REM sleep where Iast small brain
wave activity is also seen.
7. d Waking the sleepwalker is not dangerous. However, it might be hard to do because
they are in Stage 4 deep sleep.
8. b Barbiturate sleeping pills interIere with REM sleep, so since Ted has been deprived
oI REM he is likely to spend a longer than usual amount oI time in REM Ior the
next Iew nights. This phenomenon is known as REM rebound.
9. c During REM sleep the pons sends messages to the spinal cord that inhibits the
movements oI skeletal muscles.
10. c REM behavior disorder occurs when REM paralysis does not work and a person
acts out their dreams. The paralysis is mediated by the pons in the brainstem.
11. b The sleepwalking deIense is actually reIerring to a suspect thought to have REM
behavior disorder.
12. d The idea is that the only association you should have with your bed is sleeping and
this will make it easier Ior you to Iall asleep when you get in bed.
13. c Sleep apnea is a sleeping disorder in which a person actually stops breathing Ior
brieI periods throughout the night.
14. c Freud thought dreams had two levels: the actual content that he called the maniIest
content and then the real meaning which he called the latent content.
15. a The activation-inIormation-mode model (AIM) proposes that content experienced
while an individual is awake may inIluence the synthesis oI dreams.
16. c Women tend to dream about men and women as well as people they know, while
men tend to dream about other men.
17. a Hypnosis is a state oI consciousness in which the person is especially susceptible to
suggestion.
18. b The tests used to determine how likely it is Ior a person to be hypnotized generally
include a list oI suggestions.
19. d Hypnosis has only been Iound to induce temporary amnesia, reduce pain, and alter
sensory perceptions.
20. c Ernst Hilgard suggested that hypnosis was possible because the subject dissociates
himselI into a part that is aware oI what is going on (the hidden observer) and a part
that is unaware.
21. b Speeding up the central nervous system, slowing down the central nervous system,
and the derivatives oI opium that relieve pain or produce euphoria all describe a
speciIic categories oI psychoactive drugs rather than the overall deIinition.
Consciousness CHAPTER 4 -52-

22. c Psychological dependence is the Ieeling that a drug is needed to continue a Ieeling
oI emotional or psychological well-being.
23. a Stimulants speed up heart rate, blood pressure, breathing, among other activities.
24. c Nicotine has been linked to nearly 430,000 deaths per year in the United States
alone.
25. b Barbiturate is the only drug listed that is a depressant.
26. c The depressants known as the mild tranquilizers, or benzodiazepines, are oIten
prescribed to lower anxiety levels.
27. b Alcohol slows down the activity oI the central nervous system; thus, it is classiIied
as a depressant.
28. d See the table in the textbook.
29. b All narcotics are derived Irom the opium poppy. All three oI the drugs listed are
classiIied as narcotics.
30. c LSD and PCP are both hallucinogens.
31. d Hallucinogens produce Ialse sensory perceptions.
32. b The eIIect oI psychological dependence can be very powerIul.

CHAPTER GLOSSARY
activation-information mode
model (AIM)
revised version oI the activation-synthesis explanation oI dreams in which
inIormation that is accessed during waking hours can have an inIluence
on the synthesis oI dreams.
activation-synthesis
hypothesis
premise that states that dreams are created by the higher centers oI the
cortex to explain the activation by the brain stem oI cortical cells during
REM sleep periods.
adaptive theory theory oI sleep proposing that animals and humans evolved sleep patterns
to avoid predators by sleeping when predators are most active.
alcohol the chemical resulting Irom Iermentation or distillation oI various kinds
oI vegetable matter.
alpha waves brain waves that indicate a state oI relaxation or light sleep.
altered states of consciousness state in which there is a shiIt in the quality or pattern oI mental activity as
compared to waking consciousness.
amphetamines stimulants that are synthesized (made) in laboratories rather than being
Iound in nature.
barbiturates depressant drugs that have a sedative eIIect.
benzodiazepines depressant drugs that lower anxiety and reduce stress.
beta waves smaller and Iaster brain waves, typically indicating mental activity.
caffeine a mild stimulant Iound in coIIee, tea, and several other plant-based
substances.
circadian rhythm a cycle oI bodily rhythm that occurs over a 24-hour period.
cocaine a natural drug derived Irom the leaves oI the coca plant.
consciousness a person`s awareness oI everything that is going on around him or her at
any given moment, which is used to organize behavior.
delta waves long, slow waves that indicate the deepest stage oI sleep.
depressants drugs that decrease the Iunctioning oI the nervous system.
hallucinogens drugs that cause Ialse sensory messages, altering the perception oI reality.
hallucinogenics drugs including hallucinogens and marijuana that produces hallucinations
or increased Ieelings or relaxation and intoxication.
heroin narcotic drug derived Irom opium that is extremely addictive.
hypnosis state oI consciousness in which the person is especially susceptible to
suggestion.
Consciousness CHAPTER 4 -53-
insomnia the inability to get to sleep, stay asleep, or get a good quality oI sleep.
LSD (lysergic acid
diethylamide)
powerIul synthetic hallucinogen.
marijuana mild hallucinogen (also known as 'pot or 'weed) derived Irom the
leaves and Ilowers oI a particular type oI hemp plant.
MDMA (Ecstasy or X) designer drug that can have both stimulant and hallucinatory eIIects.
mescaline natural hallucinogen derived Irom the peyote cactus buttons.
microsleeps brieI sidesteps oI sleep lasting only a Iew seconds.
morphine narcotic drug derived Irom opium, used to treat severe pain.
narcolepsy sleep disorder in which a person Ialls immediately into REM sleep during
the day, without warning.
narcotics a class oI opium-related drugs that suppress the sensation oI pain by
binding to and stimulating the nervous system`s natural receptor sites Ior
endorphins.
nicotine the active ingredient in tobacco.
night terrors relatively rare disorder in which the person experiences extreme Iear and
screams or runs around during deep sleep without waking Iully.
nightmares bad dreams occurring during REM sleep.
non-REM (NREM) sleep any oI the stages oI sleep that do not include REM.
opium substance derived Irom the opium poppy Irom which all narcotic drugs
are derived.
PCP synthesized drug now used as an animal tranquilizer that can cause
stimulant, depressant, narcotic, or hallucinogenic eIIects.
physical dependence condition occurring when a person`s body becomes unable to Iunction
normally without a particular drug.
psilocybin natural hallucinogen Iound in certain mushrooms.
psychoactive drug drugs that alter thinking, perception, and memory.
psychological dependence the Ieeling that a drug is needed to continue a Ieeling oI emotional or
psychological well-being.
rapid eye movement (REM)
sleep
stage oI sleep in which the eyes move rapidly under the eyelids and the
person is typically experiencing a dream.
REM behavior disorder a rare disorder in which the mechanism that blocks the movement oI the
voluntary muscles Iails, allowing the person to thrash around and even
get up and act out nightmares.
REM paralysis the inability oI the voluntary muscles to move during REM sleep.
REM rebound increased amounts oI REM sleep aIter being deprived oI REM sleep on
earlier nights.
restorative theory theory oI sleep proposing that sleep is necessary to the physical health oI
the body and serves to replenish chemicals and repair cellular damage.
sleep apnea disorder in which the person stops breathing Ior nearly halI a minute or
more during sleep.
sleep deprivation any signiIicant loss oI sleep, resulting in problems in concentration and
irritability.
sleepwalking (somnambulism) occurring during deep sleep, an episode oI moving around or walking
around in one`s sleep.
social-cognitive theory of
hypnosis
theory that assumes that people who are hypnotized are not in an altered
state but are merely playing the role expected oI them in the situation.
stimulants drugs that increase the Iunctioning oI the nervous system.
stimulatory hallucinogenics drugs that produce a mixture oI psychomotor stimulant and
hallucinogenic eIIects.
theta waves brain waves indicating the early stages oI sleep.
Consciousness CHAPTER 4 -54-
waking consciousness state in which thoughts, Ieelings, and sensations are clear, organized, and
the person Ieels alert.
withdrawal physical symptoms that can include nausea, pain, tremors, crankiness, and
high blood pressure, resulting Irom a lack oI an addictive drug in the
body systems.

Consciousness CHAPTER 4 -55-
Consciousness CHAPTER 4 -56-
CHAPTER 5 - LEARNING


YOU KNOW YOU ARE READY FOR THE TEST IF YOU ARE ABLE TO.
DeIine learning.
Explain what classical conditioning is, how it works, and how it was discovered.
Describe the mechanisms oI operant conditioning, its application in the real world, and the
researchers who contributed to our understanding oI the process.
Discuss cognitive learning theory and the phenomenon oI learned helplessness.
DeIine observational learning and describe Bandura`s classic experiments in the area oI
observational learning.

RAPID REVIEW
Learning is the process that allows us to adapt to the changing conditions oI the environment around
us and is deIined as any relatively permanent change in behavior brought about by experience or practice
(as opposed to changes brought about by maturation). Ivan Pavlov, a Russian physiologist, discovered
one oI the simplest Iorms oI learning called classical conditioning. In his classic paradigm, Pavlov used
dogs as research subjects. Dogs like to eat meat, and when they are exposed to meat, they salivate. The
salivation is a reflex, or an involuntary response that is not under personal choice or control. In Pavlov`s
study, the presentation oI meat was repeatedly paired with the ticking sound oI a metronome. The dogs
heard the ticking sound and then they were presented with the meat. Eventually, the dogs began to
salivate when they heard the ticking sound alone. This process may be described using Iive important
terms. First, the meat is an unconditioned (or ~unlearned) stimulus (UCS), and salivation is a
reIlexive unconditioned response (UCR). Dogs naturally salivate in response to the presence oI meat,
without having to be trained to do so. A ticking sound does not normally cause dogs to salivate; thus, the
ticking sound is originally a neutral stimulus (NS) that does not cause a response on its own. AIter being
paired repeatedly with the meat, the ticking sound begins to produce the same type oI reIlexive response
as the meat. At that point, the ticking sound has become a conditioned stimulus (CS) and the salivation
in response to the ticking sound is a conditioned, or learned, response (CR).
The repeated pairing oI the NS and UCS is known as acquisition. In order Ior classical conditioning
to occur, the CS must occur beIore the UCS, the CS and UCS must occur close together in time, the CS
and UCS must be paired together repeatedly, and the CS should be distinctive. Two other principles oI
classical conditioning are stimulus generalization, the ability oI a stimulus that resembles the CS to
produce a CR, and stimulus discrimination, learning to respond to diIIerent stimuli in diIIerent ways. In
classical conditioning, extinction occurs aIter the CS is repeatedly presented without the UCS, and the
CS no longer produces a CR. Spontaneous recovery occurs when the CS is presented aIter being absent
Ior a period oI time and produces a mild CR. When a powerIul conditioned stimulus is paired with a
neutral stimulus, the conditioned stimulus itselI can Iunction as a UCS and turn the neutral stimulus into a
second conditioned stimulus. This process is called higher-order conditioning.
John Watson demonstrated a particular type oI classical conditioning called conditional emotional
response with Little Albert and his learned phobia oI white rats. Vicarious conditioning occurs when a
person becomes classically conditioned simply by watching someone else respond to a stimulus.
Conditioned taste aversions are a unique Iorm oI classical conditioning that can occur with only one
neutral stimulusunconditioned stimulus pairing. Conditioning is believed to occur so rapidly due to the
biological preparedness oI most mammals. Pavlov suggested that classical conditioning works through
the process oI stimulus substitution, in that the close pairing in time oI the CS with the UCS eventually
leads to the CS serving as a substitute stimulus Ior the UCS and activating the same brain area as the
UCS. Psychologists who agree with the cognitive perspective, such as Robert Rescorla, have suggested
that the CS must provide some inIormation about the upcoming UCS and that it is this expectancy that
causes the association to occur.
Learning CHAPTER 5 -57-
Operant conditioning is a type oI learning more strongly associated with voluntary behavior and is
based on Edward Thorndike`s work with cats and the puzzle box. Based on his research, Thorndike
Iormulated the law of effect, which states that iI a response is Iollowed by a pleasurable consequence it
will tend to be repeated and iI a response is Iollowed by an unpleasant consequence it will tend not to be
repeated. B.F. Skinner expanded on Thorndike`s law oI eIIect and coined the term operant conditioning
Ior this type oI learning, since the term !"#$%&' reIers to any voluntary behavior. While classical
conditioning Iocuses on what happens before the response, the key to operant conditioning is what
happens after the response, or in other words, the consequence. Reinforcement is a consequence that is
pleasurable and strengthens the response that came beIore it. There are two types oI reinIorcers: primary
reinforcers satisIy basic needs and don`t need to be learned. Secondary reinforcers get their reinIorcing
power through prior associations with a primary reinIorcer and thus are learned. ReinIorcement works by
adding a pleasurable consequence aIter a response occurs (positive reinforcement) or removing
something unpleasant aIter a response occurs (negative reinforcement). Both positive and negative
reinIorcement increase the likelihood that the response will occur again.
An important principle that Skinner discovered is that the timing oI reinIorcement can make a
signiIicant diIIerence in how Iast a response is learned. Continuous reinforcement occurs when a
reinIorcer is presented aIter every response. Partial reinforcement occurs when a reinIorcer is given
aIter some, but not all, oI the correct responses. Partial reinIorcement takes longer to go through
extinction, or in other words, is more resistant to extinction. This is known as the partial reinforcement
effect. The timing oI partial reinIorcement is reIerred to as the schedule of reinforcement. There are Iour
diIIerent schedules oI reinIorcement: fixed ratio, variable ratio, fixed interval, and variable interval. A
ratio schedule occurs when a reinIorcer depends on the number oI responses that are made. In an interval
schedule, reinIorcers are presented aIter a certain period oI time has passed. II the reinIorcers are always
given aIter a set period oI time or number oI responses, the schedule is said to be Iixed. II the reinIorcer is
given aIter varying periods oI time or numbers oI responses, the schedule is labeled as variable.
Punishment, on the other hand, decreases the likelihood oI a response. Punishment is any
consequence oI a response that causes that response to be less likely to happen again. While
reinIorcement strengthens a response that already exists, the goal oI punishment is oIten to eliminate the
response, which is usually a much harder task. Typically punishment only temporarily suppresses the
response. Punishment by application describes the situation in which a response is Iollowed by the
addition oI something unpleasant. Punishment by application is not the most eIIective way to modiIy
behavior and has a number oI serious drawbacks. For instance, rather than stopping the behavior that
elicited the punishment, severe punishment may cause the child (or animal) to simply avoid the punisher.
Similarly, severe punishment may encourage lying to avoid punishment, or it may create Iear and anxiety,
which may actually have a negative eIIect on learning. In particular, hitting may provide a successIul
model Ior aggression. Punishment by removal occurs when a response is Iollowed by the removal oI
something pleasant. Many parents and educators Iind this type oI punishment to be less objectionable.
However, one limitation oI this method is that it teachers the child what not to do but it does not teach the
child what he or she should do. Punishment can be made more eIIective iI it is administered immediately
aIter the undesired behavior, is administered consistently, and is paired with reinIorcement Ior the right
behavior.
Shaping involves the use oI operant conditioning to reward successive approximations until the
desired response is obtained. Operant conditioning has several parallels with classical conditioning. For
instance, extinction involves the removal oI reinIorcement, and spontaneous recovery occurs when an
organism attempts a previously learned response in order to receive a reward. In addition, a
discriminative stimulus is deIined as any stimulus that provides an organism with a signal or cue Ior
making a certain response in order to get reinIorcement. In the lab, researchers Iound that even though
animals could be operantly conditioned to perIorm certain tasks, they oIten had a tendency to go back to
their genetic, or natural, way oI doing things. This tendency to revert to genetically controlled patterns is
called instinctive drift.
The term (#)%*+!$ -!.+/+0%'+!& is used to describe the process oI using operant conditioning to
change behavior. A token economy involves the use oI tokens to modiIy behavior. Time-outs are an
Learning CHAPTER 5 -58-
example oI punishment by removal where the child is removed Irom a situation where they could get
attention Irom others. Applied behavior analysis, or ABA, uses shaping techniques to obtain a desired
behavior and is particularly successIul with children with disorders such as autism. The technique called
biofeedback uses operant conditioning to modiIy involuntary behaviors such as blood pressure and heart
rate. When this technique is used to try to change brain activity, as observed using an
electroencephalogram or an IMRI machine, it is reIerred to as neurofeedback.
Cognitive learning theorists Iocus on the mental processes (or cognitions) that occur during learning.
Edward Tolman studied the phenomenon oI latent learning in rats placed in a maze but not reinIorced
Ior Iinding their way out. He Iound that when the rats were subsequently reinIorced, learning occurred
much Iaster than Ior rats that had never been in the maze. Martin Seligman studied a phenomenon he
called learned helplessness in dogs. Two groups oI dogs participated in this experiment. For one group,
the researchers paired a tone with a harmless but painIul electric shock. The dogs were harnessed and not
allowed to escape the shock; that is, they heard the tone and knew the shock was coming, but there was
nothing they could do to prevent it. The researchers expected that, once the dogs had learned to associate
the tone with the shock, they would try to escape whenever they heard the tone. The second group oI dogs
did not undergo conditioning; they were not exposed to the tone and the shock. In the next phase oI the
experiment, dogs oI both groups were placed individually in a two-sided box. The Iloor oI one side oI the
box Ieatured an electriIied grid that could be used to shock the dogs; whereas, the other side oI the box
(the 'saIe side) had no grid. The two sides oI the box were separated by a hurdle. The dogs could jump
over the hurdle to saIety whenever they heard the tone that signaled the impending shock. The researchers
Iound that, upon the sounding oI the tone, the group oI dogs that had not undergone conditioning quickly
jumped over the hurdle in the center oI the box to land on the 'saIe side. However, the conditioned dogs,
those that previously learned that escape was impossible, stayed on the side oI the box in which the shock
occurred, not even trying to leap over the hurdle. The conditioned dogs had essentially learned to be
helpless, and rather than trying to escape, they merely sat down and endured the shock. Seligman
extended the concept oI learned helplessness to humans in an attempt to explain depression. A third
cognitive psychologist, Wolfgang Khler, studied the phenomenon oI insight learning in animals.
Khler believed insight learning involved a sudden perception oI relationships that could not be gained
through trial-and-error learning. All three theories oI learning are related in that they Iocus on what`s
going on inside the learner`s mind during the learning process as opposed to the external stimuli and
rewards oI classical and operant conditioning.
A third category oI learning is that oI observational learning, or the learning oI a new behavior by
observing someone else who is perIorming that behavior. The term 1#%$&+&23"#$/!$-%&0# .+4'+&0'+!&
describes the Iact that learning can take place without actual perIormance. Albert Bandura has been a
major contributor to the study oI observational learning and conducted a series oI classic studies
observing children`s learned behaviors with a blow-up 'Bobo doll. Bandura concluded that Iour
elements were needed Ior observational learning to occur, the Iour elements are attention, memory,
imitation, and motivation.

Learning CHAPTER 5 -59-
STUDY HINTS
9.

Many students get conIused with the terms oI classical conditioning. The Iour major components
to this type oI learning include unconditioned stimulus (UCS), conditioned stimulus (CS),
unconditioned response (UCR) and conditioned response (CR). The best way to keep these terms
straight is to ask yourselI two questions.

1. Is the event I am interested in a stimulus or a response?

2. Is the stimulus/response something that was learned or something that occurs
naturally, by instinct?

The Iirst question is the easiest way to break down the inIormation. II an event is a stimulus, it
will cause something else to happen. List some examples oI stimuli here.



You might have mentioned anv number of stimuli including events such as a bright light, a puff
of air, a loud siren, a soft whisper, a touch on vour arm, the smell of cookies, a written word.
The list is quite large. A stimulus is anv event that causes a response.

Now that you have a good Ieeling Ior what stimuli are, try listing some examples oI some
possible responses.



You might have mentioned events such as blinking vour eves, laughing, crving, fumping up,
heart rate increasing, feeling scared, raising vour hand, driving faster. A response is anv
behavior (inside or outside of vour bodv) that can be observed.

Once you determine whether your event is a stimulus or response, the second question is Iairly
easy. Is the stimulus something the subject had to learn how to respond to? II so, then it would
be a learned or conditioned stimulus. II the stimulus is something that causes the response
automatically, then it is an unlearned or unconditioned stimulus. The same rule applies Ior the
responses. II this is a response that does not occur by instinct, but instead has been learned
through experience, then this is a learned or conditioned response. II the response happens the
Iirst time you encounter the stimulus, as an instinct, then it is an unlearned or unconditioned
response. Now try some examples and see how you do.


A puII oI air is aimed at your eye and you blink.
The event we are interested in is: the blink

Question 1: Is this a stimulus or a response?



If vou wrote response, then vou are correct.
Blinking is a behavior that we can observe.

Question 2: Is this response learned or unlearned?


Learning CHAPTER 5 -60-

If vou wrote unlearned, then vou are correct.
Blinking to a puff of air is an instinct.

Now you can Iill in the blanks.
The Iirst answer tells you this is a response, so it is either a CR or a UCR.
The second answer tells you this is unlearned or unconditioned, so it must be a UCR.

Now circle the right term:
Stimulus Response
Learned CS CR
Unlearned UCS UCR

Try some more on your own.


A picture oI a piece oI chocolate cake causes your mouth to water.
The event we are interested in is: the picture oI the cake

Question 1: Is this a stimulus or a response?



Question 2: Is this response learned or unlearned?



Now circle the right term:
Stimulus Response
Learned CS CR
Unlearned UCS UCR


Your heart speeds up as you see a police car pull up behind you.
The event we are interested in is: your heart speeding up

Question 1: Is this a stimulus or a response?



Question 2: Is this response learned or unlearned?



Now select the right term:
Stimulus Response
Learned CS CR
Unlearned UCS UCR

Learning CHAPTER 5 -61-

A loud noise causes someone to jump.
The event we are interested in is: the loud noise

Question 1: Is this a stimulus or a response?



Question 2: Is this response learned or unlearned?



Now select the right term:
Stimulus Response
Learned CS CR
Unlearned UCS UCR


You should have selected the following.
blinking vour eves is a UCR
the piece of cake is a CS
vour heart speeding up is a CR
the loud noise is a UCS

Learning CHAPTER 5 -62-


10.


Negative reinIorcement and punishment are oIten conIused. In negative reinIorcement,
something bad is taken away. In punishment by removal, something good or desirable is taken
away. Most people would enjoy being negatively reinIorced but would be upset about being
punished. Work through the Iollowing scenarios to determine whether the person is being
negatively reinIorced or punished. The Iirst one has been completed Ior you.

Behavior Consequence
Is something
good or bad
taken away?
Is this negative
reinIorcement or
punishment?
Will the
behavior
increase or
decrease?
Taking an
aspirin Ior a
headache.
Headache goes
away.
bad
negative
reinforcement
increase
Running a red
light.
Driver`s license
is taken away.

Cleaning your
room so that you
are no longer
grounded.
You are no
longer
grounded.

Drinking coIIee
in the morning
when you are
very tired.
You no longer
Ieel tired.

Staying out past
your curIew.
Your parents
ground you.

Getting in a
Iight with a
Iriend.
Your Iriend will
not talk to you
anymore.

Fastening your
seatbelt when
the buzzer is
making a noise.
The buzzer
stops.

Driving your car
until it runs out
oI gas.
You can`t drive
your car
anymore.

Your boyIriend
nags you until
you take him out
to dinner.
The nagging
stops.


Learning CHAPTER 5 -63-

Suggested answers

Behavior Consequence
Is something
good or bad
taken away?
Is this negative
reinIorcement or
punishment?
Will the
behavior
increase or
decrease?
Taking an
aspirin Ior a
headache.
Headache goes
away
bad
negative
reinforcement
increase
Running a red
light.
Driver`s license
is taken away.
good punishment decrease
Cleaning your
room so that you
are no longer
grounded.
You are no
longer
grounded.
bad
negative
reinforcement
increase
Drinking coIIee
in the morning
when you are
very tired.
You no longer
Ieel tired.
bad
negative
reinforcement
increase
Staying out past
your curIew.
Your parents
ground you.
good punishment decrease
Getting in a
Iight with a
Iriend.
Your Iriend will
not talk to you
anymore.
good punishment decrease
Fastening your
seatbelt when
the buzzer is
making a noise.
The buzzer
stops.
bad
negative
reinforcement
increase
Driving your car
until it runs out
oI gas.
You can`t drive
your car
anymore.
good punishment decrease
Your boyIriend
nags you until
you take him out
to dinner.
The nagging
stops.
bad
negative
reinforcement
increase



Learning CHAPTER 5 -64-
LEARNING OBJECTIVES
5.1 What does the term learning reallv mean?
5.2 How was classical conditioning first
studied, and what are the important
elements and characteristics of classical
conditioning?
5.3 What is a conditioned emotional response,
and how do cognitive psvchologists explain
classical conditioning?
5.4 How does operant conditioning occur, and
what were the contributions of Thorndike
and Skinner?
5.5 What are the important concepts in operant
conditioning?
5.6 What are the schedules of reinforcement?
5.7 How does punishment differ from
reinforcement?
5.8 What are some of the problems with using
punishment?

5.9 How do operant stimuli control behavior,
and what kind of behavior is resistant to
operant conditioning?
5.10 What is behavior modification, and how can
behavioral techniques be used to modifv
involuntarv biological responses?
5.11 How do latent learning, learned
helplessness, and insight relate to cognitive
learning theorv?
5.12 What occurs in observational learning,
including findings from Banduras classic
Bobo doll studv and the four elements of
observational learning?
5.13 What is a real-world example of the use of
conditioning?


PRACTICE EXAM
For the Iollowing multiple choice questions, select the answer you Ieel best answers the questions.

1. is any relatively permanent change in behavior brought about by experience or practice.
a) Learning
b) Adaptation
c) Memory enhancement
d) Muscle memory

2. The researcher responsible Ior discovering classical conditioning was
a) Skinner.
b) Tolman.
c) Kohler.
d) Pavlov.

3. Which oI the Iollowing correctly describes the process oI classical conditioning?
a) pairing a stimulus that naturally causes a certain response with a second stimulus that
naturally causes the same response
b) pairing a stimulus that naturally causes a certain response with a second stimulus that does
not naturally cause that response
c) presenting a pleasurable stimulus aIter the occurrence oI a speciIic response
d) presenting an unpleasant stimulus aIter the occurrence oI a speciIic response

4. When Pavlov placed meat powder or other Iood in the mouths oI canine subjects, they began to
salivate. The salivation was a(n)
a) unconditioned response.
b) unconditioned stimulus.
c) conditioned response.
d) conditioned stimulus.

Learning CHAPTER 5 -65-

5. Judy would sometimes discipline her puppy by swatting its nose with a rolled-up newspaper. One
day she brought the newspaper into the house still rolled up, and her puppy ran Irom her in Iear. By
pairing the rolled paper with the swat, Judy`s puppy had developed a(n) response to
the rolled-up paper.
a) generalized
b) conditioned
c) unconditioned
d) discriminative

6. You decide you want to try to classically condition your pet dog. What is the correct order that you
should use to present the stimuli to your dog?
a) unconditioned stimulus neutral stimulus
b) neutral stimulus neutral stimulus
c) neutral stimulus unconditioned stimulus
d) present the unconditioned stimulus only

7. AIter you successIully classically conditioned your pet dog, you repeatedly presented the
conditioned stimulus without ever pairing it with the unconditioned stimulus. Over time, your dog
stops perIorming the conditioned response. What has happened?
a) extinction
b) spontaneous recovery
c) generalization
d) stimulus discrimination

8. John Watson oIIered a live white rat to Little Albert and then made a loud noise behind his head by
striking a steel bar with a hammer. The white rat served as the in this study.
a) discriminative stimulus
b) counterconditioning stimulus
c) conditioned stimulus
d) unconditioned stimulus

9. Pavlov discovered classical conditioning through his study oI
a) cats escaping Irom a puzzle box.
b) primate research into problem solving.
c) digestive secretions in dogs.
d) lever-pressing responses oI rats.

10. Television advertisers have taken advantage oI the Iact that most people experience positive
emotions when they see an attractive, smiling person. This association is an example oI
a) operant conditioning.
b) a conditioned emotional response.
c) negative reinIorcement.
d) punishment.

11. The current view oI why classical conditioning works the way it does, by cognitive theorists such as
Rescorla, adds the concept oI to the conditioning process.
a) generalization
b) habituation
c) memory loss
d) expectancy

Learning CHAPTER 5 -66-
12. 'II a response is Iollowed by a pleasurable consequence, it will tend to be repeated. II a response is
Iollowed by an unpleasant consequence, it will tend not to be repeated. This is a statement oI
a) the law oI positive reinIorcement.
b) Rescorla`s cognitive perspective.
c) Thorndike`s law oI eIIect.
d) Garcia`s conditional emotional response.

13. Kenra has a new pet cat and decides to modiIy her cat`s behavior by administering pleasant and
unpleasant consequences aIter her cat`s behaviors. Kenra is using the principles oI
a) observational learning.
b) operant conditioning.
c) classical conditioning.
d) insight learning.

14. A box used in operant conditioning oI animals, which limits the available responses and thus
increases the likelihood that the desired response will occur, is called a
a) trial box.
b) response box.
c) Watson box.
d) Skinner box.

15. A negative reinIorcer is a stimulus that is and thus the probability oI a
response.
a) removed; increases
b) removed; decreases
c) presented; increases
d) presented; decreases

16. The partial reinIorcement eIIect reIers to a response that is reinIorced aIter some, but not all, correct
responses will be
a) more resistant to extinction than a response receiving continuous reinIorcement (a
reinIorcer Ior each and every correct response).
b) less resistant to extinction than a response receiving continuous reinIorcement (a
reinIorcer Ior each and every correct response).
c) more variable in its resistance to extinction than a response receiving continuous
reinIorcement (a reinIorcer Ior each and every correct response).
d) totally resistant to extinction unlike a response receiving continuous reinIorcement (a
reinIorcer Ior each and every correct response).

17. Which example best describes the Iixed interval schedule oI reinIorcement?
a) receiving a paycheck aIter two weeks oI work
b) receiving a bonus aIter selling 20 cell phones
c) giving your dog a treat when he seems hungry
d) giving your dog a treat at least once a day when he comes when you call him

18. Which schedule oI reinIorcement should you select iI you would like to produce the highest number
oI responses with the least number oI pauses between the responses?
a) Iixed ratio
b) variable ratio
c) Iixed interval
d) variable interval

Learning CHAPTER 5 -67-
19. When a stimulus is removed Irom a person or animal and decreases the probability oI response, it is
known as
a) positive punishment.
b) punishment by removal.
c) negative reinIorcement.
d) negative punishment.

20. Your child has begun drawing on the walls oI your house and you would like this activity to stop.
Which oI the Iollowing actions would, at least temporarily, decrease the occurrence oI the behavior
in your child?
a) use insight learning to get your child to stop drawing on the wall
b) use classical conditioning to create a positive association with drawing on the wall
c) negatively reinIorce your child aIter she draws on the wall
d) punish your child aIter she draws on the wall

21. An example oI a discriminative stimulus might be
a) a stop sign.
b) the stimulus that acts as a UCS in classical conditioning.
c) the white rat in Watson`s Little Albert study oI producing phobias.
d) none oI these.

22. In their 1961 paper on instinctive driIt, the Brelands determined that three assumptions most
Skinnerian behaviorists believed in were not actually true. Which is one oI the assumptions that
were NOT true?
a) The animal comes to the laboratory a tabula rasa, or 'blank slate, and can thereIore be
taught anything with the right conditioning.
b) DiIIerences between species oI animals are insigniIicant.
c) All responses are equally able to be conditioned to any stimulus.
d) All oI these were not true.

23. Applied behavior analysis or ABA has been used with autistic children. The basic principle oI this
Iorm oI behavior modiIication is
a) partial reinIorcement.
b) classical conditioning.
c) negative punishment.
d) shaping.

24. is a type oI operant conditioning that is used by humans to bring involuntary
responses, such as heart rate and blood pressure, under their voluntary control.
a) BioIeedback
b) Social learning
c) Preparedness
d) Instinct driIt

25. Cognition reIers to
a) behavior that is observable and external.
b) behavior that is directly measurable.
c) the mental events that take place while a person is behaving.
d) memories.

Learning CHAPTER 5 -68-

26. The idea that learning occurs, and is stored up, even when behaviors are not reinIorced is called
a) insight.
b) latent learning.
c) placebo learning.
d) innate learning.

27. A researcher places dogs in a cage with metal bars on the Iloor. The dogs are randomly given
electric shocks and can do nothing to prevent them or stop them. Later, the same dogs are placed in
a cage where they can escape the shocks by jumping over a low hurdle. When the shocks are given,
the dogs do not even try to escape. They just sit and cower. This is an example oI
a) learned helplessness.
b) stimulus discrimination.
c) aversive conditioning.
d) vicarious learning.

28. The "aha!" experience is known as
a) latent learning.
b) insight learning.
c) thoughtIul learning.
d) serial enumeration.

29. II you learn how to Iix your car by watching someone on TV demonstrate the technique, you are
acquiring that knowledge through
a) latent learning.
b) operant conditioning.
c) classical conditioning.
d) observational learning.

30. In Bandura`s study with the Bobo doll, the children in the group that saw the model punished did
not imitate the model at Iirst. They would only imitate the model iI given a reward Ior doing so. The
Iact that these children had obviously learned the behavior without actually perIorming it is an
example oI
a) latent learning.
b) operant conditioning.
c) classical conditioning.
d) insight learning.

31. In Bandura`s study oI observational learning, the abbreviation AMIM stands Ior
a) attention, memory, imitation, motivation.
b) alertness, motivation, intent, monetary reward.
c) achievement, momentum, initiative, memory.
d) achievement, motivation, intellectual capacity, memory.

32. Which oI the Iollowing real-world situations is using the principles oI classical conditioning?
a) giving a child a star Ior completing her homework assignment
b) sending a child to time-out Ior stealing his Iriend`s toy truck
c) grounding a child until she gets her room cleaned
d) a hungry child smiling at the sight oI the spoon her dad always uses to Ieed her lunch

Learning CHAPTER 5 -69-
PRACTICE EXAM ANSWERS
1. a This is the deIinition oI learning given in the textbook and restated in the summary.
2. d Skinner developed the theory oI operant conditioning, and both Kohler and Tolman
Iocused on cognitive learning.
3. b Classical conditioning occurs when you pair a neutral stimulus (NS) with an
unconditioned stimulus (UCS). AIter repeated pairings, the NS now causes a
response similar to the naturally occurring response. The stimulus is now called a
conditioned stimulus and the response is the conditioned response.
4. a An unconditioned response is a response that occurs naturally and does not have to
be learned. When Iood is placed in a dog`s mouth, the dog will naturally begin to
salivate.
5. b A conditioned response is a response that has been learned through association.
Originally, the rolled-up newspaper did not cause a response oI Iear in the puppy,
but aIter repeated pairings with a swat, it now causes the Iear response.
6. c For classical conditioning to occur, the neutral stimulus must be repeatedly paired
with an unconditioned stimulus. In addition, the neutral stimulus must be presented
before the unconditioned stimulus.
7. a Extinction occurs when the CS is continuously presented without the UCS.
8. c First, decide whether the rat is a stimulus or a response. Obviously, the rat is a
stimulus. Then Iigure out iI the rat naturally, or instinctively, will cause the
response oI Iear or iI the response needs to be learned. II it needs to be learned, then
the stimulus is a conditioned stimulus.
9. c Pavlov was a Russian physiologist who won a Nobel prize Ior his study oI the
digestive system in dogs. It was during this research that he observed the
phenomenon oI classical conditioning and devoted the rest oI his years in research
to the study oI classical conditioning.
10. b The association between attractive people and Ieelings oI happiness is learned
through classical conditioning and is speciIically reIerred to as a conditioned
emotional response because it deals with a response oI emotion. Notice that all the
other choices were related to operant conditioning.
11. d Expectancy is the idea that the conditioned stimulus has to provide some
inIormation about the upcoming unconditioned stimulus, so that we are expecting
the UCS to occur.
12. c Thorndike developed this principle through his study oI animals escaping Irom
puzzle boxes.
13. b This is a modiIied Iorm oI the deIinition oI operant conditioning.
14. d The Skinner box was designed by B. F. Skinner and typically included an apparatus
Ior the animal to move (such as a lever to press) and a mechanism Ior delivering a
reward to the animal.
15. a Always start with the Iact that reinIorcement always increases the response.
Negative reinIorcement occurs when an unpleasant stimulus is removed, making a
the correct choice.
16. a A response that is resistant to extinction that means that the person will continue
making the response even when it is not Iollowed by a reinIorcer.
17. a Fixed means that the reinIorcement will always be presented aIter the same period
oI time or number oI responses. Interval means that you are dealing with the
passage oI time.
18. b The ratio schedule produces the most rapid responses because the reward depends
on making a certain number oI responses. The variable schedule reduces the pauses
aIter receiving the reinIorcer because the next reward could be given at any time.
Learning CHAPTER 5 -70-

19. b Remember that punishment decreases behavior and reinIorcement increases
behavior. The question is asking about a behavior decrease, so it must be talking
about punishment. Removing a stimulus is described as punishment by removal.
20. d Once again, you would like the behavior to decrease so you should select
punishment.
21. a A discriminative stimulus is deIined as a stimulus that provides a cue that a
response might lead to reinIorcement. It is a term used with operant conditioning.
22. d The Brelands questioned all three oI these assumptions.
23. d ABA rewards closer and closer approximations to the desired behavior, which is the
deIinition oI shaping.
24. a BioIeedback uses Ieedback about biological conditions to bring involuntary
responses under voluntary control. It is a type oI operant conditioning. The change
in physiological state is the response and the light or tone serves as the
reinIorcement.
25. c Cognitive psychologists Iocus on our thought process and mental activities.
26. b The word latent means something that`s present but not visible.
27. a Learned helplessness was studied by Seligman as a potential animal model oI
depression.
28. b With this type oI learning, you have a sudden realization or 'insight.
29. d Observational learning occurs when you learn a new behavior or new knowledge
through the observation oI a model.
30. a Latent learning occurs when a new behavior has been acquired but the behavior is
not perIormed, as the children in Bandura`s experiment did not imitate the model
until they were encouraged and rewarded to do so.
31. a All the selections match the abbreviation, so try to think about what skills would be
needed to learn by observation. First oI all, you need to watch the person you are
trying to learn Irom, and you realize that observational learning can occur without
any rewards being oIIered.
32. d Except Ior the hungry child, the examples are oI operant conditioning, rather than
classical conditioning.

CHAPTER GLOSSARY
applied behavior analysis
(ABA)
modern term Ior a Iorm oI Iunctional analysis and behavior modiIication
that uses a variety oI behavioral techniques to mold a desired behavior or
response.
behavior modification the use oI operant conditioning techniques to bring about desired
changes in behavior.
biofeedback using bioIeedback about biological conditions to bring involuntary
responses, such as blood pressure and relaxation, under voluntary
control.
biological preparedness reIerring to the tendency oI animals to learn certain associations, such as
taste and nausea, with only one or Iew pairings due to the survival value
oI the learning.
classical conditioning learning to make an involuntary (reIlex) response to a stimulus other
than the original, natural stimulus that normally produces the reIlex.
cognitive perspective modern theory in which classical conditioning is seen to occur because
the conditioned stimulus provides inIormation or an expectancy about
the coming oI the unconditioned stimulus.
conditional emotional
response (CER)
emotional response that has become classically conditioned to occur to
learned stimuli, such as a Iear oI dogs or the emotional reaction that
occurs when seeing an attractive person.
Learning CHAPTER 5 -71-
conditioned response (CR) learned reIlex response to a conditioned stimulus.
conditioned stimulus (CS) stimulus that becomes able to produce a learned reIlex response by being
paired with the original unconditioned stimulus.
conditioned taste aversions development oI a nausea or aversive response to a particular taste
because that taste was Iollowed by a nausea reaction, occurring aIter
only one association.
continuous reinforcement the reinIorcement oI each and every correct response.
discriminative stimulus any stimulus, such as a stop sign or a doorknob, that provides the
organism with a cue Ior making a certain response in order to obtain
reinIorcement.
extinction the disappearance or weakening oI a learned response Iollowing the
removal or absence oI the unconditioned stimulus (in classical
conditioning) or the removal oI a reinIorcer (in operant conditioning).
fixed interval schedule of
reinforcement
schedule oI reinIorcement in which the interval oI time that must pass
beIore reinIorcement becomes possible is always the same.
fixed ratio schedule of
reinforcement
schedule oI reinIorcement in which the number oI responses required Ior
reinIorcement is always the same.
higher-order conditioning occurs when a strong conditioned stimulus is paired with a neutral
stimulus, causing the neutral stimulus to become a second conditioned
stimulus.
insight the sudden perception oI relationships among various parts oI a problem,
allowing the solution to the problem to come quickly.
instinctive drift tendency Ior an animal`s behavior to revert to genetically controlled
patterns.
latent learning learning that remains hidden until its application becomes useIul.
law of effect law stating that iI an action is Iollowed by a pleasurable consequence, it
will tend to be repeated, and iI Iollowed by an unpleasant consequence,
it will tend to not be repeated.
learned helplessness the tendency to Iail to act to escape Irom a situation because oI a history
oI repeated Iailures in the past.
learning/performance
distinction
reIerring to the observation that learning can take place without actual
perIormance oI the learned behavior.

negative reinforcement the reinIorcement oI a response by the removal, escape Irom, or
avoidance oI an unpleasant stimulus.
neurofeedback Iorm oI bioIeedback using brain-scanning devices to provide Ieedback
about brain activity in an eIIort to modiIy behavior.
neutral stimulus (NS) stimulus that has no eIIect on the desired response.
observational learning learning new behavior by watching a model perIorm that behavior.
operant any behavior that is voluntary.
operant conditioning the learning oI voluntary behavior through the eIIects oI pleasant and
unpleasant consequences to responses.
partial reinforcement effect the tendency Ior a response that is reinIorced aIter some, but not all,
correct responses to be very resistant to extinction.
positive reinforcement the reinIorcement oI a response by the addition or experiencing oI a
pleasure stimulus.
primary reinforcer any reinIorcer that is naturally reinIorcing by meeting a basic biological
need, such as hunger, thirst, or touch.
punishment any event or object that, when Iollowing a response, makes that response
less likely to happen again.
Learning CHAPTER 5 -72-

punishment by application the punishment oI a response by the addition or experiencing oI an
unpleasant stimulus.
punishment by removal the punishment oI a response by the removal oI a pleasurable stimulus.
reflex an involuntary response, one that is not under personal control or choice.
reinforcement any event or stimulus that, when Iollowing a response, increases the
probability that the response will occur again.
reinforcers any events or objects that, when Iollowing a response, increase the
likelihood oI that response occurring again.
secondary reinforcer any reinIorcer that becomes reinIorcing aIter being paired with a primary
reinIorcer, such as praise, tokens, or gold stars.
shaping the reinIorcement oI simple steps in behavior that lead to a desired, more
complex behavior.
spontaneous recovery the reappearance oI a learned response aIter extinction has occurred.
stimulus discrimination the tendency to stop making a generalized response to a stimulus that is
similar to the original conditioned stimulus because the similar stimulus
is never paired with the unconditioned stimulus.
stimulus generalization the tendency to respond to a stimulus that is only similar to the original
conditioned stimulus with the conditioned response.
stimulus substitution original theory in which Pavlov stated that classical conditioning
occurred because the conditioned stimulus became a substitute Ior the
unconditioned stimulus by being paired closely together.
successive approximations small steps in behavior, one aIter the other, that lead to a particular goal
behavior.
token economy type oI behavior modiIication in which desired behavior is rewarded
with tokens.
unconditioned response
(UCR)
an involuntary (reIlex) response to a naturally occurring or
unconditioned stimulus.
unconditioned stimulus
(UCS)
a naturally occurring stimulus that leads to an involuntary (reIlex)
response.
variable interval schedule of
reinforcement
schedule oI reinIorcement in which the interval oI time that must pass
beIore reinIorcement becomes possible is diIIerent Ior each trial or
event.
variable ratio schedule of
reinforcement
schedule oI reinIorcement in which the number oI responses required Ior
reinIorcement is diIIerent Ior each trial or event.
vicarious conditioning classical conditioning oI a reIlex response or emotion by watching the
reaction oI another person.

Learning CHAPTER 5 -73-
Learning CHAPTER 5 -74-
CHAPTER 6 - MEMORY


YOU KNOW YOU ARE READ FOR THE TEST IF YOU ARE ABLE TO.
Introduce the study oI memory including the basic processes oI encoding, storage, and retrieval as
well as current theories oI how memory works.
Discuss the inIormation-processing theory oI memory in detail including the concepts oI sensory,
short-term, and long-term memory.
IdentiIy the basic mechanisms and limitations in the retrieval oI inIormation including Ialse
memories.
Describe Ebbinghaus`s work on Iorgetting and proposed explanations Ior Iorgetting.
Explain the biological processes thought to underlie memory and the deterioration oI memory.
Discuss Alzheimer`s disease and its eIIects on memory.

RAPID REVIEW
Memory can be thought oI as an active system that receives inIormation Irom the senses, organizes
and alters it as it stores it, and then retrieves inIormation Irom storage. All the current models oI memory
involve the three processes oI encoding, storage, and retrieval.
Three models or theories about memory are discussed in the text. One is the levels-of-processing
model, which proposes that how long a memory will be remembered depends on the depth to which it
was processed. A second model is the parallel distributed processing model, which proposes that
memories are created and stored across a network oI neural circuits simultaneously, or in other words, in
a parallel Iashion. The third and currently most accepted model oI memory is the information-
processing model, which proposes that memory is divided into three components: sensory, short term,
and long term. Sensory memory is the Iirst stage oI memory and involves inIormation Irom our sensory
systems. Visual sensory memory is called iconic memory and was studied extensively by George
Sperling through the use oI the partial report method. The capacity oI iconic memory is everything that
can be seen at one time and the duration is around halI a second. Eidetic imagery, also known as
photographic memory, is the ability to access visual sensory memory over a long period oI time. Iconic
memory is useIul Ior allowing the visual system to view the surroundings as continuous and stable.
Echoic memory is the memory oI auditory inIormation and has the capacity oI what can be heard at any
one moment and has a duration oI about two seconds.
The inIormation-processing model proposes that inIormation moves Irom sensory memory to short-
term memory through the process oI selective attention. This process explains the phenomenon oI the
cocktail party effect, when you are at a party and hear your name in a conversation across the room.
Another name Ior short-term memory is working memory, and some researchers propose that short-term
memory consists oI a central control process along with a visual 'sketch pad and auditory 'recorder.
George Miller studied the capacity oI short-term memory using the digit-span memory test and
discovered that people can store an average oI seven chunks oI inIormation (plus or minus two) in their
short-term memory. More recently, researchers have Iound that younger adults can hold about 3 to 5
items oI inIormation at a time iI a strategy oI some type is not being used. Chunking is the process oI
reorganizing the inIormation into meaningIul units. The duration oI short-term memory is 1030 seconds
without rehearsal. Maintenance rehearsal describes the process oI continuing to pay attention to a piece
oI inIormation, such as reciting a name over and over again in your head.
Long-term memory is the third stage oI memory proposed by the inIormation-processing theory and
has an essentially unlimited capacity and duration. InIormation may by encoded into long-term memory
through elaborative rehearsal, a way oI transIerring inIormation by making it meaningIul. Long-term
memories can be divided into two types, procedural and declarative. Procedural, or nondeclarative,
memories are memories Ior skill and habits, in other words, memories Ior things people can do.
Declarative memories are memories oI Iacts, or things people can know. There are two types oI
declarative memories, semantic and episodic. Semantic memory is memory Ior the meanings oI words
Memory CHAPTER 6 -75-
and concepts while episodic memory is the memory oI events or 'episodes. Procedural memories
appear to be stored in the cerebellum and amygdala, while declarative memories most likely involve the
Irontal and temporal lobes. Procedural memory is sometimes reIerred to as implicit memory, and
declarative memory can be thought oI as explicit memory. Explicit memories are easily verbalized, while
implicit memories are nearly impossible to state in words. It is not entirely clear how the brain organizes
inIormation in long-term memory. The semantic network model suggests that inIormation is stored in
the brain in a connected Iashion with related concepts physically close to each other.
Retrieval describes the process oI pulling memories out oI long-term memory. A retrieval cue is a
stimulus that aids in the process oI remembering. When the environment in which you learned an item
serves as a retrieval cue, it is reIerred to as encoding specificity. II an emotional state serves as a retrieval
cue, it is called state-dependent learning. InIormation can be retrieved through the process oI recall,
such as Iilling in the blanks, or recognition, such as multiple choice questions in which the correct
answer only needs to be 'recognized. Not all inIormation can be recalled equally well. The serial
position effect describes the Iinding that inIormation at the beginning and end oI a list is more likely to
be remembered than the inIormation in the middle. The primacy effect proposes that the inIormation at
the beginning oI the list is remembered due to rehearsal, while the recency effect proposes that the
inIormation at the end oI the list is remembered due to the Iact that it is still in short-term memory.
Recognition is usually a much easier task than recall because the retrieval cue is the actual piece oI
inIormation you are trying to remember, yet retrieval errors are still made when using recognition. A false
positive occurs when someone recognizes a piece oI inIormation as a memory even though it did not
happen. For example, a witness says they saw broken glass at the scene oI an accident, when there was no
glass broken in the accident. Elizabeth Loftus spent more than 30 years investigating the reliability oI
eyewitness memories and Iound that what people see and hear about an event aIter the Iact can aIIect the
accuracy oI their memories Ior that event. Automatic encoding is a term used to describe the memory
process when we aren`t actively paying attention to the inIormation. A flashbulb memory is a speciIic
type oI automatic encoding that occurs when an unexpected and oIten emotional event occurs. Flashbulb
memories typically contain a great deal oI inIormation including many details but might not be as
accurate as they appear.
The retrieval oI memories is a much more constructive process than most people assume. Several
Iactors aIIect the accuracy oI inIormation retrieval. One Iactor is the misinformation effect in which
Ialse inIormation presented aIter an event inIluences the memory oI that event. When suggestions Irom
others create inaccurate or Ialse memories, this is reIerred to as the false memory syndrome. The Ialse
memory syndrome has Irequently been observed while people are under hypnosis. Research by LoItus has
suggested that in order Ior an individual to interpret a Ialse event as a true memory, the event must seem
plausible and the individual should be given inIormation that supports the belieI that the event could have
happened to them personally. Hindsight bias is the tendency oI people to Ialsely believe that they would
have been able to accurately predict a result.
Herman Ebbinghaus was one oI the Iirst scientists to systematically study the process oI Iorgetting.
Using lists oI nonsense syllables, he discovered that most Iorgetting occurs in the initial hour aIter the
material is learned. He presented his Iindings in a visual graph called the curve of forgetting. There are at
least Iour diIIerent causes Ior Iorgetting. Encoding failure occurs when the inIormation does not make it
past the initial encoding process and never really becomes a memory. Another possible cause oI
Iorgetting is the decay (or disuse) oI the memory trace in short-term memory or the disuse oI the
inIormation in long-term memory. The Iinal two causes oI Iorgetting discussed in the textbook have to do
with interIerence. Proactive interference occurs when inIormation Irom the past disrupts newly learned
inIormation. Retroactive interference occurs when the newly learned inIormation interIeres with the
memories oI the inIormation Irom the past. Ebbinghaus Iound he could greatly improve memory iI he
spaced out his study sessions, a technique called distributed practice, as opposed to 'cramming or
trying to learn all the inIormation the night beIore the exam.
It is still unclear exactly how memories are physically stored in the brain. In general, strong evidence
suggests that long-term procedural memories are stored in the cerebellum, while long-term declarative
memories are stored in the Irontal and temporal lobes. Storage oI short-term memories has been
Memory CHAPTER 6 -76-
associated with the preIrontal cortex and the temporal lobe. The process oI physically storing a memory
in your brain is called consolidation and could consist oI a number oI changes including an increase in
receptor sites, increased sensitivity at the synapse through repeated stimulation (called long-term
potentiation), changes on the dendrites, or changes in proteins in the neuron. Recently, researchers have
identiIied a speciIic protein (4E-BP2), which may control the production oI new proteins within the
mammalian nervous system. It is thought that this protein may play a role in memory consolidation.
The hippocampus has been Iound to play an important role in the Iormation oI new memories. This
Iact was mainly discovered by observing patients with damage to the hippocampus and noting their
inability to Iorm any new memories. A man named H. M. was the most Iamous oI these patients. H. M.`s
hippocampi were removed during a surgical procedure to reduce the severity oI his epileptic seizures.
AIter the surgery, H. M. could not Iorm any new declarative memories. H. M. could, however, still Iorm
new procedural memories. Amnesia is a disorder characterized by severe memory loss, such as that oI H.
M.`s and can take one oI two Iorms. Retrograde amnesia is an inability to retrieve memories Irom the
past, while anterograde amnesia is an inability to Iorm any new memories. An inability to remember
events Irom the Iirst Iew years oI liIe has been described as infantile amnesia and may be due to the
implicit, or nonverbal, nature oI those memories.
Alzheimer`s disease is one type oI dementia that is associated with severe memory loss. Anterograde
amnesia tends to be the primary memory problem in the beginning oI the disease process. Memory loss
may be rather mild at Iirst but becomes more pronounced over time. The cause Ior Alzheimer`s disease is
not completely understood, and there is currently no cure.

STUDY HINTS
11.

Two oI the most important concepts presented in this chapter consist oI a three-part model. One
concept is the basic processes involved in memory: encoding, storage, and retrieval. The other
concept is the inIormation-processing model oI memory, which consists oI sensory, short-term,
and long-term memory. Students oIten get these ideas conIused. To help you clariIy the
concepts, correctly identiIy the components oI the inIormation-processing model in the
Iollowing diagram. Remember that encoding, storage, and retrieval can happen at each oI these
stages. List an example oI encoding, storage, and retrieval Ior each stage.


Encoding:

Storage:

Retrieval:
Encoding:

Storage:

Retrieval:
Encoding:

Storage:

Retrieval:
Memory CHAPTER 6 -77-

12.

Long-term memory can be divided into two basic types oI memory: procedural and declarative.
Declarative memories can be Iurther broken down into episodic and semantic. To help you
understand the diIIerence between these types oI memories, come up with a speciIic memory
Irom your own liIe and write it in the appropriate box.


Suggested answers for Studv Hint 1

Suggested answers for Studv Hint 2


Long-Term Memories
Procedural Memories
Declarative
Procedural
Semantic Memories Episodic Memories
Long-Term Memories
Procedural Memories
Declarative
Episodic Memories Semantic Memories
Procedural
Knowing how to
serve a tennis ball
The dav that I went
to pick up mv dog
O::v
'Obrigado` is the
word for 'thank-
vou` in Portuguese
Encoding: input to
sensory systems
Storage: second Ior
visual system, 2 sec.
Ior auditory
Retrieval: inIo to STM
Sensory Memory
Short-Term Memory Long-Term Memory
Encoding: STM to
LTM
Storage: by level oI
processing
Retrieval: recall or
recognition
Encoding: sensory
inIo to STM
Storage: maintenance
and elaborative
rehearsal
Retrieval: recognition
or recall or to LTM

Memory CHAPTER 6 -78-
LEARNING OBJECTIVES

5.14 What are the three processes of memorv
and the different models of how memorv
works?
5.15 How does sensorv memorv work?
5.16 What is short-term memorv, and how does it
differ from working memorv?
5.17 How is long-term memorv different from
other tvpes of memorv?
5.18 What are the various tvpes of long-term
memorv, and how is information stored in
long-term memorv organi:ed?
5.19 What kinds of cues help people remember?

5.20 How do the retrieval processes of recall and
recognition differ, and how reliable are our
memories of events?
5.21 How are long-term memories formed, and
how can this process lead to inaccuracies in
memorv?
5.22 What is false memorv svndrome?
5.23 Whv do we forget?
5.24 How and where are memories formed in the
brain?
5.25 How does amnesia occur?
5.26 What are the facts about Al:heimers
disease?


PRACTICE EXAM
For the Iollowing multiple choice questions, select the answer you Ieel best answers the question.

1. is the active system that receives inIormation Irom the senses, organizes and alters it
as it stores it away, and then retrieves the inIormation Irom storage.
a) Classical conditioning
b) Operant conditioning
c) Learning
d) Memory

2. is retention oI memory Ior some period oI time.
a) Encoding
b) Storage
c) Retrieval
d) Evaluation

3. Janie is taking an exam in her history class. On the exam, a question asks her to state and discuss the
Iive major causes oI the Trans-Caspian War (whatever that was!). Janie remembers Iour oI them.
She knows there is a IiIth but time is up. As Janie is walking down the stairs, all oI a sudden, she
remembers that IiIth point but it is too late. Janie had a problem with
a) encoding.
b) storage.
c) retrieval.
d) evaluation.

4. The processes oI encoding, storage, and retrieval are seen as part oI the model oI
memory.
a) inIormation-processing
b) levels-oI-processing
c) parallel distributed processing
d) All oI the above are correct.

Memory CHAPTER 6 -79-

5. The levels-oI-processing concept oI Craik and Lockhart suggests that which oI the Iollowing
questions would lead to better memory oI the word frog?
a) 'Does it rhyme with blog?
b) 'Is it in capital letters?
c) 'Is it written in cursive?
d) 'Would it be Iound in a pond?

6. The three parts oI the inIormation-processing model oI memory include
a) sensory memory, short-term memory, and long-term memory.
b) CS, UCS, UR, CR.
c) encoding, storage, retrieval.
d) shallow, medium, deep processing.

7. Which memory system provides us with a brieI representation oI all the stimuli present at a
particular moment?
a) primary memory
b) sensory memory
c) long-term memory
d) short-term memory

8. Your Iriend asks you a question, and just as you say 'What? you realize what the person said.
Which part oI your memory was maintaining your Iriend`s words?
a) iconic sensory memory
b) echoic sensory memory
c) short-term memory
d) long-term memory

9. Someone a short distance away, to whom you have been paying no attention, quietly speaks your
name, and suddenly you are attending to that conversation. This is an
example oI
a) Broadbent's process oI selective memory.
b) the Phi phenomenon.
c) the cocktail party phenomenon.
d) cue-controlled inhibition.

10. Your proIessor asks you to get up in Iront oI the class and repeat a long list oI numbers that she
reads to you. II you are not given a chance to repeat the numbers to yourselI as she reads them, what
is the longest list oI numbers you will most likely to be able to remember, according to the classic
work oI Dr. George Miller?
a) 2
b) 7
c) 12
d) 25

11. You try to remember a phone number by repeating it over and over to yourselI. What type oI
rehearsal are you using?
a) condensed
b) permanent
c) elaborative
d) maintenance

Memory CHAPTER 6 -80-
12. Long-term memories are encoded in terms oI
a) sounds.
b) visual images.
c) meanings oI words and concepts.
d) all oI the above.

13. Procedural memories are to memories as declarative memories are to
memories.
a) implicit; explicit
b) explicit; implicit
c) general knowledge; personal Iacts
d) personal Iacts; general knowledge

14. Which oI the Iollowing types oI LTM are Iorms oI explicit memory?
a) procedural
b) semantic
c) episodic
d) both semantic and episodic

15. As a young child, you spent hours on your skateboard. AIter several years oI not skating, you jump
on your board as iI you never missed a day. The long-term memory oI how to skate is an example oI
what type oI memory?
a) explicit
b) episodic
c) semantic
d) procedural

16. As you are skating down the street on your skateboard, you think back to the day you accidentally
skated into a parked car and had to go the hospital to get stitches. The memory oI this event would
be described as a(n) memory.
a) procedural
b) implicit
c) episodic
d) semantic

17. According to the semantic network model, it would take more time to answer 'true to which
sentence?
a) 'A salmon is an animal.
b) 'A salmon is a Iish.
c) 'A canary is a bird.
d) All oI these would take the same time.

18. Which oI the Iollowing concepts describes why it is best to take a test in the same room in which
you learned the material?
a) state-dependent learning
b) encoding speciIicity
c) tip-oI-the-tongue phenomenon
d) cocktail party eIIect

Memory CHAPTER 6 -81-

19. While you were studying Ior your history Iinal, you were angry at your roommate Ior playing her
music too loud. II you wanted to maximize your ability to remember the inIormation on the Iinal,
what mood should you be in while you are taking the Iinal?
a) happy
b) sad
c) angry
d) surprised

20. Under most circumstances, when you are intentionally trying to remember an item oI inIormation,
is an easier task than .
a) recognition; recall
b) recall; recognition
c) priming; the savings method
d) the savings method; priming

21. The test you are taking right now requires which type oI memory retrieval process?
a) recall
b) recognition
c) encoding
d) echoic

22. Is eyewitness testimony usually accurate?
a) Yes, because seeing is believing.
b) No, because eyewitnesses are not usually honest.
c) Yes, because eyewitnesses are very conIident about their testimony.
d) No, because there is a great possibility oI a 'Ialse positive identiIication.

23. In this view, memories are literally 'built Irom the pieces stored away at encoding. This view is
called
a) constructive processing.
b) hindsight bias.
c) adaptation oI memory traces.
d) Ilashbulb integration.

24. Which oI the Iollowing phenomena provides support Ior the concept that memories are
reconstructed as they are retrieved or remembered?
a) tip oI the tongue
b) hindsight bias
c) cocktail party eIIect
d) retrograde amnesia

25. Which oI the Iollowing is an example oI the misinIormation eIIect?
a) Iorgetting where you leIt your keys
b) Ialsely remembering that a Iriend was wearing a jacket aIter being asked what color your
Iriend`s jacket was
c) remembering a traumatic event Irom childhood
d) telling someone a lie

Memory CHAPTER 6 -82-

26. Ebbinghaus Iound that inIormation is Iorgotten
a) more rapidly as time goes by.
b) gradually at Iirst, then increasing in speed oI Iorgetting.
c) quickly at Iirst, then tapering oII gradually.
d) most quickly one day aIter learning.

27. Retroactive interIerence as used in the study oI memory reIers to when
a) older inIormation already in memory interIeres with the retrieval oI newer inIormation.
b) newer inIormation interIeres with the retrieval oI older inIormation.
c) the inIormation is not attended to and Iails to be encoded.
d) inIormation that is not accessed decays Irom the storage system over time.

28. Shalissa has two exams today. One is in French and the other is in history. Last night she studied
French beIore history. When she gets to her history test, all she can remember is French! Shalissa`s
memory is suIIering Irom
a) cue-dependent Iorgetting.
b) proactive interIerence.
c) decay.
d) retroactive interIerence.

29. In the Iamous case oI H. M., aIter having part oI his brain removed, he could no longer
a) pay attention to speciIic stimuli.
b) retrieve memories.
c) Iorm new memories.
d) make sense oI memories.

30. The physical processes that occur when a memory is Iormed are called
a) consolidation.
b) actuation.
c) potentiation.
d) depolarization.

31. When a person`s is damaged or removed, anterograde amnesia results.
a) hippocampus
b) preIrontal lobe
c) amygdala
d) Cerebellum

PRACTICE EXAM ANSWERS
1. d Memory involves the three processes oI encoding, storage, and retrieval. The other
choices deal with the process oI learning.
2. b When you store something, you keep it (or retain it) Ior a certain period oI time. In
the study oI memory, the term storage involves keeping or retaining inIormation Ior
a certain period oI time.
3. c Retrieval is the process oI pulling inIormation back out oI memory.
4. d Encoding, storage, and retrieval are the basic processes Ior memory and are a
component oI ALL the theories on exactly how memory works.
5. d The levels-oI-processing model proposes that the 'deeper the level oI processing,
the more likely it is to be remembered. This means that the more meaning or
signiIicance you can give to a piece oI inIormation, the better you remember it.
Associating a Irog with the place it lives is the most meaningIul association oI the
Iour choices.
Memory CHAPTER 6 -83-
6. a All models oI memory include the concepts oI encoding, storage, and retrieval. The
aspects oI the inIormation-processing model that make it unique are the concepts oI
sensory, short-term, and long-term memory.
7. b Sensory memory is the brieIest oI all the memory stages proposed by the
inIormation-processing model. Visual sensory memory lasts only about one-halI a
second.
8. b Echoic memory is the memory oI sounds. It should be easy to remember iI you just
think oI an 'echo Ior echoic.
9. c The cocktail party eIIect is a demonstration oI our selective attention abilities.
Obviously, you are processing all oI the inIormation but you are only 'attending to
a small portion oI it. One place this phenomenon is likely to occur is at a party, thus
the name 'cocktail party eIIect.
10. b The amount oI inIormation we can retain in short-term memory was studied by
George Miller and presented in a paper called 'The magic number 7 plus or minus
two.
11. d Maintenance rehearsal is one oI the most basic methods to remember something and
involves simply repeating the inIormation over and over. Elaborative rehearsal is
more complex and involves Iorming an association with the inIormation.
12. d Memories are encoded in terms oI all these components. One theory suggests that
each component oI a memory is actually stored in a diIIerent place in the brain.
13. a Procedural memories (such as how to ride a bike) are hard to verbalize just as
implicit memories are hard to verbalize. II something is explicit, that means it is
very clear and obvious, just as declarative memories (like the memory oI your Iirst
kiss) are very easy to identiIy.
14. d Semantic memories are memories oI Iacts such as the capital oI the United States.
Episodic memories are memories oI episodes, such as your last birthday celebration.
15. d Procedural memories are memories Ior procedures (or habits and skills).
16. c This is a memory oI a speciIic episode.
17. a For this answer, you need to move across two categories: salmon to Iish to animal.
18. b Encoding speciIicity reIers to your physical surroundings and how they can act as
retrieval cues Ior inIormation.
19. c State-dependent learning reIers to your emotional state and how being in the same
mood during retrieval as you were during the encoding process can help you
remember more inIormation.
20. a Recognition simply requires 'recognizing the right answer. This means you are
given all the options and you simply select the correct choice.
21. b You are given the right answer and you simply have to select it Irom choices ad.
22. d Although eyewitness testimony can be accurate, there is always the possibility oI
Ialse positives.
23. a Constructive processing assumes that all the pieces oI a memory are stored in
diIIerent locations and 're-assembled every time the memory is retrieved.
24. b In hindsight bias, our memory oI a past event is inIluenced by new inIormation.
25. b The misinIormation eIIect occurs when a leading question or statement actually
alters your memory oI an event.
26. c Most Iorgetting occurs within the Iirst hour aIter the material is learned.
27. B Retroactive interIerence occurs when the new inIormation gets in the way or
'interIeres with the already learned material.
28. B Proactive interIerence occurs with the already learned material interIeres with the
new inIormation.
29. c AIter H. M`s hippocampus was removed, he lost the ability to move memories Irom
short-term to long-term memory.

Memory CHAPTER 6 -84-
Memory CHAPTER 6 -85-
30. a The term consolidation reIers to the physical basis oI memories. Researchers are
still working to determine the precise details oI consolidation.
31. a Anterograde amnesia is described as the inability to Iorm any new memories. Just
like the case oI H. M., when a person`s hippocampus is removed or damaged,
anterograde amnesia is oIten the result.

CHAPTER GLOSSARY
anterograde amnesia loss oI memory Irom the point oI injury or trauma Iorward, or the
inability to Iorm new long-term memories.
autobiographical memory the memory Ior events and Iacts related to one`s personal liIe story.
automatic encoding tendency oI certain kinds oI inIormation to enter long-term memory with
little or no eIIortIul encoding.
consolidation the changes that take place in the structure and Iunctioning oI neurons
when an memory is Iormed.
constructive processing reIerring to the retrieval oI memories in which those memories are
altered, revised, or inIluenced by newer inIormation.
curve of forgetting a graph showing a distinct pattern in which Iorgetting is very Iast within
the Iirst hour aIter learning a list and then tapers oII gradually.
decay loss oI memory due to the passage oI time, during which the memory
trace is not used.
declarative memory long-term memory containing inIormation that is conscious and known.
distributed practice spacing the study oI material to be remembered by including breaks
between study periods.
disuse another name Ior decay, assuming that memories that are not used will
eventually decay and disappear.
echoic memory the brieI memory oI something a person has just heard.
eidetic imagery the ability to access a visual memory Ior 30 seconds or more.
elaborative rehearsal a method oI transIerring inIormation Irom STM into LTM by making that
inIormation meaningIul in some way.
encoding set oI mental operations that people perIorm on sensory inIormation to
convert it into a Iorm that is usable in the brain`s storage systems.
encoding failure Iailure to process inIormation into memory.
encoding specificity the tendency Ior memory to be improved iI related inIormation (such as
surroundings or physiological state) that is available when the memory is
Iirst Iormed is also available when the memory is being retrieved.
episodic memory type oI declarative memory containing personal inIormation not readily
available to others, such as daily activities and events.
explicit memory memory that is consciously known, such as declarative memory.
false positive error oI recognition in which people think that they recognize some
stimulus that is not actually in memory.
flashbulb memory type oI automatic encoding that occurs because an unexpected event has
strong emotional associations Ior the person remembering it.
hindsight bias the tendency to Ialsely believe, through revision oI older memories to
include newer inIormation, that one could have correctly predicted the
outcome oI an event.
iconic memory visual sensory memory, lasting only a Iraction oI a second.
implicit memory memory that is not easily brought into conscious awareness, such as
procedural memory.
infantile amnesia the inability to retrieve memories Irom much beIore age 3.
information-processing model oI memory that assumes the processing oI inIormation Ior memory
model storage is similar to the way a computer processes memory, in a series oI
three stages.
levels-of-processing model model oI memory that assumes inIormation that is more 'deeply
processed, or processed according to its meaning rather than just the
sound or physical characteristics oI the word or words, will be
remembered more eIIiciently and Ior a longer period oI time.
long-term memory (LTM) the system oI memory into which all the inIormation is placed to be kept
more or less permanently.
maintenance rehearsal practice oI saying some inIormation to be remembered over and over in
one`s head in order to maintain it in short-term memory.
memory an active system that receives inIormation Irom the senses, puts that
inIormation into a usable Iorm, and organizes it as it stores it away, and
then retrieves the inIormation Irom storage.
memory trace physical change in the brain that occurs when a memory is Iormed.
misinformation effect the tendency oI misleading inIormation presented aIter an event to alter
the memories oI the event itselI.
parallel distributed
processing (PDP) model
a model oI memory in which memory processes are proposed to take
place at the same time over a large network oI neural connections.
primacy effect tendency to remember inIormation at the beginning oI a body oI
inIormation better than the inIormation that Iollows.
proactive interference memory problem that occurs when older inIormation prevents or
interIeres with the learning or retrieval oI newer inIormation.
procedural (nondeclarative)
memory
type oI long-term memory including memory Ior skills, procedures,
habits, and conditioned responses. These memories are not conscious but
are implied to exist because they aIIect conscious behavior.
recall type oI memory retrieval in which the inIormation to be retrieved must be
'pulled Irom memory with very Iew external cues.
recency effect tendency to remember inIormation at the end oI a body oI inIormation
better than the inIormation at the beginning oI it.
recognition ability to match a piece oI inIormation or a stimulus to a stored image or
Iact.
retrieval getting inIormation that is in storage into a Iorm that can be used.
retrieval cue a stimulus Ior remembering.
retroactive interference memory problem that occurs when newer inIormation prevents or
interIeres with the retrieval oI older inIormation.
retrograde amnesia loss oI memory Irom the point oI some injury or trauma backwards, or
loss oI memory Ior the past.
selective attention the ability to Iocus on only one stimulus Irom among all sensory input.
semantic network model model oI memory organization that assumes that inIormation is stored in
the brain in an connected Iashion, with concepts that are related stored
physically closer to each other than concepts that are not highly related.
sensory memory the very Iirst stage oI memory, the point at which inIormation enters the
nervous system through the sensory systems.
serial position effect tendency oI inIormation at the beginning and end oI a body oI
inIormation to be remembered more accurately than inIormation in the
middle oI the body oI inIormation.
short-term memory (STM) the memory system in which inIormation is held Ior brieI periods oI time
while being used.
storage holding onto inIormation Ior some period oI time.
working memory an active system that processes the inIormation in short-term memory.
Memory CHAPTER 6 -86-
CHAPTER 7 - COGNITION: THINKING, INTELLIGENCE, AND LANGUAGE


YOU KNOW YOU ARE READY FOR THE TEST IF YOU ARE ABLE TO.
Introduce the concept oI cognition, as it relates to mental images, concepts and problem solving.
Describe creative thinking and the diIIerences between convergent and divergent thinking.
Discuss the measurement oI intelligence including the StanIord-Binet and Wechsler intelligence
tests, test construction issues, and the determination oI developmental delay.
Describe several prominent theories oI intelligence including the concepts oI giItedness,
intellectual disability, heredity, and environment.
Explain the basis oI language and the relationship between language and thought processes.

RAPID REVIEW
Thinking, or cognition, can be deIined as mental activity that goes on in the brain when a person is
processing inIormation. Cognition includes both verbal and nonverbal processes. Two examples oI
cognition are mental images, which are picture-like representations that stand in Ior objects or events,
and concepts, or ideas that represent a class oI objects. Concepts can be ranked Irom general to speciIic
by applying the terms superordinate, basic level type, and subordinate. Formal concepts are deIined
by speciIic rules, while natural concepts are Iormed as a result oI experience. A prototype is a speciIic
example oI a concept that closely resembles the deIining Ieatures oI a concept. Concepts are Iormed
through experience and culture and have an impact on our thinking.
Problem solving involves using our thoughts or cognitions to reach a goal and consists oI at least Iour
diIIerent techniques. Trial-and-error problem solving makes use oI mechanical solutions. When
someone uses algorithms to problem-solve they are Iollowing step-by-step procedures to solve the
problem. Heuristics are general 'rules oI thumb that can be applied to many situations. Representative
heuristics are assumptions that any object (or person) that shares characteristics with the members oI a
particular category is also a member oI that category. Availability heuristics reIer to the act oI estimating
the Irequency or likelihood oI an event based on how easy it is to recall relevant inIormation Irom
memory or how easy it is Ior us to think oI related examples. Means-end analysis is an example oI one
type oI heuristic where the diIIerence between where you are and where you want to be is determined and
then steps are taken to reduce that diIIerence. Insight consists oI solving the problem by having a sudden
moment oI inspiration or 'aha! moment.
Some Iactors that interIere with problem solving include the Iollowing: Functional fixedness occurs
when a person thinks about objects only in terms oI their typical uses. Mental sets are tendencies to use
the same problem-solving strategies that worked in the past. Confirmation bias reIers to the act oI
searching Ior evidence that Iits your belieIs while ignoring any contradictory inIormation. Creativity
occurs when a person solves a problem by combining ideas and behaviors in a new way. Many methods
oI problem solving utilize convergent thinking, which assumes that one single answer exists Ior the
problem. Divergent thinking is the opposite process oI convergent thinking. When an individual uses
divergent thinking, he or she starts Irom one point and comes up with many possibilities or ideas based on
that point.
Intelligence can be deIined as the ability to learn Irom one`s experiences, acquire knowledge, and use
resources eIIectively in adapting to new situations or solving problems. Currently, there is still much
disagreement on exactly what is meant by the term intelligence. In 1904, Charles Spearman proposed
that intelligence was split between two abilities. The Iirst ability was a general intelligence, labeled the g
factor, and the other was a speciIic intelligence reIerred to as the s factor. Spearman believed that both
the g and s Iactors could be measured using standardized intelligence tests. Howard Gardner, on the other
hand, proposed that at least nine diIIerent kinds oI intelligence exist. Robert Sternberg proposed the
triarchic theory of intelligence, which states that intelligence can be divided into three types: analytical,
creative, and practical intelligence.
Cognition CHAPTER 7 -87-
In France in 1916, AlIred Binet and Theodore Simon developed the Iirst Iormal test Ior intelligence in
order to determine a child`s mental age. The StanIord-Binet test used a ratio oI mental age to
chronological age to determine an individual`s intelligence quotient or IQ. In the United States, the
Wechsler intelligence tests are now used more Irequently that the StanIord-Binet, and IQ scores are now
based on individual deviation IQ scores rather than a ratio. The Wechsler tests are designed Ior speciIic
age groups and can be administered individually. To determine the quality oI a psychological test, you
need to look at the test`s validity, reliability, and procedure used to obtain the norms. Validity reIers to
how well the test measures what it claims to measure. Ecological validitv is the extent to which an
obtained score accurately reIlects the intended skill or outcome in real liIe situations, not just the testing
or assessment situation. Reliabilitv indicates the test`s ability to produce the same result when given to the
same person under similar conditions. Norms are determined by the standardization group selected by
the researchers and should be a representative sample oI the population who will be taking the test. All
psychological tests should also be examined Ior the cultural biases. Adrian Dove created an intelligence
test called the Dove Counterbalance General Intelligence Test (also known as the Chitling Test) to
demonstrate the cultural biases present in many oI the intelligence tests currently in use.
Intellectual disability, which may also be reIerred to as developmental delay (Iormerly known as
mental retardation), occurs in about 3 percent oI the U.S. population and is deIined by an IQ score oI 70
(two standard deviations below the mean on the normal curve) or lower along with adaptive behaviors
signiIicantly below the expected level Ior the person`s age group. Diagnosis oI intellectual disability is
determined by the person`s intellectual and adaptive behavior skills, psychological and emotional levels,
physical health considerations, and environmental Iactors. Recent recommendations Irom the American
Association on Intellectual and Developmental Disabilities also require deIicits in more than one oI these
areas. Intellectual disability is classiIied Irom mild to moderate, severe, and proIound. Environmental
conditions during a child`s development can aIIect brain Iunction and may inIluence the development oI
intellectual disability. The three most common biological causes oI intellectual disability are Down
syndrome, Ietal alcohol syndrome, and Iragile X syndrome.
Individuals who receive scores oI 130 or above on intelligence tests are reIerred to as gifted. Lewis
Terman conducted a longitudinal study oI the traits and behaviors oI more than 1,500 giIted children. The
children were known as Terman`s Termites and his Iindings showed that many oI the common myths
about the 'nutty genius were unIounded.
More recently, the concept oI emotional intelligence has been suggested as an important Iactor Ior
success in liIe. Further research in this area is still needed. The role oI a person`s environment, or
nurture, and heredity, also reIerred to as nature, on the development oI intelligence continues to be
debated. Studies oI identical and Iraternal twins raised together and apart have provided one method Ior
investigating the role oI nature and nurture.
Language is deIined as a system Ior combining symbols (such as words) so that an unlimited number
oI meaningIul statements can be made Ior the purpose oI communicating with others. Language can be
analyzed at many levels. Semantics is the rules Ior determining the meaning oI words and sentences.
Phonemes are the most basic units oI sounds used in a speciIic language, and morphemes combine the
units oI sound into the smallest units that have meaning. Grammar includes all the rules Ior combining
morphemes into words, and syntax is the rules Ior combining words into sentences. Pragmatics deals
with the practical aspects oI communicating with others. The relationship between language and thought
has been studied extensively. The Sapir-Whorf hypothesis, also known as the linguistic relativity
hypothesis, proposes that the words people use determine how they think about themselves and the
world. An opposing theory, known as cognitive universalism, proposes that certain ways oI thinking are
shared among all groups oI people and inIluence the development oI language in similar ways. Animals
other than humans demonstrate a diverse ability to communicate but it is unclear whether they have the
capability Ior language as demonstrated by the ability to use abstract symbols to communicate. Kanzi, a
bonobo chimpanzee, has demonstrated an ability to understand about 150 spoken English words.
However, none oI the animals studied to date appear to have been able to use and comprehend syntax.
In recent years, a number oI computerized brain training programs and devices have been marketed to
the public as a means to improve cognitive Iunction. However, the most recent research suggests that,
Cognition CHAPTER 7 -88-
although practicing certain mental skills through cognitive exercises appears to help with those same
skills when tested later, these beneIits don`t appear to transIer to untrained areas. In other words,
cognitive exercises may result in very speciIic improvements that do not necessarily translate to enhanced
cognitive perIormance in the real world.

STUDY HINTS
13.

In this chapter, you were presented with Iour diIIerent approaches to problem solving. In order to
better understand how these approaches diIIer Irom each other, take the Iollowing problem and
come up with an example oI how you could solve the problem using each oI the Iour diIIerent
approaches.

Problem: You are packing up to move to college and you have one more box to Iit in the trunk oI
your car but it looks like there is simply no room leIt. You don`t want to leave the box behind.
How will you solve this problem?

Approach Solution
trial-and-error

algorithm

heuristics

insight



14.


The two most commonly used methods to assess any psychological test are to determine the
validity and reliability oI the test. Examine the Iollowing test descriptions and determine whether
the test has a potential problem with its reliability or validity.

Example Validity or Reliability Issue?
A personality test gives a very diIIerent score
Ior the same person when he retakes it six
months later.

An individual takes an online IQ test that
measures how long she can hold her breath.

A 5-year-old child is diagnosed as
developmentally delayed based on his IQ
scores, but when he is brought back and
given the same test, his scores Iall in the
above average range.



Cognition CHAPTER 7 -89-
Suggested answers to Question 1

Approach Solution
trial-and-error
Keep placing the box in various places and positions in vour car until
vou find one that works.
algorithm
Go online and find a website that deal with phvsics. Enter in the
dimensions of vour car and the exact dimensions of everv box and item
that vou are trving to fit in vour car. Get a printout of the optimal
placement for each box and follow it step bv step to fit evervthing in.
heuristics
Think back to how vour mom alwavs told vou to pack the big things first
and then squee:e the little ones in. Take vour boxes out and pack them
again using this general rule of thumb to guide vou.
insight
Sit back with vour friends for a few minutes and relax. As vou are talking
with vour friends, all of a sudden vou remember that vour familv has a
'Big Mac` container that will attach to the top of the car. Strap the
container on, place vour box in the container, and take off for school.


Suggested answers to Question 2

Example Validity or Reliability Issue?
A personality test gives a very diIIerent score
Ior the same person when he retakes it six
months later.
reliabilitv the scores are not consistent over
time for the same person
An individual takes an online IQ test that
measures how long she can hold her breath.
validitv does holding vour breath give a
verv accurate assessment of vour IQ?
A 5-year-old child is diagnosed as
developmentally delayed based on his IQ
scores, but when he is brought back and
given the same test, his scores Iall in the
above average range.
This question illustrates that without
reliabilitv a test will also lack validitv. The
test scores are inconsistent over time, which
indicates that the test is not reallv measuring
what it claims to measure when we assume
that intelligence is a fairlv constant factor.

LEARNING OBJECTIVES
5.27 How are mental images and concepts
involved in the process of thinking?
5.28 What are the methods people use to solve
problems and make decisions?
5.29 Whv does problem solving sometimes fail,
and what is meant bv creative thinking?
5.30 How do psvchologists define intelligence,
and how do various theories of intelligence
differ?
5.31 How is intelligence measured, and how are
intelligence tests constructed?
"#$% What is intellectual disabilitv and what are
its causes?
5.33 What defines giftedness, and does being
intellectuallv gifted guarantee success in
life?
5.34 What is the influence of hereditv and
environment on the development of
intelligence?
5.35 How is language defined, and what are its
different elements and structure?
5.36 Does language influence the wav people
think, and are animals capable of learning
language?
5.37 What are some wavs to improve thinking?


Cognition CHAPTER 7 -90-
PRACTICE EXAM
For the Iollowing multiple choice questions, select the answer you Ieel best answers the question.

1. Mental images
a) represent abstract ideas.
b) have a picture-like quality.
c) consist entirely oI unconscious inIormation.
d) are always prototypes.

2. II three people used mental images to tell you how many windows they each had in their individual
houses, which person would take the longest to answer?
a) the person with two windows in his or her house
b) the person with eight windows in his or her house
c) the person with twelve windows in his or her house
d) They would all take the same amount oI time to answer.

3. Concepts are ideas that represent
a) a class or category oI objects, events, or activities.
b) thoughts, images, muscle patterns oI behavior.
c) higher-order conditioning and secondary reinIorcers.
d) representations that stand in Ior objects or events and have a picturelike quality

4. A very general Iorm oI a concept, such as 'vegetable represents which concept level?
a) subordinate
b) superordinate
c) basic level
d) hyperordinate

5. The trial-and-error method oI solving problems is also known as
a) the use oI a heuristic device.
b) the use oI algorithms.
c) the mechanical solution.
d) the A.I. Solution.

6. Zach could not remember the Iour-digit combination needed to open the lock on his bicycle. AIter
struggling to Iigure out what to do, he turned to start the long walk home and all oI a sudden he
remembered the combination to the lock. The problem-solving strategy Zach used would be best
described as
a) trial-and-error.
b) algorithm.
c) a heuristic.
d) insight.

7. The tendency Ior people to persist in using problem-solving patterns that have worked Ior them in
the past is known as
a) mental set.
b) conIirmation bias.
c) creativity.
d) divergent thinking.

Cognition CHAPTER 7 -91-

8. Luann needs to hammer a nail into the wall but the only tool she can Iind in the house is a
screwdriver. Luann`s inability to see how the handle oI the screwdriver could be used as a hammer,
best represents the concept oI
a) Iunctional Iixedness.
b) conIirmation bias.
c) creativity.
d) artiIicial bias.

9. The ability to produce solutions to problems that are unusual, inventive, novel, and appropriate is
called
a) creativity.
b) insight.
c) heuristics.
d) latent learning.

10. Which oI the Iollowing activities would not increase your creativity?
a) keeping a journal
b) brainstorming
c) subject mapping
d) convergent thinking

11. The ability to understand the world, think rationally or logically, and use resources eIIectively when
Iaced with challenges or problems, or the characteristics needed to succeed in one`s culture is the
psychologist`s working deIinition oI
a) divergent problem solving.
b) creative thinking.
c) heuristic usage.
d) intelligence.

12. Measuring intelligence by testing is a rather new concept in the history oI the world. It is
roughly years old.
a) 50
b) 100
c) 200
d) 500

13. An 8-year-old child who scored like an average 10-year-old on an intelligence test would have a
mental age oI and an IQ oI .
a) 8; 80
b) 8; 125
c) 10; 100
d) 10; 125

14. Because oI the need to measure the IQ oI people oI varying ages, newer IQ tests base their
evaluation oI IQ on
a) mental age alone.
b) deviation scores Irom the mean oI the normal distribution.
c) giving extra points Ior older Iolks to compensate Ior their slower processing times.
d) none oI these.

Cognition CHAPTER 7 -92-

15. II a test consistently produces the same score when administered to the same person under identical
conditions, that test can be said to be high in
a) reliability.
b) validity.
c) accuracy.
d) norms.

16. George is a successIul organic Iarmer. On which oI Gardner's nine types oI intelligence would
George be most likely to have a high score?
a) Verbal/linguistic
b) Movement
c) Intrapersonal
d) Naturalist

17. Which two oI the Iollowing aspects are included in the deIinition oI intellectual disability?
a) IQ scores and adaptive behavior
b) age and socioeconomic status
c) race and country oI origin
d) Only IQ scores are considered.

18. Which oI the Iollowing statements about the giIted is true?
a) They are more likely to suIIer Irom mental illnesses.
b) They are physically weaker than nongiIted persons.
c) They are oIten skilled leaders.
d) They are socially unskilled.

19. Which was not a Iinding oI the Terman and Oden (1974) study oI giIted kids?
a) They were socially well-adjusted.
b) They were more resistant to mental illness.
c) They were clearly much more likely to be Iemales.
d) They were average in weight, height, and physical attractiveness.

20. Sternberg has Iound that intelligence is a good predictor oI success in liIe, but has a
low relationship to intelligence.
a) practical; analytical
b) practical; creative
c) analytical; practical
d) academic; creative

21. What three types oI intelligence constitute Sternberg's triarchic theory oI intelligence?
a) global, intuitive, and special
b) general, global, and speciIic
c) analytical, creative, and practical
d) mathematical, reasoning, and verbal

22. The 'g in Spearman`s g Iactor oI intelligence stands Ior
a) giIted intelligence.
b) general intelligence.
c) graded intelligence.
d) The g does not stand Ior anything.

Cognition CHAPTER 7 -93-
23. II intelligence is determined primarily by heredity, which pair should show the highest correlation
between IQ scores?
a) Iraternal twins
b) identical twins
c) brothers and sisters
d) parents and children

24. II a researcher believed that nature was the most important Iactor in determining an individual`s
intelligence level, she would most closely agree with which oI the Iollowing statements?
a) Intelligence is largely inherited Irom your parents.
b) Intelligence has no relationship to your biological Iamily.
c) The environment is the most important Iactor in determining a child`s intelligence level.
d) A child`s intelligence can be greatly increased by providing stimulating toys throughout
inIancy.

25. Language, by deIinition,
a) is symbolic.
b) can be written, spoken, or signed.
c) is capable oI an inIinite set oI meaningIul utterances.
d) includes all oI these characteristics.

26. The basic units oI sound are called
a) morphemes.
b) phonemes.
c) semantics.
d) syntax.

27. Syntax is
a) a system oI rules Ior combining words and phrases to Iorm sentences.
b) the smallest units oI meaning within a language.
c) the basic units oI sound.
d) the rules to determine the meaning oI words.

28. The linguistic relativity hypothesis suggests that
a) one's language determines the pattern oI one's thinking and view oI the world.
b) one's thinking and view oI the world determines the structure oI one's language.
c) we decide which objects belong to a concept according to what is most probable or
sensible, given the Iacts at hand.
d) perception oI surIace structure precedes deep structure in understanding a sentence.

29. Which theory would support the idea that certain concepts are shared by all people regardless oI the
language spoken?
a) Sapir-WhorI hypothesis
b) linguistic relativity hypothesis
c) cognitive universalism
d) heuristic theory

Cognition CHAPTER 7 -94-

30. Dolphins, according to TV and movies, are very intelligent and have strong language abilities. They
might even be able to talk! However, which statement is true Irom the research?
a) Dolphins have been shown to master syntax.
b) Dolphins have the language abilities oI a 3-year-old.
c) Dolphin communication with parrots has been Iirmly established.
d) None oI these are true.

PRACTICE EXAM ANSWERS
1. b Mental images are mental representations oI objects that have a picture-like quality.
2. c Research has Iound that iI the individuals used mental images to answer the
question, they would actualize visualize the house and have to count the windows,
so the person with the most windows would take the longest time to answer.
3. a The deIinition Ior concepts is that they are ideas that represent a class or category oI
objects or events.
4. b Superordinate is the highest or most general level oI a concept. Basic level is the
level most commonly used (such as potato or lettuce), subordinate is the most
speciIic such as a russet potato or romaine lettuce.
5. c Trial-and-error problem solving tries one solution aIter another until one that works
is Iound.
6. d Insight problem solving occurs when you get a sudden inspiration that leads you to
the solution to your problem.
7. a A mental set exists when someone continues to use the same approaches that
worked in the past. ConIirmation bias occurs when someone pays attention to
inIormation that conIirms his ideas and ignores any contradictory input.
8. a Functional Iixedness occurs when an individual is fixed on only one function oI a
particular object.
9. a This is the deIinition oI creativity.
10. d Convergent thinking occurs when you assume there is only one single answer or
solution to a problem. Typically, convergent thinking decreases creative ability.
11. d As can be seen, intelligence is a broad idea that can be diIIicult to deIine.
12. b AlIred Binet started testing children in France in 1916.
13. d The IQ is based on a mental age oI 10 divided by a chronological age oI 8 and
multiplied by 100. This gives an IQ 125.
14. b Deviation IQ scores are based on the norms oI a representative sample oI the
population (also known as the standardization group).
15. a Reliability indicates a test consistency, while validity indicates accuracy, or how
well the test measures what it says it measures.
16. d Naturalist intelligence is the ability to recognize the patterns Iound in nature, which
would help to make George a successIul organic Iarmer.
17. a The diagnosis oI intellectual disability is based on IQ scores as well as how well the
individual is able to Iunction in day-to-day liIe.
18. c Skilled leaders oIten are giIted individuals; the other three statements are myths that
have not been supported by research.
19. c Slightly more males than Iemales were selected Ior the Terman study.
20. a Sternberg has Iound that practical intelligence is a good predictor oI success in liIe,
but has a low relationship to analytical intelligence.
21. c Sternberg proposed that intelligence should actually be broken down into three
components that can be thought oI as book smarts, street smarts, and creativity.
22. b Spearman proposed a two-Iactor theory oI intelligence. The g Iactor was Ior general
intelligence and the s Iactor was Ior speciIic intelligence.
23. b Identical twins should show the strongest correlation since they share 100 percent oI
Cognition CHAPTER 7 -95-
the same genes.
24. a Nature reIers to the inIluence oI heredity on behaviors and traits.
25. d The deIinition oI language includes all three oI these attributes.
26. b Phonemes are the basic units oI sound.
27. a Syntax reIers to the rules we use to Iorm meaningIul sentences.
28. a Linguistic relativitv hypothesis (also reIerred to as the Sapir-WhorI hypothesis)
states that our thought processes are relative to the language (or linguistic setting)
in which we grew up.
29. c Cognitive universalism proposes that our basic thought processes, or cognitions, are
universally shared by all people.
30. d Chimpanzees have demonstrated a vocabulary equal to a 2-year-old child, but no
animal to date has demonstrated the ability to use and comprehend syntax.

CHAPTER GLOSSARY
algorithms very speciIic, step-by-step procedures Ior solving certain types oI problems.
analytical intelligence the ability to break problems down into component parts, or analysis, Ior
problem solving.
availability heuristic estimating the Irequency or likelihood oI an event based on how easy it is to
recall relevant inIormation Irom memory or how easy it is Ior us to think oI
related examples.
basic level type an example oI a type oI concept around which other similar concepts are
organized, such as 'dog, 'cat, or 'pear.
cognition (thinking) mental activity that goes on in the brain when a person is organizing and
attempting to understand inIormation, and communicating inIormation to
others.
cognitive universalism theory that concepts are universal and inIluence the development oI language.
concepts ideas that represent a class or category oI objects, events, or activities.
confirmation bias the tendency to search Ior evidence that Iits one`s belieIs while ignoring any
evidence that does not Iit those belieIs.
convergent thinking type oI thinking in which a problem is seen as having only one answer, and
all lines oI thinking will eventually lead to that single answer, using previous
knowledge and logic
creative intelligence the ability to deal with new and diIIerent concepts and to come up with new
ways oI solving problems.
creativity the process oI solving problems by combining ideas or behavior in new ways.
deviation IQ score a type oI intelligence measure that assumes that IQ is normally distributed
around a mean oI 100 with a standard deviation oI about 15.
divergent thinking type oI thinking in which a person starts Irom one point and comes up with
many diIIerent ideas or possibilities based on that point.
emotional intelligence the awareness oI and ability to manage one`s own emotions as well as the
ability to be selI-motivated, able to Ieel what others Ieel, and socially skilled.
formal concepts concepts that are deIined by speciIic rules or Ieatures.
functional fixedness a block to problem solving that comes Irom thinking about objects in terms oI
only their typical Iunctions.
g factor the ability to reason and solve problems, or general intelligence.
gifted the 2 percent oI the population Ialling on the upper end oI the normal curve
and typically possessing an IQ oI 130 or above.
grammar the system oI rules governing the structure and use oI a language.
heuristic an educated guess based on experiences that help narrow down the possible
solutions Ior a problem. Also known as a 'rule oI thumb.
Cognition CHAPTER 7 -96-

intellectual disability condition in which a person`s behavioral and cognitive skills exist at an
earlier developmental stage than the skills oI others who are the same
chronological age; may also be reIerred to as developmentally delayed. This
condition was Iormerly known as mental retardation.
intelligence the ability to learn Irom one`s experiences, acquire knowledge, and use
resources eIIectively in adapting to new situations or solving problems.
intelligence quotient
(IQ)
a number representing a measure oI intelligence, resulting Irom the division
oI one`s mental age by one`s chronological age and then multiplying that
quotient by 100.
language a system Ior combining symbols (such as words) so that an unlimited number
oI meaningIul statements can be made Ior the purpose oI communicating with
others.
Lewis Terman 18771956. Cognitive psychologist well known Ior his longitudinal study oI
giIted children, aIIectionately reIerred to as Terman`s Termites.
linguistic relativity
hypothesis
the theory that thought processes and concepts are controlled by language.
means-end analysis heuristic in which the diIIerence between the starting situation and the goal is
determined and then steps are taken to reduce that diIIerence.
mental images mental representations that stand in Ior objects or events and have a
picturelike quality.
mental set the tendency Ior people to persist in using problem-solving patterns that have
worked Ior them in the past.
morphemes the smallest units oI meaning within a language.
natural concepts concepts people Iorm as a result oI their experiences in the real world.
nurture the role a person`s environment plays in his or her development.
phonemes the basic units oI sound in language.
practical intelligence the ability to use inIormation to get along in liIe and become successIul.
pragmatics aspects oI language involving the practical ways oI communicating with
others, or the social 'niceties oI language.
problem solving process oI cognition that occurs when a goal must be reached by thinking and
behaving in certain ways.
prototype an example oI a concept that closely matches the deIining characteristics oI a
concept.
reliability the tendency oI a test to produce the same scores again and again each time it
is given to the same people.
representative heuristic assumption that any object (or person) sharing characteristics with the
members oI a particular category is also a member oI that category.
s factor the ability to excel in certain areas, or speciIic intelligence.
semantics the rules Ior determining the meaning oI words and sentences.
subordinate concept the most speciIic category oI a concept, such as one`s pet dog or a pear in
one`s hand; subordinate reIers to lowest in status or standing.
superordinate concept the most general Iorm oI a type oI concept, such as 'animal or 'Iruit;
superordinate reIers to highest in status or standing.
syntax the system oI rules Ior combining words and phrases to Iorm grammatically
correct sentences.
thinking (cognition) mental activity that goes on in the brain when a person is organizing and
attempting to understand inIormation, and communicating inIormation to
others.
Cognition CHAPTER 7 -97-
Cognition CHAPTER 7 -98-

trial and error
(mechanical solution)
problem-solving method in which one possible solution aIter another is tried
until a successIul one is Iound.
triarchic theory of
intelligence
Sternberg`s theory that there are three kinds oI intelligences: analytical,
creative, and practical.
validity the degree to which a test actually measures what it`s supposed to measure.

CHAPTER 8 - DEVELOPMENT ACROSS THE LIFE SPAN


YOU KNOW YOU ARE READY FOR THE TEST IF YOU ARE ABLE TO.
Introduce the research methods and major issues in developmental psychology, including the
nature versus nurture controversy.
Describe the stages oI prenatal development and potential hazards.
Discuss the theories oI Piaget and Vygotsky with regards to cognitive development.
Describe the physical and cognitive development in inIancy and childhood including language
development.
Explain the concept oI personality including the idea oI temperament, attachment theory, and
Erikson`s psychosocial model.
IdentiIy the major stages oI development in adolescence and adulthood.
Discuss three theories oI aging and Kbler-Ross`s stages oI dying.
Understand cross-cultural views on death.

RAPID REVIEW
Human development is the scientiIic study oI the changes that occur in people as they age Irom
conception to death. Because age cannot be directly manipulated by a researcher, developmental
psychologists have had to develop alternative methods to investigate the eIIects oI aging on psychological
processes. Three common methods used are longitudinal, cross-sectional, and cross-sequential studies.
Longitudinal studies have the advantage oI Iollowing the same subject across time but are limited due to
the amount oI time and money required to complete the study and the problem oI attrition. Cross-
sectional studies are cheaper, Iaster, and easier to conduct because they gather inIormation Irom diIIerent
age groups at one particular period oI time; however, results Irom these studies may be conIounded due to
individual and history diIIerences. Cross-sequential studies are a combination oI longitudinal and cross-
sectional techniques and oIten represent an ideal compromise. One oI the biggest debates among
developmental psychologists is the question oI nature versus nurture. Nature reIers to the inIluence oI
everything you inherit genetically Irom your biological parents and nurture reIers to the inIluence oI your
environment on your development. More recently, the question oI interest has switched Irom nature
versus nurture to the interaction oI nature and nurture. Behavioral genetics is the Iield oI science that
studies the interactions oI nature, or genes, and nurture, or the environment.
Genetics is the science oI heredity and involves the study oI DNA, genes, and chromosomes. DNA
(deoxyribonucleic acid) is the smallest unit oI the three and is composed oI strands oI molecules linked
together like a twisted ladder. The links are made up oI amines and their names are abbreviated with the
letters A, T, G, and C. The next largest unit is the genes, which are sections oI the ladder containing
instructions on how to make a speciIic protein. One way to think oI genes is as individual recipes Ior
proteins. The biggest unit is the chromosomes, which are long strands oI DNA twisted together and
wound up in coils. The chromosomes are Iound in the nucleus oI all the cells oI your body except red
blood cells. Humans have a total oI 46 chromosomes, 23 Irom the mother`s egg and 23 Irom the Iather`s
sperm. Each chromosome Irom the mother matches a chromosome Irom the Iather to Iorm 23 pairs. Both
chromosomes in the pair have the same genes (Ior example, each chromosome contains a gene Ior hair
color). Even though they contain the same gene, the instructions on that gene might be slightly diIIerent;
Ior example, one oI the genes may have the instructions Ior blonde hair while the other gene may contain
the instructions Ior brown hair. The Iirst 22 pairs oI chromosomes are called autosomes, and the last pair
(the 23rd) is called the sex chromosomes and contains the instructions Ior determining sex. Dominant
genes are the genes that are more likely to inIluence the trait. Recessive genes are not as strong and will
only get their instructions carried out iI the other chromosome in the pair also contains a recessive gene.
In reality, almost all traits are determined by multiple gene pairs. This is called polygenic inheritance.
Some diseases result Irom problems with recessive genes and are only expressed when both parents have
the recessive gene, while some disorders result Irom the wrong number oI chromosomes in the Iertilizing
Development CHAPTER 8 -99-
egg or sperm. Some examples oI chromosomal abnormalities include Down syndrome (e.g., Trisomy 21),
KlineIelter`s syndrome (e.g., XXY), and Turner`s syndrome (e.g., a single X chromosome).
Many people believe that conception represents the beginning oI liIe. Fertilization occurs when the
sperm penetrates the egg (or ovum). The result is a single cell with 46 chromosomes (23 Irom the sperm
and 23 Irom the egg). This cell is called a zygote. Monozygotic (or identical) twins result Irom the
zygote splitting into two separate masses early in the division process. Dizygotic (or fraternal) twins
result Irom two eggs being Iertilized by two separate sperm. Conjoined twins result Irom an incomplete
separation oI the zygotic mass. Abby and Brittany Hensel are an example oI conjoined twins.
The germinal period oI pregnancy is the Iirst two weeks aIter Iertilization during which the zygote
migrates down to the uterus and attaches to the uterine wall. The placenta and umbilical cord both begin
to develop during this period. The embryonic period lasts Irom about 2 weeks to 8 weeks, aIter which
the embryo is about one inch long with primitive eyes, nose, lips, teeth, arms and legs. Critical periods
are times in development during which an environmental inIluence can impact the development oI the
Ietus. DiIIerent organs and structures have diIIerent critical periods. The environmental inIluences that
can negatively impact the development oI the Ietus are called teratogens. The fetal period lasts Irom the
eighth week aIter conception to the end oI the pregnancy. Tremendous growth oI the fetus occurs during
this time. A baby born beIore the 38th week oI pregnancy is considered preterm and may need liIe
support to survive, especially iI he or she weighs less than 5 pounds. Most miscarriages, also called
spontaneous abortions, occur in the Iirst three months oI a pregnancy.
InIants have a large number oI capabilities even immediately aIter birth. Most inIants are able to
perIorm Iive innate reIlexes: the grasping reIlex, the startle reIlex, the rooting reIlex, the stepping reIlex,
and the sucking reIlex. In an inIant, touch is the most well-developed sense, Iollowed by smell and taste.
Vision is the least Iunctional oI the senses. In terms oI photoreceptors, the rods are developed at birth but
cones must develop over a six-month time period. At birth, an inIant`s vision is most clear seven to ten
inches Irom their Iace. InIants appear to show a preIerence Ior the human voice and human Iaces. The
visual cliII experiment provides evidence that inIants may also possess depth perception.
The brain triples in weight during the Iirst two years oI liIe due to the expansion oI existing cells not
the growth oI new ones. 1ean Piaget believed that the primary Iactor in the development oI a child`s
cognitive abilities was the child`s interactions with objects in the environment. Piaget believed that
children Iorm mental concepts or schemes as they experience new situations and events. He proposed
Iour stages oI cognitive development Irom inIancy to adolescence. The sensorimotor stage, lasting Irom
inIancy to age 2, involves the use oI the senses and muscles to learn about the environment and includes
the development oI object permanence and symbolic thought. The preoperational stage lasts Irom age
2 to 7 and involves language and concept development through the process oI asking questions. Children
in this stage display the ability oI symbolic thought through make-believe play and also display
characteristics oI egocentrism, centration, and irreversibility. The concrete operational stage lasts
Irom age 7 to 12 and includes the development oI concepts such as conservation and reversible thinking.
However, children in this stage are still unable to deal with abstract concepts such as Ireedom. The
formal operational stage is the Iinal stage oI cognitive development, according to Piaget, and lasts Irom
the age oI 12 on. During this stage abstract, hypothetical thinking develops. Research suggests that only
about one-halI oI the adults in the U.S. reach this stage oI cognitive development. Piaget`s concepts have
been successIully applied in schools but have also been criticized Ior their emphasis on distinct stages oI
development, overemphasis on egocentrism, and Iailure to mention the role oI the Iamily or social
environment in the child`s development.
Lev Vygotsky was a Russian psychologist who Ielt the primary Iactor in development was the social
environment. He proposed a concept called scaffolding in which a more highly skilled person gives the
learner help and then stops as the learner develops on his or her own. Vygotsky believed that each child
has a zone of proximal development or ZPD, which is the diIIerence between what a child can do alone
and what he or she can do with the help oI a teacher. Vygotsky`s principles have been applied in the
classroom through the use oI cooperative learning and reciprocal teaching.
Psychologists interested in inIormation-processing theory have investigated the memory capabilities
oI the developing inIant and have Iound that inIants demonstrate memory Irom birth. Four- to 5-year-olds
Development CHAPTER 8 -100-
appear to be able to store about three items in their short-term memory and have both episodic and
procedural components oI long-term memory. Language development appears to be an important step in
cognitive development and Iacilitates the development oI symbolic thinking. The stages oI language
development experienced by all speakers include cooing around 2 months oI age, babbling at 6 months
oI age, one-word speech or holophrases around 1 year oI age, telegraphic speech at around a year and a
halI, and then whole sentences.
Temperament reIers to the behavioral and emotional characteristics observed in inIancy. Several
researchers have suggested the existence oI three types oI temperaments, easy, diIIicult, and slow to warm
up. Attachment is the emotional bond between an inIant and caregiver. Mary Ainsworth and others
studied attachment using the Strange Situation and observed Iour attachment styles, secure, avoidant,
ambivalent, and disorganized/disoriented. Harry Harlow studied the nature oI attachment behaviors by
observing Rhesus monkeys interact with two diIIerent 'surrogate mothers. He Iound that contact
comfort was an important Iactor in attachment.
Erik Erikson, who originally trained as a Freudian psychoanalyst, proposed an eight-stage theory oI
development that occurred over the entire liIe span. Each stage involved an emotional crisis in the
individual`s social interactions. The Iirst Iour stages occurred during inIancy and childhood and consisted
oI the crises oI trust versus mistrust, autonomy versus shame and doubt, initiation versus guilt, and
industry versus inferiority. Most children begin to understand gender diIIerences around the age oI 2
and begin to develop their own gender identity. Although a person`s sex is deIined by the physical
characteristic oI being male or Iemale, a person`s gender is the behavior associated with being male or
Iemale. One`s gender identity is inIluenced by cultural expectations as well as biology.
Adolescence is the period oI time Irom around age 14 to the early 20s and is most clearly identiIied
by the physical changes that occur in puberty. Mentally, many adolescents are moving into Piaget`s
Iormal operational stage oI development, which includes the ability to think in the abstract and to
consider hypothetical situations. At the same time, adolescents still demonstrate a considerable amount oI
egocentric thinking as can be seen in the thought processes oI the personal fable, in which adolescents
Ieel they are diIIerent Irom all others, and the imaginary audience, where the adolescent is convinced
that everyone is looking at him or her.
Lawrence Kohlberg proposed a theory about the development oI moral thinking and divided the
development into three levels, preconventional, during which the individual conIorms to social norms;
conventional, during which time the consequences determine morality; and postconventional, during
which a person`s individual moral principles determine right and wrong. The social crisis proposed by
Erikson Ior the period oI adolescence is that oI identity versus role confusion.
Adulthood can be roughly identiIied as the time period Irom the early 20s until death. Middle age is
oIten associated with an increase in health problems and includes the events oI menopause Ior women
and andropause Ior men. Cognitive abilities do not decline overall but the speed oI processing does
appear to slow down and people tend to have greater diIIiculty retrieving speciIic inIormation Irom their
memory. Erikson proposed three psychosocial developmental stages Ior adulthood. The stages are
intimacy versus isolation, generativity versus stagnation, and ego integrity versus despair. The term
ego integritv reIers to a sense oI wholeness that comes Irom having lived a Iull liIe and possessing the
ability to let go oI regrets. Parenting is a signiIicant part oI many people`s adulthood. Diana Baumrind
proposed three basic parenting styles, authoritarian, permissive, and authoritative. Permissive parents
can either be permissive neglectful or permissive indulgent.
A number oI theories explain why our bodies physically age. The cellular clock theory suggests that
cells are limited in the number oI times they can reproduce. The wear-and-tear theory proposes that aging
is a result oI outside stressors such as physical exertion and bodily damage. The Iree radical theory states
that as people get older, more and more Iree radicals accumulate in their bodies. Socially, the activity
theory suggests that elderly people adjust more positively to aging when they remain active in some way.
Elizabeth Kbler-Ross proposed a well-known theory oI the dying process. Based on her work with
dying patients, Kbler-Ross Ielt that people experienced a series oI Iive diIIerent emotions including
denial, anger, bargaining, depression, and acceptance. Others see dying as more oI a process rather than a
series oI stages. In addition, diIIerent cultures may vary in the way that they deal with death. For instance,
Development CHAPTER 8 -101-
within the Hindu culture in India, Iamily members may have extensive contact with the deceased. In
contrast, the Navajo tribal culture dictates that Native Americans should severely limit their contact with
the dead and dying.

STUDY HINTS
15.

Perhaps the most inIluential theory on cognitive development is Jean Piaget`s theory. He
proposed Iour stages oI cognitive development. To enhance your learning oI these stages, Iill in
the Iollowing chart. Try to Iill it in as much as possible without going back to your notes and/or
the textbook. The Iirst stage has been Iilled in as an example.

Stage Age Characteristics
How would you test to see if
someone is in this stage?
Sensorimotor 02
vears
Children explore using
their sensorv and
motor svstems.
Develop obfect
permanence.
Hide a tov under a blanket and see
if the child looks under the blanket
for the tov.





Development CHAPTER 8 -102-

2.
Another major theory oI development is the theory proposed by Erik Erikson. Erikson`s theory
Iocuses more on the development oI personality, with each stage marked by a crisis that needs to
be resolved. The crisis typically involves the social interactions oI the individual and is
represented by the two extremes oI the possible outcomes (Ior example, industry versus
inIeriority). One way to keep track oI these stages is to realize that the labels Iollow a pattern oI
'desirable outcome versus undesirable outcome. Also, they reIlect the social activities that are
typically going on at that age. Fill in the Iollowing chart to help you understand Erikson`s theory
oI development.

Age Social Activities Desirable Outcome Undesirable Outcome
01 vears being fed, taken care of bv
someone else
sense of trust sense of mistrust








Development CHAPTER 8 -103-
Suggested answers for Studv Hint 1

Stage Age Characteristics
How would you test to see
if someone is in this stage?
Sensorimotor 02
years
Explore using sensory and
motor systems.
Develop object
permanence.
Hide a toy under a blanket
and see iI the child looks
under the blanket Ior the
toy.
Preoperational 27
years
A lot oI egocentric thinking.
Represent objects mentally.
Engage in make-believe play.
Do not understand concepts
oI conservation.
Tend to Iocus on one aspect
oI an object.
Ask the child iI she would
rather have two quarters or
Iive pennies (she will
probably want the Iive
pennies).
See iI the child can play a
make-believe game.
Concrete
Operational
712
years
Show an understanding Ior
the principles oI
conservation.
Demonstrate logical thinking
and can solve analogies.
Focus mostly on concrete
objects and ideas.
Divide a string oI clay into
Iive pieces and see iI the
child thinks there is as much
clay in the Iive pieces as
there was in the one string.
Formal
Operational
12
years
and on
Can use abstract reasoning to
solve problems.
Able to consider hypothetical
situations.
Ask the child an abstract
question and see how he
responds. An example oI a
question could be 'What iI
snow was black?
Development CHAPTER 8 -104-

Suggested answers for Studv Hint 2

Age Social Activities Desirable Outcome Undesirable Outcome
01 years being Ied, taken care oI by
someone else
sense oI trust sense oI mistrust
13 years learning to walk, talk, dress
yourselI, etc.
sense oI autonomy
(Ieeling that you are in
control oI your own
body)
sense oI shame and
doubt
35 years going to preschool, being
responsible to obey Iamily
rules, learning your role as a
member oI a Iamily
sense oI initiative sense oI guilt or
irresponsibility
512
years
going to school, completing
school assignments,
participating in social
activities with peers
sense oI industry
(Ieeling capable oI
completing your work)
sense oI inIeriority or
incompetence
1218
years
deciding 'what you want to
be when you grow up,
choosing a career path,
selecting your own group oI
Iriends
sense oI identity Ieeling oI role
conIusion, unsure oI
who you are
1840
years
Iinding a partner to Iorm a
liIe-long commitment,
succeeding in a career
sense oI intimacy, Ieel
comIortable Iorming
close relationships
sense oI isolation, not
able to Iorm close ties
with others
4060
years
Iocusing on career and Iamily
(perhaps grandchildren begin
to enter the picture), begin
thinking oI the legacy that
you will leave Ior your
children and/or the Iuture
generation
sense oI generativity,
or succeeding in
creating something
that will beneIit others
in the Iuture
sense oI stagnation, or
Ieeling that you have
done nothing Ior the
next generation
60 years
and on
dealing with retirement Irom
your career, Iamily might be
more involved in their own
lives, Iacing the Iact oI death
among those in your social
group
sense oI ego integrity,
or a sense oI
acceptance oI your liIe
and acceptance oI
death
sense oI despair about
your liIe and a Iear oI
your inevitable death

Development CHAPTER 8 -105-
LEARNING OBJECTIVES
5.38 What are some of the special research
methods used to studv development?
5.39 What is the relationship between hereditv
and environmental factors in determining
development?
5.40 How do chromosomes, genes, and DNA
determine a persons characteristics or
disorders, and what causes multiple births?
5.41 What happens during the germinal,
embrvonic, and fetal periods of pregnancv,
and what are some ha:ards in prenatal
development?
5.42 What kind of phvsical changes take place in
infancv and childhood?
5.43 What are two wavs of looking at cognitive
development, and how does language help?


5.44 How do infants and children develop
personalities and form relationships with
others, and what are Eriksons stages of
psvchosocial development for children?
5.45 What are the phvsical, cognitive, and
personalitv changes that occur in
adolescence, including concepts of moralitv
and Eriksons search for identitv?
5.46 What are the phvsical, cognitive, and
personalitv changes that occur during
adulthood and aging, including Eriksons
last three psvchosocial stages and patterns
of parenting?
5.47 How do psvchologists explain whv aging
occurs, and what are the stages of death and
dving?
5.48 What are some cross-cultural differences in
views of death and dving?


PRACTICE EXAM
For the Iollowing multiple choice questions, select the answer you Ieel best answers the question.

1. A researcher who selects a sample oI people oI varying ages and studies them at one point in time is,
by deIinition, using the method.
a) cohort design
b) longitudinal design
c) behavior genetics design
d) cross-sectional design

2. Which oI the Iollowing is an example oI a longitudinal study?
a) observing three groups oI children (ages 2, 6, and 12) Ior a two-hour period
b) observing three groups oI children (ages 2, 6, and 12) Ior a two-week period
c) observing a group oI 30 children at age 2 and again at age 6, and once more when the
children turn 12 years oI age
d) surveying a group oI middle-aged adults, halI male and halI Iemale

3. What relatively new Iield investigates the inIluence oI genes and heredity on behavior?
a) psychobiology
b) neuropsychology
c) behavioral genetics
d) psychoanalysis

4. When a researcher discusses the contributions oI 'nature on development, she is reIerring to the
eIIects oI
a) environment.
b) heredity.
c) social interactions.
d) teratogens.

Development CHAPTER 8 -106-

5. Which oI the Iollowing is a special molecule that contains the genetic material oI the organism?
a) DNA
b) gene
c) chromosomes
d) amines

6. Which oI the Iollowing is essentially a 'recipe or set oI instructions Ior making a protein?
a) DNA
b) a chromosome
c) a gene
d) an enzyme

7. Dizygotic twins are Iormed Irom one egg and two sperm.
a) True
b) False

8. Monozygotic twins
a) are genetically identical.
b) are genetically diIIerent.
c) will be oI a diIIerent sex.
d) are more likely to occur when a woman is taking Iertility drugs.

9. Brittany and Abby Hensel are a type oI twin reIerred to as
a) nonidentical.
b) dizygotic.
c) Iraternal.
d) conjoined.

10. The specialized organ that provides nourishment and Iilters away waste products Irom the
developing baby is called the
a) placenta.
b) uterus.
c) umbilical cord.
d) embryo.

11. What are some oI the common consequences to a child iI the mother smoked while pregnant?
a) increased birth weight and lethargy
b) lower birth weight and short stature
c) severe hearing loss and heart deIects
d) severely deIormed limbs and muscle spasms

12. The longest prenatal period during which tremendous growth occurs and the organs continue to
develop and become Iunctional is called
a) germinal.
b) embryonic.
c) Ietal.
d) gestational.

Development CHAPTER 8 -107-

13. In the reIlex, the baby moves its head toward any light touch to its Iace.
a) Sucking
b) Startle
c) rooting
d) grasping

14. Which sense is the most well developed at birth?
a) taste
b) touch
c) sight
d) hearing

15. Your little sister picks up objects, Ieels every part oI them, and then puts them in her mouth. What
stage oI Jean Piaget's model oI cognitive development does this behavior suggest?
a) concrete operations
b) sensorimotor
c) preoperational
d) Iormal operations

16. A theory that looks at the way people deal with the inIormation that comes in through the senses is
called
a) inIormation-processing theory.
b) sensorimotor intelligence.
c) habituation.
d) metamemory.

17. Which oI the Iollowing would a child in Piaget`s preoperational stage oI cognitive development be
unable to do?
a) mentally represent an object
b) play make-believe
c) see the world Irom someone else`s perspective
d) use symbolic thought

18. Vygotsky`s idea that children develop cognitively when someone else helps them by asking leading
questions and providing examples is called
a) scaIIolding.
b) centration.
c) conservation.
d) metamemory.

19. The Iirst noticeable signs oI language development in inIants is
a) babbling.
b) cooing.
c) telegraphic speech.
d) holophrases.

20. By about 12 months, most inIants
a) begin to use intonation in their language.
b) build a vocabulary oI one-word phrases, or holophrases.
c) begin to distinguish, in their language, between themselves and others.
d) begin to Iorm two- and three-word sentences.
Development CHAPTER 8 -108-

21. InIants in diIIerent cultures and oI diIIerent languages experience a diIIerent series oI stages in
language development.
a) True
b) False

22. II an inIant in Mary Ainsworth`s Strange Situation was unwilling to explore, became very upset by
the stranger, and demanded to be picked up by his mother but then kicked to get away, he would
most likely be classiIied as
a) secure.
b) avoidant.
c) ambivalent.
d) disorganizeddisoriented.

23. Chester is irritable, loud, and negative most oI the time. He really doesn't like when new
people pick him up and hold him, and he has irregular sleeping, eating, and waking schedules. What
temperament does he exhibit?
a) active
b) slow-to-warm-up
c) diIIicult
d) easy

24. Erikson`s theory oI social development viewed the ages oI 3 to 6, his third stage, as being
characterized by the major challenge oI
a) identity versus role diIIusion.
b) industry versus inIeriority.
c) initiative versus guilt.
d) autonomy versus shame and doubt.

25. According to Erikson, when children between the ages oI 5 and 12 succeed at learning new skills,
they develop a sense oI ; and iI they Iail to develop new abilities, they Ieel
.
a) shame; doubt
b) trust; guilt
c) industry; inIerior
d) identity; despair

26. The growth spurt Ior boys typically begins at age
a) 9.
b) 12.
c) 10.
d) 15.

27. FiIteen-year-old Todd is writing an impassioned novel about growing up in America. In his novel he
describes his experiences in a way that portrays him as unique and special, such that no one has ever
thought such deep thoughts or experienced such ecstasy beIore. Todd`s writings most clearly reIlect
a) his sense oI autonomy.
b) the personal Iable.
c) the period oI rebellion common to all adolescents.
d) his developing sense oI conscience.

Development CHAPTER 8 -109-

28. Which oI the Iollowing questions would an adolescent who has not reached Piaget`s stage oI Iormal
operations have trouble thinking about?
a) What date did Columbus arrive in America?
b) How many 2-inch pieces can a 10-inch rope be cut into?
c) What iI you had been born to diIIerent parents?
d) What is the deIinition oI democracy?

29. Jeremy is 17 years old. According to Erikson, his chieI task will be acquiring a sense oI
a) identity.
b) intimacy.
c) generativity.
d) autonomy.

30. The cessation oI menstruation and ovulation is called
a) climacteric.
b) perimenopause.
c) menopause.
d) andropause.

31. Which oI the Iollowing reasons does not explain why middle adults experience changes in memory?
a) stress
b) more inIormation to remember
c) more inIormation stored in memory
d) hardening oI the arteries

32. A young adult who is having diIIiculty trusting others is most likely still trying to resolve Erikson`s
stage oI
a) autonomy versus shame and doubt.
b) ego integrity versus despair.
c) industry versus inIeriority.
d) intimacy versus isolation.

33. Which oI the Iollowing is an example oI generativity?
a) completing a crossword puzzle
b) becoming a mentor
c) getting married
d) Iinding your own identity

34. The theory oI aging suggests that unstable oxygen molecules cause damage to the structure
oI cells, increasing with age.
a) wear-and-tear
b) cellular clock
c) disengagement
d) Iree radical

Development CHAPTER 8 -110-

35. Several weeks oI diagnostic tests revealed the cancer has spread throughout Barry's body. His
physician suggested that he "take care oI important matters." Barry realizes his Iamily's home needs
repairs, so he arranges to have that done right away. To relieve his Iamily oI the agony oI planning
his Iuneral, he has made all the arrangements. Barry told his minister he has a good liIe and just
wants to make sure he provides Ior his Iamily aIter his death. This description Iits the stage Kbler-
Ross called
a) denial.
b) acceptance.
c) bargaining.
d) depression.

PRACTICE EXAM ANSWERS
1. d Cross-sectional design studies several diIIerent age groups at the same time.
2. b A longitudinal study involves the study oI a group oI individuals at two or more
time points in their lives. It may be helpIul to remember that a longitudinal study
takes a long time to complete.
3. c Genetics is a Iield that investigates the eIIects oI genes and environmental
inIluences on behavior, whereas psychobiology is the study oI the biological bases
oI behavior.
4. b Nature reIers to everything a person inherits biologically. Nurture reIers to the
eIIects oI a person`s surroundings, or environment.
5. a DNA, genes, and chromosomes all contain the genetic material oI an organism, but
DNA is the only molecule listed.
6. c A gene is a section oI DNA that contains instructions Ior making proteins.
Chromosomes are large strands oI DNA that contain many genes.
7. b 'Di means two, and zygotic is reIerring to the zygote, which is Iormed when the
egg and sperm unite. Dizygotic (or Iraternal) twins are Iormed Irom two eggs and
two sperm.
8. a Monozygotic twins are Iormed Irom one egg and one sperm ('mono means one).
AIter the egg and sperm unite they split to Iorm two zygotes. Because all the DNA
comes Irom the same egg and sperm, monozygotic twins are genetically identical.
9. d Conjoined twins (commonly reIerred to as Siamese twins) are physically joined due
to the Iact that the zygotes do not completely separate Irom each other.
10. a The placenta provides protection and nourishment to the Ietus.
11. b Multiple studies have Iound that babies oI mothers who smoked are smaller in
weight and height than babies Irom mothers who didn`t smoke.
12. c The Ietal period is the longest and last stage oI prenatal development and is when
the most growth occurs in the Ietus.
13. c The rooting reIlex is thought to help the baby with breast-Ieeding.
14. b Touch and taste are Iairly well developed at birth, with touch being the most highly
developed. The sense oI sight takes the longest to Iully develop aIter birth.
15. b The sensorimotor stage involves exploring the world through the use oI the sensory
and motor systems. During the sensorimotor stage inIants interact deliberately with
objects by chewing, grasping, and tasting them It is the Iirst oI Piaget`s Iour stages
oI cognitive development.
16. a InIormation-processing theory looks at the way in which people deal with the
inIormation that comes in through the senses. Metamemory is one`s knowledge
about the workings oI memory and memory strategies.
17. c In the preoperational stage oI development, children are still very egocentric and
have a hard time seeing the world Irom someone else`s viewpoint.

Development CHAPTER 8 -111-
18. a ScaIIolding is the process oI helping a child develop by providing the Iramework
Ior learning.
19. b Cooing is the Iirst visible sign oI language development in inIants, Iollowed by
babbling, holophrases, and then telegraphic speech.
20. b By about 1 year oI age, children are communicating one-word 'phrases.
Telegraphic speech, which consists oI two- or three-word sentences, usually
develops around 1 to 2 years oI age.
21. b It appears that all inIants experience the same stages oI language development.
22. c The ambivalent child exhibits ambivalent behaviors towards his or her mother. An
example is begging to be picked up by the mother and then struggling to get away
Irom the mother once he or she is picked up.
23. c DiIIicult babies tend to be irritable, are not comIortable with change, and have
irregular schedules.
24. c Initiative versus guilt is Erikson`s third stage oI development. Autonomy versus
shame and doubt is the second stage, and industry versus inIeriority is the Iourth
stage.
25. c Industry versus inIeriority is Erikson`s Iourth stage oI development and corresponds
closely with the primary school years.
26. b Boys show a growth spurt around age 12, whereas girls typically show a growth
spurt earlier, around age 10.
27. b The personal Iable describes a phenomenon commonly seen in adolescents in which
they Ieel that no one else has experienced the emotions or thoughts that they are
currently experiencing.
28. c The key to Piaget`s Iourth and Iinal stage oI cognitive development is the ability to
consider hypothetical and abstract situations.
29. a Erikson believed most oI the adolescent years involved the crisis oI identity versus
role conIusion.
30. c Menopause is the correct answer. Perimenopause is the term used to describe the
period oI 510 years during which a woman`s reproductive system begins to
decline.
31. d Memory changes during middle age have not been Iound to be associated with
physical decline.
32. d Erikson proposed the stage oI intimacy versus isolation Ior young adults as they try
to Iorm intimate relations with others and learn to trust in someone other than
themselves.
33. b Generativity involves helping a younger generation and engaging in activities that
will leave a legacy.
34. d Radicals are oxygen molecules in the cells that are thought to cause damage.
35. b Kbler-Ross described the stage oI acceptance as an emotional state oI
acknowledging one`s impending death and being at peace with the idea.

CHAPTER GLOSSARY
activity theory theory oI adjustment to aging that assumes older people are happier iI
they stay active in some way, such a volunteering or starting a hobby.
adolescence the period oI liIe Irom about age 13 to the early 20s, during which a
young person is no longer physically a child but is not yet an
independent, selI-supporting adult.
andropause gradual changes in the sexual hormones and reproductive system oI
middle-aged males.
attachment the emotional bond between an inIant and the primary caregiver.
authoritarian parenting style oI parenting in which a parent is rigid and overly strict, showing
little warmth to the child.
Development CHAPTER 8 -112-
authoritative parenting style oI parenting in which parents combine warmth and aIIection with
Iirm limits on a child`s behavior.
centration in Piaget`s theory, the tendency oI a young child to Iocus on only one
Ieature oI an object while ignoring other relevant Ieatures.
chromosome tightly wound strand oI genetic material or DNA.
cognitive development the development oI thinking, problem solving, and memory.
concrete operational stage Piaget`s third stage oI cognitive development in which the school-age
child becomes capable oI logical thought processes but is not yet
capable oI abstract thinking.
conservation in Piaget`s theory, the ability to understand that simply changing the
appearance oI an object does not change the object`s nature.
conventional morality second level oI Kohlberg`s stages oI moral development in which the
child`s behavior is governed by conIorming to society`s norms oI
behavior.
critical periods times during which some environmental inIluences can have an impact
on the development oI the inIant.
cross-sectional design research design in which several diIIerent participant age-groups are
studied at one particular point in time.
cross-sequential design research design in which participants are studied by means oI a cross-
sectional design but are also Iollowed and assessed longitudinally.
dizygotic twins oIten called Iraternal twins, occurs when two eggs each are Iertilized by
separate sperm, resulting in two zygotes in the uterus at the same time.
DNA (deoxyribonucleic acid) special molecule that contains the genetic material oI the organism.
dominant reIerring to a gene that actively controls the expression oI a trait.
ego integrity sense oI wholeness that comes Irom having lived a Iull liIe and possessing
the ability to let go oI regrets; the Iinal completion oI the ego.
egocentrism the inability to see the world through anyone else`s eyes.
embryo developing organism Irom 2 weeks to 8 weeks aIter Iertilization.
embryonic period the period Irom 2 to 8 weeks aIter Iertilization, during which the major
organs and structures oI the organism develop.
fertilization the union oI the ovum and sperm.
fetal period the time Irom about 8 weeks aIter conception until the birth oI the baby.
fetus developing organism Irom 8 weeks aIter Iertilization to the birth oI the
baby.
formal operational stage Piaget`s last stage oI cognitive development in which the adolescent
becomes capable oI abstract thinking.
gender the behavior associated with being male or Iemale.
gender identity perception oI one`s gender and the behavior associated with that
gender.
gene section oI DNA having the same arrangement oI chemical elements.
generativity providing guidance to one`s children or the next generation, or
contributing to the well-being oI the next generation through career or
volunteer work.
genetics the science oI inherited traits.
germinal period Iirst 2 weeks aIter Iertilization, during which the zygote moves down to
the uterus and begins to implant in the lining.
human development the scientiIic study oI the changes that occur in people as they age,
Irom conception until death.
identity versus role confusion stage oI personality development in which the adolescent must Iind a
consistent sense oI selI.

Development CHAPTER 8 -113-
Development CHAPTER 8 -114-
imaginary audience type oI thought common to adolescents in which young people believe
that other people are just as concerned about the adolescent`s thoughts
and characteristics as they themselves are.
intimacy an emotional and psychological closeness that is based on the ability to
trust, share, and care, while still maintaining a sense oI selI.
irreversibility in Piaget`s theory, the inability oI the young child to mentally reverse
an action.
longitudinal design research design in which one participant or group oI participants is
studied over a long period oI time.
menopause the cessation oI ovulation and menstrual cycles and the end oI a
woman`s reproductive capability.
monozygotic twins identical twins Iormed when one zygote splits into two separate masses
oI cells, each oI which develops into a separate embryo.
nature the inIluence oI our inherited characteristics on our personality,
physical growth, intellectual growth, and social interactions.
nurture the inIluence oI the environment on personality, physical growth,
intellectual growth, and social interactions.
object permanence the knowledge that an object exists even when it is not in sight.
ovum the Iemale sex cell, or egg.
permissive parenting style oI parenting in which the parents make Iew, iI any demands on a
child`s behavior.
permissive indulgent permissive parenting style in which parents are so involved that
children are allowed to behave without set limits.
permissive neglectful permissive parenting style in which the parents are uninvolved with the
child or child`s behavior.
personal fable type oI thought common to adolescents in which young people believe
themselves to be unique and protected Irom harm.
postconventional morality third level oI Kohlberg`s stages oI moral development in which the
person`s behavior is governed by moral principles that have been
decided on by the individual and which may be in disagreement with
accepted social norms.
preconventional morality Iirst level oI Kohlberg`s stages oI moral development in which the
child`s behavior is governed by the consequences oI the behavior.
preoperational stage Piaget`s second stage oI cognitive development in which the preschool
child learns to use language as a means oI exploring the world.
puberty the physical changes that occur in the body as sexual development
reaches its peak.
recessive reIerring to a gene that only inIluences the expression oI a trait when
paired with an identical gene.
scaffolding process in which a more skilled learner helps a less skilled learner,
reducing the amount oI help as the less skilled learner becomes more
capable.
scheme a mental concept Iormed through experiences with objects and events.
sensorimotor stage Piaget`s Iirst stage oI cognitive development in which the inIant uses its
senses and motor abilities to interact with objects in the environment.
temperament the behavioral characteristics that are Iairly well established at birth,
such as 'easy, 'diIIicult, and 'slow to warm up.
teratogen any Iactor that can cause a birth deIect.
zone of proximal
development (ZPD)
Vygotsky`s concept oI the diIIerence between what a child can do alone
and what that child can do with the help oI a teacher.
zygote cell resulting Irom the uniting oI the ovum and sperm.
CHAPTER 9 - MOTIVATION AND EMOTION


YOU KNOW YOU ARE READY FOR THE TEST IF YOU ARE ABLE TO.
Introduce the concept oI motivation and discuss the major theories proposed to explain motivated
behavior.
Discuss the speciIic motivation oI hunger and examine the physiological and social components
in addition to common eating disorders.
Describe the three elements oI emotion and present six theories on how emotions are processed.

RAPID REVIEW
Motivation is the process by which activities are started, directed, and continued so that physical or
psychological needs or wants are met. When motivation comes Irom outside the selI it is called extrinsic
motivation. On the other hand, iI a person does something because it is satisIying in some internal
manner, the motivation is called intrinsic motivation. Several theories have been proposed to explain the
process oI motivation, including the theories oI instinct, drive-reduction, needs, arousal, incentive,
humanistic, and selI-determination. Instincts are biologically determined innate patterns oI behavior that
exist in both people and animals; thus, instinct approach theory suggests that people are motivated by
biologically determined internal Iorces. UnIortunately, instinct theory only describes behavior and is
unable to explain why people do what they do. The drive-reduction approach proposes that a need, or
requirement, produces a drive, or psychological tension and physiological arousal, and that people act in
order to reduce these drives. The drives can be primary drives such as hunger and thirst or acquired
(secondary) drives such as the need Ior money. The rationale Ior drive-reduction includes the idea that
the body has a tendency to try to maintain a steady state reIerred to as homeostasis. When the body is out
oI balance, a need develops and the tension provides the drive to reduce the need and return the body to a
state oI balance. Drive-reduction theory, however, cannot explain why people would increase their
internal tension by doing things like parachuting out oI an airplane. Need theory attempts to explain
motivation by understanding three speciIic needs, the need for achievement (nAch), need for affiliation
(nAff), and need for power (nPow). Arousal theory suggests that people are motivated to maintain an
optimal level oI arousal or tension. The level oI arousal is achieved by increasing or decreasing
stimulation and is driven by a proposed stimulus motive. The Yerkes-Dodson law demonstrates that Ior
an easy task, perIormance is best when arousal is a little higher than average, whereas Ior a diIIicult task,
perIormance is best when arousal is a little below average. Individuals who consistently seek out high
levels oI arousal have been labeled as sensation seekers. According to incentive approaches oI
motivation, individuals` actions are determined by the rewards or incentives Ior their behaviors.
Expectancy-value theories are a subset oI incentive theories and assume that the expectancies or belieIs
oI an individual need to be taken into account in order to understand his or her motivation.
Abraham Maslow was a major proponent oI the humanistic approach to motivation and proposed a
hierarchy oI needs that individuals must IulIill beIore they can reach the highest need oI self-
actualization, where a person reaches his or her Iullest potential. According to Maslow, basic needs such
as hunger and thirst must be satisIied beIore higher-level needs can be achieved. Also, Maslow reIerred to
the times in which selI-actualization is attained as peak experiences. Another theory oI motivation
similar to Maslow`s is the self-determination theory, which proposes humans work to satisIy three
inborn and universal needs. These needs are the need Ior autonomy, competence, and relatedness.
One speciIic area oI motivation that has been studied extensively is the motivation to eat, also known
as hunger. The hunger drive can be divided into physiological and social components. Physiologically,
insulin and glucagons are hormones that regulate the level oI glucose in the bloodstream. Insulin
increases blood sugar levels, which leads to Ieelings oI hunger. Several areas in the hypothalamus also
play an important role in regulating eating behavior, perhaps by inIluencing the speciIic weight that our
bodies try to maintain, known as the weight set point. Another Iactor that inIluences the weight set point
is the basal metabolic rate, which decreases as we age, causing a corresponding increase in the weight
set point. The social Iactors that inIluence hunger include the times oI day when a person typically eats,
Motivation and Emotion CHAPTER 9 -115-
using Iood to reduce stress or provide comIort, and the appeal oI a tempting dessert item. One problem
associated with eating behaviors is obesity, in which a person weighs 20 percent over his or her ideal
weight. Studies have shown that heredity plays a major role in the development oI obesity; however, low
exercise rates and overeating are also major contributors to weight gain. Recent research also suggests
that a hormone called leptin may play a role in controlling how hungry an individual Ieels. Anorexia
nervosa and bulimia are two eating disorders associated with weight loss, rather than gain.
Emotions can be deIined as the 'Ieeling aspect oI consciousness, characterized by a certain physical
arousal, a certain behavior, and an inner awareness oI Ieelings. As can be seen in the deIinition, emotions
can be divided into three components: physiology, behaviors, and subjective experience. DiIIerent
emotions have been Iound to be associated with diIIerent physiological reactions. The amygdala has been
Iound to play a role in the regulation oI emotions in humans as well as other animals. Much oI what we
know about the amygdala`s role in emotion comes Irom the work oI Dr. Joseph LeDoux and colleagues.
For instance, recent research suggests that emotional stimuli travel to the amygdala by both a Iast, direct
(subcortical) route and a slow, indirect (preIrontal cortex) route. The direct route allows Ior quick
responses to stimuli that are possibly dangerous, whereas, the awareness provided by the indirect route
can override the direct route to take control oI our emotional responses. In this way, higher brain areas
such as the preIrontal cortex can take control oI our implicit responses to emotional stimuli. Research also
suggests that, in individuals with emotional disorders, the indirect route may be unable to eIIectively
control the more automatic activity oI the direct route. In this situation, a person may exhibit poor
regulation oI their emotions. Three common strategies Ior regulating one`s emotions include distraction,
reappraisal, and controlling the inIluence oI emotions on decision making. Research suggests that
distraction involves activation oI the anterior cingulated cortex, and reappraisal is supported by activity in
the lateral orbitoIrontal cortex. Both distraction and reappraisal are accompanied by lower activity in the
amygdala.
The behaviors oI emotions include Iacial expressions, body movements, and other actions. Research
has supported the idea that at least seven basic Iacial expressions are recognized and mimicked in cultures
around the world. However, the display rules, or exactly when, where, and how these emotions can be
expressed, appears to diIIer across cultures. The subjective experience oI emotions involves the cognitive
process oI assigning a label, such as happy, to your Ieelings. Several theories have been developed in an
attempt to explain the process humans use to label our emotions. Common sense theory suggests that a
stimulus causes a particular emotion to occur, which then leads to the behavioral and physiological
response. The 1ames-Lange theory oI emotion proposes that a stimulus leads to a particular
physiological response, which then leads to the subjective experience oI an emotion. The Cannon-Bard
theory suggests that the physical and subjective experience oI emotions occur at the same time. A
stimulus leads to activation oI the thalamus, which then simultaneously activates the sympathetic nervous
system and higher cortical areas that interpret the signal as a particular emotion. Schachter and Singer
proposed the cognitive arousal theory in which a stimulus occurs and then our body has a physical
reaction and we make a cognitive appraisal oI the situation. Based on Ieedback Irom both these sources,
we then come up with a subjective label Ior the emotion we are experiencing. The facial feedback
hypothesis assumes that Iacial expressions provide Ieedback to the brain regarding the emotion being
expressed and can then intensiIy or even cause the emotion. Lazarus`s cognitive-mediational theory oI
emotion suggests that Iollowing a stimulus, we engage in a cognitive appraisal oI the situation that
triggers a subjective experience oI an emotion Iollowed by a physiological response.
Author David Allen has written extensively on the importance oI time- or task-management systems
that may help people to keep track oI commitments and to accomplish speciIic tasks and general goals.
Using such systems may help to reduce stress and decrease negative emotion. Allen`s hints Ior getting
things done include (1) physically collect all items that require your attention in one place, such as a
Iolder; (2) process and deIine what you can take action on and identiIy concrete steps toward those goals;
(3) organize inIormation and reminders into categories or contexts based on how and when you need
them; (4) complete weekly reviews oI your projects, next actions, and new items; and (5) do all oI your
next actions in the appropriate context or time Irame Ior doing so.

Motivation and Emotion CHAPTER 9 -116-
STUDY HINTS
16.

By Iar, the most conIusing concept oI this chapter is keeping track oI the theories oI emotion.
The Iollowing hints are designed to help you work through this process. To start with, try Iilling
in the Iollowing table correctly. Remember when we are discussing emotions, we are interested
in several components. The theories vary according to which component comes Iirst. The
components are
physiological experience oI emotion (increased heart rate, sweating, etc.)
subjective experience oI emotion (the 'Ieeling oI happiness, sadness, or anger)
cognitive appraisal (using your thought process to assess the situation)
subcortical brain activity (not considered cognitive types oI action)

Using these key components, Iill in the Iollowing table. The Iirst row has already been Iilled in
Ior you.

Theory Event 1
st
response 2
nd
response 3
rd
response
Common Sense
stimulus
(dog barking)
subfective
experience
(fear)
phvsiological
experience
(increased heart
rate)

James-Lange
Theory

Cannon-Bard
Theory

Cognitive
Arousal Theory

Facial Feedback
Hypothesis

Cognitive-
mediational
Theory


Motivation and Emotion CHAPTER 9 -117-


17.


Now look over the chart you just completed. Which oI the theories are similar and which are
diIIerent? Can you come up with a way to group the theories together based on similarity?

As you learned in the chapter on memory, processing the inIormation in this manner will help
you better retain the material and make retrieval Ior the exam an easier process. Try grouping the
theories into the Iollowing three categories:

Category 1: Physiological experience occurs aIter you 'Ieel the emotion








Category 2: 'Feeling the emotion occurs aIter the physiological changes











Category 3: 'Feeling the emotion and the physiological changes occur at the same time





Motivation and Emotion CHAPTER 9 -118-

Suggested answers for Question 1

Theory Event 1
st
response 2
nd
response 3
rd
response
Common Sense
stimulus
(dog barking)
subfective
experience
(fear)
phvsiological
experience
(increased heart
rate)

James-Lange
Theory
stimulus
(dog barking)
phvsiological
(increased heart
rate)
subfective
(fear)

Cannon-Bard
Theory
stimulus
(dog barking)
subcortical
brain activitv
phvsiological
and subfective at
the same time

Cognitive
Arousal Theory
stimulus
(dog barking)
phvsiological
response
(increased heart
rate)
cognitive
appraisal
(there is a scarv-
looking dog
barking at me)
subfective
experience
(fear)
Facial Feedback
Hypothesis
stimulus
(dog barking)
facial
expression of
fear
subcortical
brain activitv
subfective
experience
(fear)
Cognitive-
mediational
Theory
stimulus
(dog barking)
cognitive
appraisal
(there is a scarv-
looking dog
barking at me)
subfective
experience
(fear)
phvsiological
experience
(increased heart
rate)

Motivation and Emotion CHAPTER 9 -119-


Suggested answers for Question 2

Category 1: Physiological experience occurs aIter you 'Ieel the emotion

common sense theorv


cognitive-mediational theorv

Category 2: 'Feeling the emotion occurs aIter the physiological changes

James-Lange theorv


cognitive arousal theorv


facial feedback hvpothesis

Category 3: 'Feeling the emotion and the physiological changes occur at the same time

Cannon-Bard theorv


LEARNING OBJECTIVES
5.49 How do psvchologists define motivation, and
what are the kev elements of the earlv instinct
and drive-reduction approaches to motivation?
5.50 What are the characteristics of the three
tvpes of needs?
5.51 What are the kev elements of the arousal
and incentive approaches to motivation?
5.52 How do Maslows humanistic approach and
self-determination theorv explain
motivation?
"#"$ What happens in the bodv to cause hunger,
and how do social factors influence a
persons experience of hunger?
5.54 What are some problems in eating behavior,
and how are thev affected bv biologv and
culture?
5.55 What are the three elements of emotion?
5.56 How do the James-Lange and Cannon-Bard
theories of emotion differ?
5.57 What are the kev elements in cognitive
arousal theorv, the facial-feedback
hvpothesis, and the cognitive-mediational
theorv of emotion?


PRACTICE EXAM
For the Iollowing multiple choice questions, select the answer you Ieel best answers the question.

1. The process by which activities are started, directed, and continued so that physical or psychological
needs or wants are met is called
a) motivation.
b) emotion.
c) achievement.
d) synergy.

Motivation and Emotion CHAPTER 9 -120-

2. Which statement about motivation is true?
a) A motive energizes and directs behavior.
b) We are always aware oI motivational processes.
c) DiIIerent motives always lead to diIIerent behaviors.
d) Two people motivated by the same Iactor will satisIy that motive through similar
means.

3. In the early twentieth century, psychologists were inclined to explain motivated behavior by
attributing it to
a) emotions.
b) incentives.
c) learned responses.
d) instincts.

4. Each oI the Iollowing is a valid criticism oI instinct theories oI motivation EXCEPT
a) human behavior is rarely rigid, inIlexible, and Iound throughout the species.
b) instinct theories name behaviors without pinpointing their origins.
c) they were the dominant explanation Ior human behavior early in the twentieth century.
d) description is less important than explanation.

5. Salmon swimming upstream to spawn are an example oI
a) incentives.
b) motives.
c) instinct.
d) needs.

6. Drives serve to activate responses that are aimed at reducing the drive, thereby returning the
body to a more normal state called
a) stability.
b) equilibrium.
c) homeostasis.
d) physiological balance.

7. Some psychologists believe that behavior is motivated by the body's attempts to achieve a state oI
balance in which the body Iunctions eIIectively, or in
a) reciprocity.
b) acquiescence.
c) propinquity.
d) homeostasis.

8. Primary drives are
a) exceptions to the drive-reduction principle.
b) learned.
c) inIluenced by stimuli within the body.
d) inIluenced by stimuli outside the body.

Motivation and Emotion CHAPTER 9 -121-

9. Monica put all her time and energy into getting into the acting club because her main goal in liIe
'was to be a Iamous star! Monica`s drive to be Iamous was a drive.
a) primary
b) reIlexive
c) tertiary
d) secondary

10. Which oI the Iollowing is correct Ior people high in need achievement?
a) They look Ior careers that make a lot oI money.
b) They look Ior careers and hobbies that allow others to evaluate them.
c) They look Ior careers that require little education.
d) They look Ior careers that will make them Iamous.

11. According to Carol Dweek, need achievement is closely related to
a) genetics.
b) geography.
c) luck.
d) personal Iactors.

12. In arousal theory, people are said to have a(n) level oI tension.
a) ultimate
b) lower
c) optimal
d) high

13. Indiana Jones goes oII to Ioreign lands in search oI artiIacts hidden in dangerous places and guarded
by Iierce protectors. Dr. Jones would be described as in arousal theory.
a) a sensation seeker
b) nAII
c) Iool-hardy
d) high nPow

14. As a class assignment you are required to collect advertising slogans and describe how they
may be relevant to concepts in psychology. You select the Jell-O slogan, "There's always room Ior
Jell-O," and describe in class that it is relevant to one oI the theories oI motivation. Which theory?
a) instinctive
b) incentive
c) drive-reduction
d) optimum-level

15. One interesting thing about incentive approaches is that incentives
a) exist inside a narrow collection oI internal stimuli.
b) exist independently oI any need or level oI arousal.
c) exist inside a narrow collection oI internal stimuli.
d) only work Ior adults.

Motivation and Emotion CHAPTER 9 -122-

16. Jill is motivated by money and the things money will bring her. Jack is motivated by doing good
things, and his incentives are based on that idea and belieI. What theory incorporates both these
types oI motivational causes?
a) sensation-seeking theory
b) entity theory
c) increment theory
d) expectancy-value theory

17. According to Abraham Maslow, developing one's potential to its Iullest extent results in
a) saIety.
b) selI-esteem.
c) belongingness.
d) selI-actualization.

18. Which oI the Iollowing Iails to show the motivating power oI selI-actualization?
a) Joan wants to live in a house with all the modern conveniences so that she may have
more time to seek IulIillment Irom her career and Iamily.
b) Frank Ieels that he is a good salesman because he likes what he does and knows how to
do it well.
c) Barbara knows that, as a teacher, she is a good person because she realizes the
importance oI imparting knowledge to society.
d) Mark works hard as an attorney only so that he can attract more clients, more money, and
be secure in the knowledge that his Iamily can survive.

19. SelI-determination theory (SDT) best Iits which type oI motivation?
a) the need Ior aIIiliation
b) intrinsic motivation
c) extrinsic motivation
d) a mastery goal

20. Intrinsic motivation is deIined as
a) the pursuit oI an activity Ior external rewards.
b) the pursuit oI an activity Ior its own sake.
c) the pursuit oI an activity to relieve the state oI tension caused by deprivation.
d) the pursuit oI an activity in order to be judged Iavorably by others.

21. One Iactor in hunger seems to be the increase in that occurs aIter we begin to eat.
a) cholesterol
b) lipoproteins
c) insulin
d) glucose

22. The ventromedial hypothalamus (VMH) may be involved in
a) increasing hunger.
b) decreasing hunger.
c) processing low Iats.
d) Iood allergies.

Motivation and Emotion CHAPTER 9 -123-

23. The lateral hypothalamus (LH) may be involved in
a) stoppage oI eating.
b) the onset oI eating.
c) processing low Iats.
d) Iood allergies.

24. Anna weighed about 125 pounds most oI her adult liIe. However, it seemed like whenever Anna
gained weight it was easy to lose and get back to 125. But when she wanted to go below 125 it took
Iorever and even the slightest deviation Irom her diet got her back to 125. What explanation would
you give Anna?
a) Use better diet products.
b) Start a reality TV show.
c) Her weight, 125, is her set point. Leave it alone.
d) Her BMR is causing all the problems.

25. The concept oI 'comIort Iood suggests that Iood
a) may be inIluenced by social Iactors.
b) has genetic ways to comIort.
c) may release hormones and neurotransmitters that are comIorting.
d) is reIlexive.

26. Which component oI hunger is most likely contributing to the Iact that sometimes a person who has
just had a late breakIast will still Ieel hungry at noon?
a) social
b) behavioral
c) physiological
d) intrinsic

27. Which individual has the highest risk Ior developing anorexia nervosa?
a) lower-class 26-year-old European man
b) an upper-class 16-year-old American boy
c) a lower-class 26-year-old European woman
d) an upper-class 16-year-old American girl

28. An eating disorder characterized by binges oI eating Iollowed by selI-induced vomiting is called
a) anorexia nervosa
b) bulimia
c) Karposi`s anemia
d) Huntington`s chorea

29. All oI the Iollowing statements are correct about bulimia EXCEPT
a) individuals with bulimia have a distorted view oI how much Iood is too much Iood.
b) bulimia is not as damaging as anorexia nervosa.
c) binge eating and vomiting are common symptoms.
d) individuals with bulimia have a distorted body image.

Motivation and Emotion CHAPTER 9 -124-

30. Which oI the Iollowing describes a hormone that is secreted into the bloodstream by Iatty tissue and
signals to the hypothalamus that the body has enough Iood, reducing appetite and increasing the
Ieeling oI being Iull?
a) adrenalin
b) peptic acid
c) leptin
d) Lippotor

31. What Latin word connects both motive and emotion?
a) emote
b) move
c) mote
d) mate

32. Paul Ekman and his colleagues gathered abundant evidence supporting the universality oI
basic Iacial expressions oI emotion.
a) three
b) Iive
c) seven
d) nine

33. According to Ekman, which oI the Iollowing is not one oI the universal Iacial expressions?
a) disgust
b) Iear
c) contempt
d) shame

34. To explain the human universality and variability oI emotions, Ekman and his associates
a) developed a concept oI 'display rules, which are rules Ior emotional expression.
b) developed an interobserver system to make sure that observers deIined expressions
reliably.
c) interviewed all participants in order to assess unexpressed Ieelings and motivations.
d) monitored the brain waves oI participants to determine which hemisphere had higher
activation.

35. Which one oI the Iollowing is not one oI the three elements oI emotion discussed in the text?
a) physiology
b) labeling
c) behavior
d) environment

36. Which theory states that a stimulus triggers physiological changes that produce emotion?
a) Cannon-Bard theory
b) James-Lange theory
c) cognitive arousal theory
d) commonsense view oI emotions

Motivation and Emotion CHAPTER 9 -125-

37. Which statement is most consistent with the James-Lange theory oI emotion?
a) 'I run because I'm aIraid.
b) 'I'm laughing because I am happy.
c) 'I'm crying because I'm sad.
d) 'I'm anxious because I perspire.

38. What is the correct sequence oI events in emotional response according to the Cannon-Bard theory?
a) stimulus emotion physiological changes
b) stimulus physiological changes emotion
c) physiological changes stimulus emotion
d) stimulus emotion AND physiological changes (simultaneous)

39. 'I think I`m aIraid, thereIore I am aIraid is a statement that is most consistent with which oI the
Iollowing theories?
a) the James-Lange theory
b) activation theory
c) cognitive arousal theory
d) the Cannon-Bard theory

40. According to the theory oI emotion proposed by Schachter and Singer, what is the most important
determinant oI your subjective experience oI emotion?
a) physiological reactions
b) cognitive appraisal oI the situation
c) Iacial expressions
d) intensity oI the stimulus

41. In the classic study oI emotion conducted by Schachter and Singer, aIter receiving the epinephrine,
the subjects placed in the room with the angry man reported Ieeling
a) angry.
b) happy.
c) both angry and happy.
d) no emotions.

42. According to the Iacial Ieedback hypothesis, iI you would like to make yourselI Ieel more happy
you should
a) spend time with Iriends.
b) talk to a counselor.
c) think about all the positive aspects oI your liIe.
d) smile.

43. According to the cognitive-mediational theory, which Iactor would be most important in
determining whether you Ieel nervous when asked to speak in Iront oI the class?
a) your physiological reaction to the request
b) activation oI subcortical brain activity
c) your cognitive appraisal oI the situation
d) your change in blood pressure

Motivation and Emotion CHAPTER 9 -126-
PRACTICE EXAM ANSWERS
1. a Motivation encompasses these three activities.
2. a This is simply the deIinition oI motive.
3. d Instinct theory was one oI the Iirst proposed theories oI motivation in psychology.
Be careIul not to conIuse incentive with instincts!
4. c This is not a criticism but simply a statement.
5. c Instincts are innate, biologically determined behaviors.
6. c Homeostasis is the term psychologists and physiologists use to reIer to the body`s
state oI balance. Equilibrium means the same thing but is not the term used by
psychologists.
7. d Homeostasis is a sense oI balance.
8. c These internal stimuli would include things such as hunger or thirst.
9. d Secondary drives are drives that we acquire through learning. The drive to be
Iamous is learned.
10. b High achievers need Ieedback Irom others.
11. d There was no mention oI genetics in her theory, but there was considerable
discussion about an individual`s sense oI selI and views on intelligence.
12. c Arousal theory argues that arousal should be neither too high nor too low.
13. a His actions indicate that he needs a higher level oI arousal than most people.
14. b Incentive theory suggests that we oIten eat Iood items because oI their reward value
and not simply because we are hungry.
15. b Incentives motivate behavior regardless oI a speciIic need or arousal level.
16. d Expectancy value theory states that the values oI a person determine his or her
motivation levels.
17. d SelI-actualization is at the top oI Maslow`s hierarchy oI needs.
18. d This is the only situation in which the person is Iocusing on basic needs Ior himselI
and his Iamily. In all the other examples, the individuals were Iocused on growth
needs.
19. b SelI-determination theory is characterized by intrinsic motivations.
20. b All the other motivations are based on external Iactors. Intrinsic motivation deals
with Iorces within the individual.
21. c Insulin is associated with Ieelings oI hunger and is related to blood sugar levels.
22. b When the ventromedial hypothalamus was removed in experimental animals, the
animals no longer controlled their eating and became extremely obese.
23. b When the lateral hypothalamus was removed in experimental animals, the animals
stopped eating and had to be Iorce-Ied Iood.
24. c The set point is the level oI weight the body tends to maintain.
25. a Factors in hunger include the social cues associated with Iood.
26. a Social Iactors can have a strong impact on Ieelings oI hunger.
27. d According to government statistics, white upper-class Iemales in the United States
show the highest prevalence rates Ior anorexia.
28. b This is the deIinition given in the textbook. Note that the vomiting distinguishes
bulimia Irom anorexia.
29. b Although the damage Irom bulimia is diIIerent than that Irom anorexia, it is still a
very dangerous disorder.
30. c Leptin appears to be the hormone that causes you to Ieel Iull.
31. b This is the deIinition given in your textbook.
32. c The research Iound seven Iacial expressions that appear to be universal.
33. d Shame was not Iound in all cultures. The seven Iacial expressions he did Iind were
anger, Iear, disgust, happiness, surprise, sadness, and contempt.
34. a Ekman Iound that display rules tend to vary across cultures, whereas the recognition
oI basic emotions tends to be universal.
Motivation and Emotion CHAPTER 9 -127-
35. d Emotion was broken down into its physical, behavioral, and subjective (or labeling)
components.
36. b The James-Lange theory states that the changes in our body come Iirst, Iollowed by
our subjective experience oI an emotion.
37. d The physiological change comes beIore the experience oI the emotion.
38. d Cannon-Bard believed the subjective and physiological experience occurred
simultaneously.
39. c The cognitive aspect (or thinking component) is the Iactor that determines your
emotions according to the cognitive arousal theory.
40. b Schachter and Singer`s theory suggests that one`s appraisal oI a situation determines
one`s subjective emotional response to a stimulus.
41. a In accordance with Schachter and Singer`s theory, the participants use the
environmental cues oI an angry co-subject to determine that their own arousal was
due to anger as well.
42. d The Iacial Ieedback hypothesis proposes that our brain receives Ieedback on our
Iacial expressions that then serves to enhance whatever emotion we are expressing.
43. c The cognitive-mediational theory states that a stimulus must be interpreted, or
appraised, by a person in order to result in a physical response to an emotional
situation.

CHAPTER GLOSSARY
acquired (secondary)
drives
those drives that are learned through experience or conditioning, such as
the need Ior money or social approval.
amygdala brain structure located near the hippocampus, responsible Ior Iear
responses and memory oI Iear.
arousal theory theory oI motivation in which people are said to have an optimal (best or
ideal) level oI tension that they seek to maintain by increasing or
decreasing stimulation.
basal metabolic rate the rate at which the body burns energy when the organism is resting.
Cannon-Bard theory of
emotion
theory in which the physiological reaction and the emotion are assumed
to occur at the same time.
cognitive arousal theory theory oI emotion in which both the physical arousal and the labeling oI
that arousal based on cues Irom the environment must occur beIore the
emotion is experienced.
cognitive-mediational
theory
theory oI emotion in which a stimulus must be interpreted (appraised) by a person
in order to result in a physical response and an emotional reaction.
display rules learned ways oI controlling displays oI emotion in social settings.
drive a psychological tension and physical arousal arising when there is a need that
motivates the organism to act in order to IulIill the need and reduce the tension.
drive-reduction theory approach to motivation that assumes behavior arises Irom physiological
needs that cause internal drives to push the organism to satisIy the need
and reduce tension and arousal.
emotion the 'Ieeling aspect oI consciousness, characterized by a certain physical
arousal, a certain behavior that reveals the emotion to the outside world,
and an inner awareness oI Ieelings.
expectancy-value theories incentive theories that assume the actions oI humans cannot be predicted
or Iully understood without understanding the belieIs, values, and the
importance that a person attaches to those belieIs and values at any given
moment in time.
extrinsic motivation type oI motivation in which a person perIorms an action because it leads
to an outcome that is separate Irom or external to the person.
Motivation and Emotion CHAPTER 9 -128-
facial feedback hypothesis theory oI emotion which assumes that Iacial expressions provide
Ieedback to the brain concerning the emotion being expressed, which in
turn causes and intensiIies the emotion.
glucagon hormone that is secreted by the pancreas to control the levels oI Iats, proteins, and
carbohydrates in the body by increasing the level oI glucose in the bloodstream.
homeostasis the tendency oI the body to maintain a steady state.
incentive approaches theories oI motivation in which behavior is explained as a response to the
external stimulus and its rewarding properties.
incentives things that attract or lure people into action.
instinct approach approach to motivation that assumes people are governed by instincts
similar to those oI animals.
instincts the biologically determined and innate patterns oI behavior that exist in
both people and animals.
insulin a hormone secreted by the pancreas to control the levels oI Iats, proteins, and
carbohydrates in the body by reducing the level oI glucose in the bloodstream.
intrinsic motivation type oI motivation in which a person perIorms an action because the act
itselI is rewarding or satisIying in some internal manner.
1ames-Lange theory of
emotion
theory in which a physiological reaction leads to the labeling oI an
emotion.
leptin a hormone that, when released into the bloodstream, signals the
hypothalamus that the body has had enough Iood and reduces the appetite
while increasing the Ieeling oI being Iull.
motivation the process by which activities are started, directed, and continued so that
physical or psychological needs or wants are met.
need a requirement oI some material (such as Iood or water) that is essential
Ior survival oI the organism.
need for achievement
(nAch)
a need which involves a strong desire to succeed in attaining goals, not
only realistic ones but also challenging ones.
need for affiliation (nAff) the need Ior Iriendly social interactions and relationships with others.
need for power (nPow) the need to have control or inIluence over others.
peak experiences according to Maslow, times in a person`s liIe during which selI-
actualization is temporarily achieved.
primary drives those drives that involve needs oI the body such as hunger and thirst.
self-actualization according to Maslow, the point that is seldom reached at which people have
suIIiciently satisIied the lower needs and achieved their Iull human potential.
self-determination theory
(SDT)
theory oI human motivation in which the social context oI an action has
an eIIect on the type oI motivation existing Ior the action.
sensation seeker someone who needs more arousal than the average person.
stimulus motive a motive that appears to be unlearned but causes an increase in
stimulation, such as curiosity.
weight set point the particular level oI weight that the body tries to maintain.
Yerkes-Dodson law law stating perIormance is related to arousal; moderate levels oI arousal
lead to better perIormance than do levels oI arousal that are too low or too
high. This eIIect varies with the diIIiculty oI the task: Easy tasks require a
high-moderate level whereas more diIIicult tasks require a low-moderate
level.

Motivation and Emotion CHAPTER 9 -129-
Motivation and Emotion CHAPTER 9 -130-
CHAPTER 10 - SEXUALITY AND GENDER


YOU KNOW YOU ARE READY FOR THE TEST IF YOU ARE ABLE TO.
Discuss physical sex diIIerences between males and Iemales.
Describe the psychological gender diIIerences between males and Iemales including gender
development and gender stereotypes.
Introduce three oI the most inIluential studies on sexual behavior in the United States.
Explain the concept oI sexual orientation.
Describe the physical and psychological problems that can lead to sexual dysIunction.
Discuss the most common sexually transmitted inIections in the United States.

RAPID REVIEW
Sex is deIined as the physical diIIerences between males and Iemales. Primary sex
characteristics are those physical characteristics that are present at birth and are directly involved in
human reproduction. In the Iemale, the primary sex characteristics include the vagina, uterus, and
ovaries. In the male the primary sex characteristics include the penis, the testes (also called the testicles),
the scrotum, and the prostate gland. In the Iemale embryo, the development oI the gonads into ovaries
causes the release oI estrogens, which leads to the development oI the remaining sex organs, while in the
male, the development oI the gonads into testes leads to the release oI androgens and Iurther
development oI the male sex organs. Secondary sex characteristics develop during puberty and are
indirectly involved in human reproduction. Female secondary sex characteristics include a growth spurt
aIter the Iirst menstrual cycle, enlarged breasts, maturation oI mammary glands, wider hips, pubic hair,
and Iat deposits on the buttocks and thighs. Male secondary sex characteristics include a deepening voice,
Iacial chest and pubic hair, development oI coarser skin texture, and a growth spurt.
Approximately 1 out oI 1,500 children in the United States are born with ambiguous sexual
organs, a condition previously reIerred to as hermaphroditism, but now more commonly called
intersexed or intersexual. Many physicians view the condition as an abnormality that should be repaired
by sexual reassignment surgery. However, many intersexed individuals Ieel that the decision regarding
surgery should be made by the individual themselves when they are old enough to make their own choice.
Gender is deIined as the psychological aspects oI being Ieminine or masculine. Gender roles are
a culture`s expectation Ior behavior oI a person who is perceived as male or Iemale, and gender typing is
the process by which individuals learn their expected gender role. A person`s sense oI being Iemale or
male is called their gender identity and is inIluenced by both biology and environment. For example,
some researchers believe that exposure to certain hormones during Ietal development inIluences gender
identity in addition to the strong environmental pressures oI Iamily and Iriends to behave in the
'expected manner. Social learning theory proposes that individuals learn their gender identities by
observing the behaviors oI the people around them and being rewarded Ior imitating the appropriate
gender behavior. Gender schema theory suggests that children acquire their gender role by organizing
their own behavior around their internalized schema oI 'boy or 'girl. A stereotype is a concept held
about a person or group oI people that is based on superIicial, irrelevant characteristics. A gender
stereotype is a generalization about males or Iemales that ignores individual diIIerences. Female gender
stereotypes oIten include characteristics such as illogical, emotional, sensitive, nurturing, while male
gender stereotypes can include characteristics such as aggressive, logical, decisive, and unemotional.
Sexism reIers to prejudice about males and Iemales. Benevolent sexism reIers to the acceptance oI
positive stereotypes about males and Iemales that leads to unIair treatment. Psychologist Sandra Bem
coined the term %&.$!25&5 to describe people who display both male and Iemale characteristics. With
regard to cognitive diIIerences between gender, men tend to perIorm better than women on certain spatial
tasks, while women tend to perIorm better than men on tests oI perceptual speed. Researchers are still
investigating the relative contributions oI the environment and heredity on these gender diIIerences.
However, more recent research by Else-Quest and colleagues in 2010 indicated that the supposed
Sexuality and Gender CHAPTER 10 -131-
diIIerences in math abilities between boys and girls is likely due to girls` lack oI conIidence rather than any
biological diIIerence in the working oI the brain. With regard to communication, women tend to use a
'relate style oI communication while men oIten use a 'report style.
Three landmark studies have provided much oI the inIormation available today in the United
States about human sexuality. In 1957, William Masters and Virginia Johnson conducted the Iirst direct
observational study on the physical aspects oI the human sexual response by recording the physiological
reactions oI 700 Iemale and male volunteers while they were engaged in sexual intercourse or
masturbation. Their research led them to propose Iour stages oI the sexual response cycle: excitement,
plateau, orgasm, and resolution, respectively. It is during the orgasm stage that men release semen, the
sperm-containing Iluid that is ejaculated Irom the penis. Men show a refractory period aIter the Iourth
phase during which time they cannot achieve erection. The valuable research oI Masters and Johnson has
helped a tremendous number oI individuals but was extremely controversial when it was originally
published. In 1948, AlIred Kinsey published his Iindings Irom a large survey oI adult sexual behavior in
the United States. His Iindings were based on Iace-to-Iace interviews with participants and included
details about the Irequency oI behaviors such as masturbation, anal sex, premarital sex, and sexual
orientation. Some have criticized the Kinsey study on the basis oI methodological issues. The next large-
scale study oI human sexual behavior was published in 1993 by Samuel Janus and Cynthia Janus. The
Janus Report described sexual behaviors based on the survey responses oI 3,000 individuals Irom across
the United States. In addition to topics examined previously, The Janus Report also looked at sexual
deviance among other new topics. A recent study by Sanders and colleagues in 2010 suggests that surveys
oI sexual behavior can be problematic because the deIinition oI 'having sex appears to vary considerably
across individuals.
Sexual orientation reIers to a person`s sexual attraction Ior members oI a particular sex. The
term )#'#$!4#67%1 reIers to people who are sexually attracted to members oI the opposite physical sex,
and the term )!-!4#67%1 reIers to individuals who are attracted to members oI their own physical sex. A
recent national survey by Mosher and colleagues in 2005 indicated that about 2.3 percent oI men and 1.3
percent oI women aged 15 to 44 years consider themselves to be exclusively or predominantly
homosexual. A person who is bisexual may be either male or Iemale and is attracted to both sexes.
Mosher`s national survey indicated that 1.8 percent oI men and 2.8 percent oI women consider
themselves to be bisexual.
Many research studies have examined the biology oI homosexuality. One oI the earliest studies,
conducted by Ellis and colleagues in the 1980s, suggested that male children oI mothers who experienced
severe stress during the second trimester oI pregnancy had a signiIicantly higher chance oI being
homosexual in orientation. A more recent study, conducted by Savic and colleagues in 2005, Iound that
homosexual men and heterosexual women respond similarly, and quite diIIerently Irom heterosexual
men, to a testosterone-based pheromone (glandular chemical) that is secreted in perspiration.
A sexual dysfunction is a problem with sexual Iunctioning or the actual physical workings oI the
sex act and can be caused by a number oI Iactors. Organic or stress-induced dysfunctions are the sexual
problems that are caused by physical disorders or by psychological distress. The sexual problems can be
in three areas oI sexual activity: sexual interest, arousal, and response. Paraphilia is a sexual dysIunction
in which the person achieves sexual arousal and IulIillment through sexual behavior that is unusual or
socially unacceptable.
A sexually transmitted infection, or STI, is an inIection spread primarily through sexual contact.
Some common STIs in the United States include chlamydia, syphilis, gonorrhea, genital herpes, genital
warts, and AIDS (or acquired immune deficiency syndrome). AIDS is caused by the human
immunodeIiciency virus (HIV), which wears down the body`s immune system, making the individual
highly susceptible to inIections. The virus can be transmitted Irom person to person through exposure to
blood, vaginal Iluid, semen, and breast milk. There are no documented cases oI the spread oI AIDS
through tears or saliva. The Centers Ior Disease Control estimates that about 1.1 million adults and
adolescents are currently living with HIV in the United States; nearly halI a million individuals have
developed AIDS. Individuals can protect themselves Irom STIs by using condoms, having a sexual
relationship with one uninIected partner, not sharing needles or other drug equipment, having regular
Sexuality and Gender CHAPTER 10 -132-
exams Ior STIs, learning the common symptoms oI STIs, talking openly with your partner about
inIections and condom use, and realizing that abstinence is the only 100 percent eIIective prevention.

STUDY HINTS
18.

This chapter introduced three oI the most important studies on human sexuality conducted to
date. Use the Iollowing table to help you summarize the details oI these studies

Researcher(s) Date of Study Method Used Major Findings




Sexuality and Gender CHAPTER 10 -133-


19.


Try matching each oI the STIs listed on the leIt with the correct set oI symptoms on the right.

chlamydia
initial symptoms oI a painless open sore that usually
appears on the penis or around or in the vagina; iI
untreated, may go on to more advanced stages,
including a transient rash and, eventually, serious
involvement oI the heart and central nervous system

syphilis
inIection caused by the herpes simplex virus, which
causes a painIul rash oI Iluid-Iilled blisters on the
genitals

gonorrhea
aIIects the genitals oI both sexes, causes burning or
diIIiculty with urination, itching, and a yellow or green
discharge; easily treated with antibiotics

genital herpes
lesions produced by the human papillomavirus (HPV)
and transmitted through sexual contact; the lesions may
be raised and bumpy, or Ilat and almost impossible to
see

genital warts
causes damage to the Iemale and male reproductive
systems resulting in inIertility; may remain undetected
Ior long periods oI time

acquired immune deIiciency
syndrome (AIDS)
viral disorder that causes deterioration oI the immune
system and eventually results in death due to
complicating inIections that the body can no longer
Iight

Sexuality and Gender CHAPTER 10 -134-


Suggested Answers for Question 1

Researcher(s) Date of Study Method Used Major Findings
Masters and
Johnson
late 1950s,
published in
1957
direct
observation in a
laboratory
Iour phases oI human sexual
response. Men and women both
go through the Iour phases but
men have a reIractory period that
is not typically seen in women.
AlIred Kinsey 1948 one-on-one
personal
interviews
sexual orientation was seen more
along a continuum. Frequency oI
masturbation, premarital, and
extramarital sex was much higher
than many people previously
thought.
Samuel Janus and
Cynthia Janus
1993 one-on-one
interviews as
well as mass
questionnaires
Iindings on typical sexual
behavior as well as sexual
deviance, single people`s sexual
behavior, marriage, divorce, and
decisions to have children
Answers for Question 2

chlamydia initial symptoms oI a painless open sore that usually
appears on the penis or around or in the vagina; iI
untreated, may go on to more advanced stages,
including a transient rash and, eventually, serious
involvement oI the heart and central nervous system

syphilis inIection caused by the herpes simplex virus, which
causes a painIul rash oI Iluid-Iilled blisters on the
genitals

gonorrhea
aIIects the genitals oI both sexes, causes burning or
diIIiculty with urination, itching, and a yellow or green
discharge; easily treated with antibiotics

genital herpes lesions produced by the human papillomavirus (HPV)
and transmitted through sexual contact; the lesions may
be raised and bumpy, or Ilat and almost impossible to
see

genital warts
causes damage to the Iemale and male reproductive
systems resulting in inIertility; may remain undetected
Ior long periods oI time

acquired immune deIiciency
syndrome (AIDS)

viral disorder that causes deterioration oI the immune
system and eventually results in death due to
complicating inIections that the body can no longer
Iight

Sexuality and Gender CHAPTER 10 -135-
LEARNING OBJECTIVES
10.1 What are the phvsical differences between
females and males?
10.2 What is gender, and how can biologv and
learning influence gender role
development?
10.3 How do gender roles develop, and how can
thev be influenced bv stereotvpes or an
emphasis on androgvnv?
10.4 How do men and women differ in thinking,
social behavior, and personalitv?
10.5 What happens in the bodies of women and
men during sexual intercourse?

10.6 What did the earlv and most recent survevs
of human sexual behavior reveal?
10.7 How do different sexual orientations
develop?
10.8 How do phvsical and psvchological sexual
problems differ?
10.9 What are sexuallv transmitted infections, and
what can be done to prevent the spread of
these disorders?

PRACTICE EXAM
For the Iollowing multiple choice questions, select the answer you Ieel best answers the question.

1. The growth spurt in Iemale puberty usually starts at years oI age.
a) 810
b) 1012
c) 1214
d) 1416

2. Which oI the Iollowing is a primary sex characteristic in males?
a) Iacial and chest hair
b) deepening voice
c) development oI coarser skin texture
d) the prostate gland

3. ________ is the process by which people learn their culture's preIerences and expectations Ior
proper "masculine" and "Ieminine" behavior.
a) Gender role
b) Gender identity
c) Gender typing
d) Gender stereotyping

4. Whereas can be deIined as the physical characteristics oI being Iemale or male, is
deIined as the psychological aspects oI being Ieminine or masculine.
a) sex; gender
b) gender roles; gender identity
c) gender typing; gender
d) gender; sex

5. Traditional views oI gender roles are more likely Iound in
a) collectivist cultures.
b) individualistic cultures.
c) countries like the Netherlands, Germany, Italy, and England.
d) cultures where men have less traditional views.

Sexuality and Gender CHAPTER 10 -136-

6. II an individual`s gender identity was completely determined by the DNA he or she inherited, we
would say that gender identity is determined by
a) nature.
b) nurture.
c) both nature and nurture.
d) the individual`s temperament.

7. When children observe their same-sex parents behaving in certain ways and imitate that behavior, a
psychologist uses which theory to explain the situation?
a) Freudian psychoanalysis
b) Piaget's theory oI development
c) gender schema theory
d) social learning theory

8. A child who develops her identity and organizes her behavior around a mental concept oI 'girl is
relying on
a) simple imitation.
b) positive reinIorcement.
c) social pressures.
d) a gender schema.

9. Desperate Ior help with her computer, Dana calls her Iiance, thinking that he will know what to do
because he is a man, and men are natural Iixers. Dana's thinking in this instance is an example oI
a) androgyny.
b) schema error.
c) benevolent sexism.
d) negative stereotyping.

10. Stereotypes about males and Iemales consist oI only negative characteristics.
a) True
b) False

11. Psychologist Sandra Bem (1975, 1981) developed the concept oI
a) androgyny.
b) benevolent sexism.
c) social learning theory.
d) ambiguity.

12. MRI studies have suggested that men listen with
a) the right hemisphere oI the brain.
b) the leIt hemisphere oI the brain.
c) both hemispheres oI the brain.
d) no consistent pattern.

13. One diIIerence that has been reported in the communication styles oI men and women is that
a) no diIIerences have been Iound in communication styles.
b) men talk more than women.
c) men are more likely to switch topics Irequently.
d) women are more likely to interrupt.

Sexuality and Gender CHAPTER 10 -137-

14. Approximately how many children in the United States are born with both male and Iemale sex
organs?
a) 1 out oI 100
b) 1 out oI 1,500
c) 1 out oI 100,000
d) There are currently no estimates oI the number.

15. The Iinal phase oI the sexual response is
a) excitement.
b) plateau.
c) orgasm.
d) resolution.

16. The research oI Masters and Johnson represents the Iirst major contribution to our understanding oI
a) common sexual behaviors.
b) prevalence oI sexually transmitted inIections.
c) common sexual disorders.
d) the physical response during sexual activity.

17. One seemingly amazing aspect oI Masters and Johnson's research program concerning human
sexual response was
a) their ability to get senior citizens to volunteer as subjects in the Iirst studies.
b) that the study was Iunded by churches open to discovering ways to prevent masturbation.
c) that they were able to convince the newspapers to keep the research secret Ior a long
period oI time.
d) their use oI seismographic-like machines to measure the sexual response oI volunteers.

18. The Kinsey Report, which was published in 1948 by AlIred Kinsey and reported on common sexual
behaviors oI adults in United States, was based on what type oI research method?
a) direct observation
b) anonymous surveys
c) experimental laboratory studies
d) one-on-one personal interviews

19. According to Kinsey, what percentage oI husbands reported anal sex with their wives?
a) 11 percent
b) 14 percent
c) 92 percent
d) 26 percent

20. The Iirst large-scale study oI human behavior to be done aIter the Kinsey and Masters and Johnson
reports was conducted by
a) Janus and Janus.
b) Hite and Rose.
c) Hilton and Paris.
d) Erickson and Schlomo.

Sexuality and Gender CHAPTER 10 -138-

21. According to the Janus report, percent oI men and percent oI women reported
masturbating?
a) 80, 70
b) 80, 15
c) 70, 80
d) 15, 80

22. When a person reIers to himselI or herselI as 'heterosexual or 'homosexual, the person is
reIerring to his or her
a) sexual identity.
b) sex.
c) gender identity.
d) sexual orientation.

23. According to a recent study by Savic and colleagues, both homosexual men and respond
similarly to a testosterone-based pheromone (glandular chemical).
a) heterosexual women
b) heterosexual men
c) homosexual women
d) None oI the above. The pheromone-related response oI homosexual men is unique.

24. Jaime enjoys sexual activity with his partner. However, he cannot reach an orgasm during sexual
intercourse even though Iully aroused. Jaime is suIIering Irom
a) male erectile disorder.
b) male orgasmic disorder.
c) dyspareunia.
d) premature ejaculation.

25. Jennie experiences persistent contractions oI her vaginal muscles. These contractions cause
intercourse to be painIul and sometimes even impossible Ior Jennie. She likely has a condition
known as
a) vaginismus.
b) Irotteurism.
c) Iemale orgasmic disorder.
d) sexual diversion disorder.

26. is a disorder in which an individual achieves sexual arousal and IulIillment
through sexual behavior that is unusual or not socially acceptable.
a) Schizophrenia
b) Borderline personality disorder
c) Gender identity disorder
d) Paraphilia

27. Which cause oI a sexually transmitted inIection is hardest to treat?
a) bacterial
b) Iungal
c) viral
d) All are equally diIIicult to treat.

Sexuality and Gender CHAPTER 10 -139-

28. AIDS can be passed Irom one individual to the next through each oI the Iollowing ways EXCEPT
a) vaginal Iluid.
b) semen.
c) tears.
d) blood.

29. AIDS is caused by
a) a bacterial inIection.
b) an air-borne Iungus.
c) a viral inIection.
d) The cause is not yet known.

30. Which cause oI a sexually transmitted disease is hardest to treat?
a) bacterial
b) Iungal
c) viral
d) all are equally diIIicult to treat


PRACTICE EXAM ANSWERS
1. b The Iemale growth spurt starts about age 1012, while the male growth spurt starts
about age 1214.
2. d The prostate gland is a primary sex characteristic in males.
3. c Gender typing is the process oI learning proper masculine and Ieminine behaviors.
Gender roles are the actual expectations each culture has Ior males and Iemales, and
gender identity is the individuals` sense oI being male or Iemale.
4. a Sex reIers to physical diIIerences, and gender reIers to psychological and social
diIIerences.
5. a Several research studies have supported the idea that collectivist cultures, such as
those Iound in many Asian and South American countries, display more traditional
views on gender roles.
6. a Nature reIers to inherited, biological diIIerences, while nurture reIers to the eIIects
oI the environment.
7. d Social learning theory emphasizes observational learning, which is reinIorced
through attention and positive remarks.
8. d A gender schema is a mental concept oI what it means to be a 'boy or a 'girl.
9. c Benevolent sexism is the result oI thinking that all men or all women have some
particular desirable trait, simply because oI their sex.
10. b Stereotypes can be both negative and positive.
11. a Androgyny describes people who exhibit both male and Iemale typical behaviors.
12. b Several studies have shown that males listen primarily with the leIt hemisphere oI
the brain, while Iemales tend to show activity in both hemispheres while they are
engaged in listening activities.
13. c Men are more likely to use a 'report style oI communication which involves
switching topics Irequently, while women are more likely to use a 'relate style oI
communication.
14. b The best estimate to date is that about 1 in every 1,500 children in the United States
is born with ambiguous sexual genitalia.
15. d Masters and Johnson labeled the Iourth and Iinal phase oI the human sexual
response as the 'resolution phase.
16. d Masters and Johnson used direct observations in the laboratory to investigate the
Sexuality and Gender CHAPTER 10 -140-
physical human sexual response.
17. c Masters and Johnson were able to convince the newspapers to keep the research
secret Ior almost 12 years.
18. d AlIred Kinsey traveled across the country with a team oI researchers and conducted
one-on-one personal interviews to gather data on sexual behavior.
19. a According to Kinsey, 11 percent oI husbands reported anal sex with their wives.
20. a The Januses did the Iirst major study on common sexual behaviors aIter Kinsey.
21. a The Janus study revealed that 80 percent oI men and 70 percent oI women reported
masturbating.
22. d Sexual orientation reIers to a person`s attraction Ior members oI a particular sex.
23. a Savic`s recent study suggests that both homosexual men and heterosexual women
respond similarly to the presence oI a testosterone-based pheromone and that these
responses are quite diIIerent Irom those oI heterosexual males.
24. b Jaime does get Iully aroused, which means he does have erections and is thereIore
not suIIering Irom an erectile disorder. Instead he has a condition known as male
orgasmic disorder.
25. a Jennie has vaginismus.
26. d Paraphilias are a group oI disorders in which sexual arousal is achieved through
unusual or socially unacceptable methods.
27. c Bacterial inIections can normally be treated with antibiotics but viral diseases are
very hard to treat.
28. c Currently, there are no documented cases oI a person becoming inIected with AIDS
through exposure to tears.
29. c AIDS is caused by the human immunodeIiciency virus, more commonly reIerred to
as HIV.
30. c Viral inIections are the hardest to treat, and they are oIten incurable

CHAPTER GLOSSARY
AIDS, acquired immune
deficiency syndrome
sexually transmitted viral disorder that causes deterioration oI the
immune system and eventually results in death due to complicating
inIections that the body can no longer Iight.
androgens male hormones.
androgyny characteristic oI possessing the most positive personality characteristics
oI males and Iemales regardless oI actual sex.
benevolent sexism acceptance oI positive stereotypes oI males and Iemales that leads to
unequal treatment.
bisexual person attracted to both men and women.
estrogens Iemale hormones.
gender the psychological aspects oI being male or Iemale.
gender identity the individual`s sense oI being male or Iemale.
gender roles the culture`s expectations Ior masculine or Ieminine behavior, including
attitudes, actions, and personality traits associated with being male or
Iemale in that culture.
gender schema theory theory oI gender identity acquisition in which a child develops a mental
pattern, or schema, Ior being male or Iemale and then organizes
observed and learned behavior around that scheme.
gender stereotype a concept held about a person or group oI people that is based on being
male or Iemale.
gender typing the process oI acquiring gender role characteristics.
hermaphroditism the condition oI possessing both male and Iemale sexual organs.
heterosexual person attracted to the opposite sex.
Sexuality and Gender CHAPTER 10 -141-
Sexuality and Gender CHAPTER 10 -142-
homosexual person attracted to the same sex.
intersexed, intersexual modern term Ior a hermaphrodite, a person who possesses ambiguous
sexual organs, making it diIIicult to determine actual sex Irom a visual
inspection at birth.
mammary glands glands within the breast tissue that produce milk when a woman gives
birth to an inIant.
menstrual cycle monthly shedding oI the blood and tissue that line the uterus in
preparation Ior pregnancy when conception does not occur.
organic or stress-induced
dysfunctions
sexual problem caused by physical disorder or psychological stress.
orgasm a series oI rhythmic contractions oI the muscles oI the vaginal walls or
the penis, also the third and shortest phase oI sexual response.
ovaries the Iemale sexual glands.
paraphilia a sexual disorder in which the person`s preIerred method oI sexual
arousal and IulIillment is through sexual behavior that is unusual or
socially unacceptable.
penis the organ through which males urinate and which delivers the male sex
cells or sperm.
primary sex characteristics sexual organs present at birth and directly involved in human
reproduction.
prostate gland gland that secretes most oI the Iluid holding the male sex cells or sperm.
refractory period time period in males just aIter orgasm in which the male cannot become
aroused or achieve erection.
resolution the Iinal phase oI the sexual response in which the body is returned to a
normal state.
scrotum external sack that holds the testes.
secondary sex
characteristics
sexual organs and traits that develop at puberty and are indirectly
involved in human reproduction.
semen Iluid released Irom the penis at orgasm that contains sperm.
sexism prejudice about males and/or Iemales leading to unequal treatment.
sexual dysfunction a problem in sexual Iunctioning.
sexual orientation a person`s sexual attraction and aIIection Ior members oI either the
opposite or the same sex.
sexually transmitted
infection (STI)
an inIection spread primarily through sexual contact.
stereotype a concept held about a person or group oI people that is based on
superIicial, irrelevant characteristics.
testes (testicles) the male sex glands.
uterus the womb in which the baby grows during pregnancy.
vagina the tube that leads Irom the outside oI a Iemale`s body to the opening oI
the womb.

CHAPTER 11 - STRESS AND HEALTH


YOU KNOW YOU ARE READY FOR THE TEST IF YOU ARE ABLE TO.
DeIine stress and identiIy the external and psychological Iactors that inIluence an individual`s
experience oI stress.
Discuss the causes oI stress.
Describe the physical reaction to stress and the relation oI this reaction to cognitive, personality,
and social Iactors.
Explain the methods used to cope with stress including those inIluenced by culture and religion.
Understand the importance oI exercise in reducing the negative physical and psychological
eIIects oI stress.

RAPID REVIEW
Health psychology is a new area oI psychology Iocusing on how physical activities,
psychological traits, and social relationships aIIect overall health. Stress is the physical, emotional,
cognitive, and behavioral responses to events that are perceived as threatening or challenging. When a
person`s stress response is unpleasant or undesirable it is called distress, and when it is an optimal
amount that helps a person Iunction it is called eustress. The events that cause stress are called stressors
and can be either internal or external events. Stressors can include external events such as catastrophes,
major liIe changes, and hassles, along with internal experiences such as pressure, uncontrollability, and
Irustration. A catastrophe is an unpredictable event that happens on a large scale such as a tornado,
hurricane, or Ilood. A number oI researchers have suggested that any major liIe change, such as moving,
getting married, or getting a new job, would result in stress. Holmes and Rahe developed the Social
Readjustment Rating Scale (SRRS) to measure the amount oI change and thus stress in a person`s liIe.
Researchers have Iound a moderate correlation between scores on the SRRS and physical health.
Alternate Iorms oI the SRRS have been designed Ior speciIic populations such as the College
Undergraduate Stress Scale (CUSS) Ior college students. A majority oI the stressors that people have to
deal with are little daily annoyances, or hassles. Surveys that measure the number oI hassles an individual
has to deal with are actually a better predictor oI short-term illnesses than the SRRS. The internal
experience oI pressure is also considered a stressor. Pressure is the psychological experience produced
by demands and expectations Irom outside sources. Two additional internal causes oI stress are
uncontrollability, or a lack oI control in a situation, and frustration, or being blocked Irom achieving a
desired goal. Typical reactions to Irustration include persistence and aggression, or actions meant to harm
or destroy. Displaced aggression occurs when a person takes out his or her Irustrations on less
threatening, more available targets and is a Iorm oI displacement. Another possible reaction to Irustration
is escape or withdrawal. This approach can take the Iorm oI leaving, dropping out oI school, quitting a
job, or ending a relationship. Some people may escape psychologically by withdrawing into apathy,
Iantasy, or the use oI drugs. Others may resort to suicide.
ConIlict is another source oI stress and occurs when a person Ieels pulled toward two or more
goals but can only achieve one oI them. Approach-approach conflict occurs when an individual is
attempting to choose between two desirable goals. Avoidance-avoidance conflict occurs when someone
must choose between two undesirable goals. Approach-avoidance conflict describes a single goal that
has both desirable and undesirable outcomes. An individual Iaced with two options in which each option
has positive and negative aspects is dealing with a double approach-avoidance conflict. II there are
more than two options, the conIlict is called a multiple approach-avoidance conflict.
Psychologist Hans Selye was a pioneer in the study oI the physical consequences oI exposure to
stressors. He proposed that the body goes through a sequence oI three stages he called the general
adaptation syndrome. The initial stage is called alarm and represents the immediate reaction to stress
mediated by our sympathetic nervous system. Typical alarm reactions include increased heart rate and
blood pressure, and release oI sugar into the blood stream. As the stress continues, the body enters the
Stress and Health CHAPTER 11 -143-
resistance stage during which time the sympathetic nervous system works overtime to give the body more
energy. When the body`s resources have been exhausted, the parasympathetic nervous system is activated
and the body enters the exhaustion stage. Selye believed that it was the prolonged release oI stress
hormones during the resistance stage that led to the breakdown oI the body`s immune system and the
onset oI the stress-related physical conditions. Researchers in the Iield oI psychoneuroimmunology who
study the eIIects oI psychological Iactors on the immune system have Iound that stress actually causes an
increase in the activity oI the immune system. High levels oI stress have been associated with weight
problems and have been linked to increased risk oI heart disease and Type 2 diabetes, a type oI diabetes
that is oIten related to obesity and is usually diagnosed beIore the age oI 40. Also, stress has been shown
to decrease the amount oI natural killer (NK) cells, which are the cells responsible Ior Iighting
cancerous growths. Recent research suggests that hormones also play a role in helping the immune
system Iight the eIIects oI stress. In particular, a hormone called dehydroepiandrosterone (DHEA) aids
humans in stress tolerance, perhaps by regulating the eIIects oI stress on the hippocampus.
The cognitive-mediational theory oI emotions proposed by Richard Lazarus states that the way
people think about and appraise a stressor is a major Iactor in their stress response. The Iirst step in
appraising a stressor is called primary appraisal and involves estimating the severity oI the stressor and
classiIying it as a threat, challenge, or loss. In secondary appraisal, an individual determines what
resources he or she has available Ior dealing with the threat or loss. Personality has also been linked to
stress-related health risks. In 1974, Meyer Freidman and Ray Rosenman published a book describing the
Type A and Type B personalities and their link to heart disease. Based on studies oI their own patients,
Freidman and Rosenman proposed that individuals with Type A personality (a person who is competitive,
ambitious, workaholic, with a constant sense oI pressure) were more likely to develop heart disease than
someone with a Type B personality. Several studies Iound that the speciIic trait oI hostility in Type A
individuals was the best predictor oI Iuture heart problems. A third personality type called Type C (in
which a person holds in their emotions and tends to be pleasant) was later identiIied and is currently being
investigated as to its possible link with cancer rates. Recent research on personality traits suggests that
individuals who have a high level oI neuroticism, or the tendency to worry, be moody, and emotionally
intense (especially prevalent among those with Type A personalities), may have an increased risk oI an
earlier death because people with these traits tend to engage in poor health habits such as poor diet,
excessive drinking, smoking, and lack oI exercise. Finally, research has suggested a Iourth personality
type, the hardy personality, which is associated with decreased illness due to stress. An individual with a
hardy personality shows commitment, displays a sense oI control, and sees stresses as challenges to be
met and answered. The tendency Ior hardiness may have genetic roots. Research by Cole and colleagues
in 2010 indicated a biochemical link between Ieeling miserable and an increased risk oI death. There may
be a genetic variation in some individuals that severs that link, making the individual more biologically
resilient. In addition to personality, links have been Iound between an individual`s attitude and his or her
physical reactions to stress. SpeciIically, pessimists have been Iound to have signiIicantly more stress-
related health problems than optimists. One way to become an optimist is to recognize any negative
thoughts you are having and to work to get rid oI them.
Social Iactors also play a key role in the amount oI stress an individual experiences. Living in
poverty and job stress are two major sources oI stress. A serious consequence oI job stress is burnout, or
negative changes in thoughts, emotions, and behaviors as a result oI prolonged stress or Irustration.
Acculturative stress describes the stress an individual experiences when having to adapt to a new
culture. The method oI adaptation can aIIect the stress level. Some oI the methods oI adapting to a new
culture include integration, assimilation, separation, and marginalization. The eIIects oI negative social
Iactors on health can be minimized by a strong social support system, or network oI Iamily and Iriends
who can oIIer help when a person is in need.
Coping strategies are actions that people take to master, tolerate, reduce, or minimize the eIIects
oI stressors and include both behavioral and psychological strategies. Problem-focused coping occurs
when a person tries to eliminate the source oI stress or reduce its impact by taking some action, while
emotion-focused coping involves changing the way you Ieel or react to a stressor. Meditation is a series
oI exercises meant to reIocus attention and achieve a trancelike state oI consciousness. Concentrative
Stress and Health CHAPTER 11 -144-
meditation is a Iorm oI meditation in which a person Iocuses the mind on some repetitive or unchanging
stimulus so that the mind can be cleared oI disturbing thoughts and the body can experience relaxation. In
contrast, in receptive meditation, a person attempts to become aware oI everything in immediate
conscious experience, resulting in an expansion oI consciousness. Both concentrative and receptive
meditation have been Iound to be eIIective coping strategies. Culture and religion have also been Iound to
aIIect an individual`s level oI stress as well as the strategies used to cope with that stress.
Exercise can help individuals cope with stress. Exercise has a number oI physical eIIects. It
makes the heart healthier, raises the body`s metabolic rate, helps to maintain a healthy weight, raises
'good and lowers 'bad cholesterol, strengthens bones, improves sleep quality, reduces tiredness, and
increases natural killer cell activity. Exercise has psychological beneIits as well. Individuals who exercise
report lower levels oI anxiety, depression, stress, and anger.

STUDY HINTS
20.

One important component to understanding this chapter is to understand the diIIerence between
a stressor and stress. The stressor is the event that causes us to experience stress. The event can
be external, such as getting stuck in traIIic, or internal, such as worrying about an upcoming
exam. Our reaction to the event is called stress and can be physical, emotional, mental, and
behavioral. Try coming up with some examples oI events that could be considered stressors
along with possible stress reactions. The Iirst example has already been completed Ior you.

Stressor Stress Reaction
Having to take an exam
increased heart rate




21.

Many students Iind the diIIerent types oI conIlicts conIusing. Look over the Rapid Review
section to reIresh yourselI on the meaning oI each type oI conIlict and then try to come up with
an example Irom your own liIe that illustrates each type oI conIlict. List your examples in the
space below.

ApproachApproach ConIlict:





ApproachAvoidance ConIlict:





Stress and Health CHAPTER 11 -145-
AvoidanceAvoidance ConIlict:






LEARNING OBJECTIVES
11.1 How do psvchologists define stress?
11.2 What kinds of external events can cause
stress?
11.3 What are some psvchological factors in
stress?
11.4 How does stress affect the phvsical
functioning of the bodv and its immune
svstem?

11.5 How do cognitive factors and personalitv
differences affect the experience of stress?
11.6 What social factors influence stress reactions?
11.7 What are some wavs in which people cope
with stress reactions?
11.8 How is coping with stress affected bv culture
and religion?
11.9 What are the psvchological benefits of
exercise?


PRACTICE EXAM
For the Iollowing multiple choice questions, select the answer you Ieel best answers the question.

1. The term used to describe the physical, emotional, cognitive, and behavioral responses to events that
are viewed as threatening or challenging is
a) stress.
b) stressors.
c) uncontrollability.
d) pressures.

2. The response an individual might have to an unpleasant stressor, such as losing his job, would be
called
a) eustress.
b) distress.
c) stress appraisal.
d) negative stressors.

3. AIter we have decided that a certain event is a stressor, we must decide how we will deal with it and
what resources are available Ior coping with the stressor. This process is called
a) primary appraisal.
b) secondary appraisal.
c) stress-related decision.
d) hassle-related decision.

4. According to the cognitive-mediational theory oI emotions proposed by Richard Lazarus, which oI
the Iollowing would be the best way to reduce the stress oI losing a job?
a) Try to ignore the problem.
b) Try to understand all the negative implications oI the loss.
c) List all the resources that you do not have available and will need to acquire.
d) View the loss as a challenge and opportunity to explore a new career.

Stress and Health CHAPTER 11 -146-

5. Which oI the Iollowing is an example oI a stressor that would be classiIied as a hassle according to
Richard Lazarus?
a) getting married
b) locking your keys in the car
c) losing your house due to a Ilood
d) the death oI a Iamily member

6. The Social Readjustment Rating Scale (SRRS) measures stress related to
a) positive and negative liIe events.
b) only negative liIe events.
c) only positive liIe events.
d) internal stressors.

7. Gloria is a tax accountant and is very busy Irom January to April 15, which is the tax return Iiling
deadline. She Ieels that she must work long hours during this time to meet the April 15 deadline Ior
her clients. Gloria is experiencing
a) anxiety.
b) pressure.
c) overload.
d) cognitive dissonance.

8. A woman who had an unpleasant conIrontation with her boss and then goes home and yells at the
dog would be displaying
a) uncontrollability.
b) pressure.
c) displaced aggression.
d) catastrophe.

9. Arnold was repeatedly passed up Ior a promotion. In reaction to this Irustration, Arnold quit his job.
Which oI the Iollowing best describes Arnold`s approach to dealing with Irustration?
a) displaced aggression
b) escape or withdrawal
c) downward social comparison
d) projection

10. Three students, Fred, Alice and Carl, were all preparing Ior an important exam. Fred was not at all
stressed about the exam and chose to see a movie instead oI studying. Fred Iailed the exam. Alice
was somewhat stressed about the exam, so she studied each day Ior two weeks. Alice passed the
exam. Carl was extremely stressed about the exam. Although he`d originally planned to study Ior
Iour hours each day, each time he sat down to study, Carl had an anxiety attack. Like Fred, Carl
ultimately Iailed the exam. Which student in the above scenario was experiencing eustress, as
eIined by your textbook? d

a) Fred
b) Alice
c) Carl
d) None oI the students experienced eustress.


Stress and Health CHAPTER 11 -147-

11. Which oI the Iollowing is an example oI an avoidanceavoidance conIlict?
a) a person who enjoys the ocean has to choose between retiring in the Bahamas or in Tahiti
b) a student has to decide whether to turn in an unIinished paper and receive a Iailing grade
or hand it in late and lose many points
c) someone wanting to eat some cake but not wanting the calories
d) a person who loves chocolate must choose between chocolate cake or chocolate ice cream

12. Trying to decide on whether to take a trip to the Bahamas, which would be very enjoyable but
would severely limit the amount oI money you would have to spend on other items, is an example oI
a(n)
a) approachapproach conIlict.
b) approachavoidance conIlict.
c) avoidanceavoidance conIlict.
d) multiple approachavoidance conIlict.

13. The general adaptation syndrome proposed by Hans Selye describes how we respond to stress with
regard to our
a) psychological reactions.
b) emotional reactions.
c) social reactions.
d) physical reactions.

14. According to Selye, some people may develop illnesses such as high blood pressure or weakened
immune system during the stage oI the general adaptation syndrome.
a) alarm
b) collapse
c) exhaustion
d) resistance

15. Stress has been shown to be related to
a) increased resistance to environmental threats.
b) decreased eIIiciency oI the reticular Iormation.
c) increased galvanic skin response.
d) decreased eIIiciency oI the body's immune system.

16. When stress levels are elevated, the amount oI natural killer cells in the body tends to
a) increase.
b) decrease.
c) stay the same.
d) There are not enough data to say at this point.

17. The Type A behavior pattern is a signiIicant predictor oI
a) mental illness.
b) coronary heart disease.
c) cancer.
d) respiratory illnesses .

Stress and Health CHAPTER 11 -148-

18. Someone who would be classiIied as having a Type C personality would be likely to
a) openly express his or her anger at someone.
b) try to always look on the bright side oI a situation.
c) display a great deal oI hostility when things don`t go his or her way.
d) internalize his or her anger so that no one can see his or her true emotion.

19. Which personality type is most likely to strongly agree with the Iollowing statement: 'I can relax
without guilt?.
a) Type A
b) Type B
c) Type C
d) Type F

20. Pepe moved Irom Argentina to France. He chose not to learn to speak and write French, continues to
maintain his old culture's styles oI dress and customs, and lives in a neighborhood where only
people Irom Argentina live. Pepe has used which method oI entering the majority culture?
a) integration
b) assimilation
c) separation
d) marginalization

21. Which method oI acculturation would tend to lead to the greatest degree oI stress?
a) integration
b) assimilation
c) separation
d) marginalization

22. Her mother is ill and Vanna is Ieeling overwhelmed and sad. To cope with this stress oI her mother's
illness, Vanna has been writing her Ieelings down in a journal. Vanna is using
a) problem-Iocused coping.
b) emotion-Iocused coping.
c) distraction.
d) reappraisal.

23. According to your textbook, which personality type may have the highest risk Ior developing
cancer?
a) Type A
b) Type B
c) Type C
d) Type F

24. Stress causes cancer.
a) True
b) False

25. Research shows that lowers blood pressure in adolescents and adults.
a) sensory deprivation
b) concentrative meditation
c) sublimation
d) acculturation

Stress and Health CHAPTER 11 -149-
26. You are a psychologist working with a new client, an immigrant Irom China, who is experiencing
adjustment problems due to stress. Which oI the Iollowing are you Iirst going to consider when
assessing your client's ability to cope?
a) use oI meditative strategies
b) use oI psychological deIense mechanisms
c) ability to use bioIeedback equipment
d) cultural background

27. Several studies have Iound a positive correlation between level oI religious commitment and liIe
expectancies.
a) True
b) False

28. Which oI the Iollowing statements is false?
a) Exercise increases the body`s metabolic rate.
b) Exercise increases levels oI 'bad cholesterol.
c) Exercise decreases anxiety and depression.
d) Exercise increases natural killer cell activity.

PRACTICE EXAM ANSWERS
1. a The response itselI is called stress and the event that causes the response is called a
stressor.
2. b The response to negative stressors is called distress and the response to positive
stressors or the optimal level oI stressors is reIerred to as eustress.
3. b Secondary appraisal involves deciding how to deal with a stressor and estimating
the resources available Ior coping with it, while primary appraisal is the Iirst step
we take when Iacing a potential threat; it involves estimating its severity and
determining whether it is a challenge or a threat.
4. d The cognitive-mediational theory oI emotions suggests that the way we think about
or interpret a stressor is the biggest Iactor in determining our response.
5. b Lazarus Iocused on the minor daily annoyances, such as losing your car keys, as a
signiIicant source oI stress in our lives.
6. a The SRRS assumes that any change (either positive or negative) will serve as a
stressor in an individual`s liIe.
7. b Although anxiety may be a result oI pressure, Gloria is experiencing pressure as a
result oI her need to work longer hours to meet a deadline.
8. c Displaced aggression oIten occurs when the person or object that a person is really
angry at is not an accessible target.
9. b Arnold`s approach to dealing with Irustration is to escape or withdrawal by leaving
his job, the perceived source oI his Irustration.
10. b Alice experienced eustress, deIined as the optimal amount oI stress that people need
to promote health and well-being. Fred was not stressed enough about the exam;
whereas, Carl was too stressed.
11. b Avoidanceavoidance conIlicts involve having to choose between two undesirable
outcomes.
12. b Approachavoidance conIlicts Iocus on one decision that has both positive and
negative aspects to it.
13. d The general adaptation syndrome describes our body`s physical reactions to stress.
14. c During the resistance stage, the body uses its resources to Iight oII the stressor. It is
not until the next stage, exhaustion, that bodily resources are so depleted that stress-
related diseases can develop.
15. d Stress is related to decreased eIIiciency oI the immune system.
Stress and Health CHAPTER 11 -150-
16. b Natural killer cells are important cells in the body that serve to limit the growth oI
cancerous cells. During times oI stress, the level oI natural killer cells tends to
decrease, thus increasing the chances oI tumor growth.
17. b The original development oI the idea oI Type A personality was in order to describe
and predict the individuals who were at high risk Ior heart disease.
18. d Type C personalities tend to internalize their emotions.
19. b According to your textbook, Type B personalities are most likely to strongly agree
with this statement.
20. c Separation occurs when a person tries to maintain his or her original cultural
identity. Assimilation occurs when a person completely gives up his or her old
cultural identity and adopts the majority culture's ways.
21. d Marginalization occurs when an individual is not a part oI his original culture, nor is
he a part oI the new culture. This method oI acculturation has been Iound to create
the greatest amount oI acculturative stress.
22. b Vanna is coping with her stress by Iocusing on and thinking about her emotions.
23. c Type C people tend to have diIIiculty expressing negative emotions. They may
internalize their anger and are oIten lonely. These characteristics have been
associated with a higher risk oI developing cancer.
24. b As stated in the textbook, '.stress itselI cannot give a person cancer. However,
stress can have a suppressing eIIect on the immune system, which makes the
unchecked growth oI cancer more likely.
25. b Concentrative mediation places one in a state oI relaxation and lowers blood
pressure.
26. d Psychological deIense mechanisms are signiIicant but would not be as important in
your initial assessment as would cultural background, especially since the client is
Irom a country with a very diIIerent culture.
27. a Although these studies do not prove a cause-and-eIIect relationship, they have
shown a correlation between religious aIIiliation and longevity.
28. b Exercise increases levels oI 'good cholesterol but decreases levels oI 'bad
cholesterol. All oI the other statements are true.

CHAPTER GLOSSARY
acculturative stress stress resulting Irom the need to change and adapt one`s ways to the
majority culture.
aggression actions meant to harm or destroy.
approach-approach conflict conIlict occurring when a person must choose between two desirable
goals.
approach-avoidance conflict conIlict occurring when a person must choose or not choose a goal that
has both positive and negative aspects.
avoidance-avoidance conflict conIlict occurring when a person must choose between two undesirable
goals.
burnout negative changes in thoughts, emotions, and behavior as a result oI
prolonged stress or Irustration.
catastrophe an unpredictable, large-scale event that creates a tremendous need to
adapt and adjust as well as overwhelming Ieelings oI threat.
College Undergraduate
Stress Scale (CUSS)
assessment that measures the amount oI stress in a college student`s liIe
over a one-year period resulting Irom major liIe events.
concentrative meditation Iorm oI meditation in which a person Iocuses the mind on some
repetitive or unchanging stimulus so that the mind can be cleared oI
disturbing thoughts and the body can experience relaxation.
coping strategies actions that people can take to master, tolerate, reduce, or minimize the
eIIects oI stressors.
Stress and Health CHAPTER 11 -151-
displaced aggression taking out one`s Irustrations on some less threatening or more available
target.
distress the eIIect oI unpleasant and undesirable stressors.
double approach-avoidance
conflict
conIlict in which the person must decide between two goals, with each
goal possessing both positive and negative aspects.
emotion-focused coping coping strategies that change the impact oI a stressor by changing the
emotional reaction to the stressor.
escape or withdrawal leaving the presence oI a stressor, either literally or by a psychological
withdrawal into Iantasy, drug abuse, or apathy.
eustress the eIIect oI positive events, or the optimal amount oI stress that people
need to promote health and well-being.
frustration the psychological experience produced by the blocking oI a desired
goal or IulIillment oI a perceived need.
general adaptation syndrome
(GAS)
the three stages oI the body`s physiological reaction to stress, including
alarm, resistance, and exhaustion.
hardy personality a person who seems to thrive on stress but lacks the anger and hostility
oI the Type A personality.
hassles the daily annoyances oI everyday liIe.
health psychology area oI psychology Iocusing on how physical activities, psychological
traits, and social relationships aIIect overall health and rate oI illnesses.
immune system the system oI cells, organs, and chemicals oI the body that responds to
attacks Irom diseases, inIections, and injuries.
meditation mental series oI exercises meant to reIocus attention and achieve a
trancelike state oI consciousness.
multiple approach-avoidance
conflict
conIlict in which the person must decide between more than two goals,
with each goal possessing both positive and negative aspects.
natural killer (NK) cell immune system cell responsible Ior suppressing viruses and destroying
tumor cells.
optimists people who expect positive outcomes.
pessimists people who expect negative outcomes.
pressure the psychological experience produced by urgent demands or
expectations Ior a person`s behavior that come Irom an outside source.
primary appraisal the Iirst step in assessing stress, which involves estimating the severity
oI a stressor and classiIying it as either a threat or a challenge.
problem-focused coping coping strategies that try to eliminate the source oI a stress or reduce its
impact through direct actions.
psychoneuroimmunology the study oI the eIIects oI psychological Iactors such as stress,
emotions, thoughts, and behavior on the immune system.
receptive meditation Iorm oI meditation in which a person attempts to become aware oI
everything in immediate conscious experience, or an expansion oI
consciousness.
secondary appraisal the second step in assessing a threat, which involves estimating the
resources available to the person Ior coping with the stressor.
Social Readjustment Rating
Scale (SRRS)
assessment that measures the amount oI stress in a person`s liIe over a
one-year period resulting Irom major liIe events.
social support system the network oI Iamily, Iriends, neighbors, coworkers, and others who
can oIIer support, comIort, or aid to a person in need.
stress the term used to describe the physical, emotional, cognitive, and
behavioral responses to events that are appraised as threatening or
challenging.
stressors events that cause a stress reaction.
Stress and Health CHAPTER 11 -152-
Type 2 diabetes disease involving Iailure oI the pancreas to secrete enough insulin
necessitating medication, usually diagnosed beIore the age oI 40 and
can be associated with obesity.
Type A personality person who is ambitious, time-conscious, extremely hard-working, and
tends to have high levels oI hostility and anger as well as being easily
annoyed.
Type B personality person who is relaxed and laid-back, less driven and competitive than
Type A and slow to anger.
Type C personality pleasant but repressed person, who tends to internalize his or her anger
and anxiety and who Iinds expressing emotions diIIicult.

Stress and Health CHAPTER 11 -153-
Stress and Health CHAPTER 11 -154-
CHAPTER 12 - SOCIAL PSYCHOLOGY


YOU KNOW YOU ARE READY FOR THE TEST IF YOU ARE ABLE TO.
Describe the role social inIluence plays on conIormity, compliance, and obedience.
Discuss the issues oI social cognition including the Iormation and development oI attitudes,
impressions, and attributions.
Introduce concepts oI social interaction including prejudice, discrimination, interpersonal
attraction, aggression, and altruism.

RAPID REVIEW
Social psychology is the scientiIic study oI how a person`s behavior, thoughts, and Ieelings are
inIluenced by the real, imagined, or implied presence oI others. Social psychology can be broadly divided
into the areas oI social inIluence, social cognition, and social interaction.
Social influence is the process in which the presence oI other people inIluences the behavior,
Ieelings, and thoughts oI an individual. Conformity involves changing one`s own behavior to more
closely match the actions oI others. In 1951, Solomon Asch conducted a classic experiment on conIormity
by having subjects judge the length oI a line aIter hearing a group oI conIederates all report an obviously
incorrect answer. Asch Iound that the subjects conIormed to the group answer around one-third oI the
time and that conIormity increased as the group size increased, up to a group oI Iour conIederates. In a
later study, Asch Iound that conIormity greatly decreased when at least one conIederate gave the right
answer. Groupthink is a type oI conIormity in which people Ieel it is more important to maintain the
group`s cohesiveness than to consider the Iacts more realistically. Social inIluence can also be used to
describe the phenomenon oI compliance, which occurs when people change their behavior as a result oI
another person or the group asking or directing them to change. Consumer psychology is an area oI
psychology that studies how people get other people to buy things. There are a number oI techniques that
people use to obtain the compliance oI others including the foot-in-the-door technique, in which
compliance with a small request is Iollowed by a larger request, and the door-in-the-face technique,
which is the process oI making a large request that is almost always reIused and then a smaller request
that is oIten agreed to. The door-in-the-Iace technique relies on the norm of reciprocity, which states that
iI someone does something Ior you, you should do something in return. Two additional compliance
techniques include the lowball technique in which the cost oI the commitment is increased after the
commitment is already made and the that`s-not-all technique in which an oIIer is made and beIore the
individual can make a decision, something 'extra is added to the oIIer. In compliance, an individual
changes his or her behavior because someone asks him or her; in obedience, an individual changes his or
her behavior because an authority Iigure gives an order to him or to her. Stanley Milgram conducted one
oI the most Iamous experiments on obedience in which he measured the number oI volts a participant
would administer to another participant simply because the experimenter instructed him or her to do so. In
reality, no electrical shocks were being administered. Milgram Iound that about two-thirds oI the subjects
(65 percent) administered electrical shocks up to a lethal level oI 450 volts when instructed to do so.
Repetition oI these experiments in the United States and in other countries has conIirmed that between 61
and 66 percent oI participants will go all the way iI instructed to do so. Interestingly, a study by Slater and
colleagues demonstrated that even when participants are given the opportunity to 'shock a virtual human
(one generated by a computer), participants reacted physiologically to the suIIering oI the virtual human
as iI the 'person being shocked were real, even though the participants were Iully aware that that they
were shocking a computer-generated, virtual human.
The presence oI others can also inIluence how well an individual perIorms a speciIic task in a
process. For instance, group polarization is the tendency Ior members involved in a group discussion to
take somewhat more extreme positions and suggest riskier actions when compared to individuals who
have not participated in a group discussion. A good example oI group polarization can occur when a jury
tries to
Social Psychology CHAPTER 12 -155-
decide on punitive damages during a civil trial: Studies have Iound that iI members oI a jury individually
Iavor a relatively low amount oI punitive damages beIore deliberation, aIter deliberation the amount
usually lessened Iurther. The positive inIluence oI others on perIormance is called social facilitation,
while the negative inIluence is sometimes called social impairment. II the task is easy, the presence oI
others seems to improve perIormance, but iI the task is diIIicult, the presence oI others actually has a
negative impact on perIormance. Social loafing describes the tendency Ior people to put less eIIort into a
simple task when working in a group as opposed to working alone.
Social cognition deals with the ways people think about other people and includes attitudes,
impressions, and attributions. An attitude can be deIined as a tendency to respond positively or
negatively toward a certain idea, person, object, or situation. Attitudes are composed oI the way people
Ieel, act, and think. The aIIective component describes the Ieelings associated with attitudes, the behavior
component describes the actions, and the cognitive component describes the thoughts. Attitudes have
been Iound to be only weak predictors oI actual behavior. Attitude Iormation is a learning process that
occurs through direct contact, direct instruction, interaction with others, and vicarious (or observational)
learning. Persuasion is the process by which one person tries to change the belieI, opinion, position, or
course oI action oI another person through argument, pleading, or explanation. Factors that inIluence the
eIIectiveness oI persuasion include the source, the message, and the target audience. The elaboration
likelihood model examines how likely it is that an individual will elaborate on a persuasive message and
what the outcome oI the elaboration will most likely be. When people attend to the content oI the
message, the model describes it as central-route processing, and when people pay attention to
inIormation outside oI the message content itselI, it is reIerred to as peripheral-route processing.
Cognitive dissonance is a sense oI discomIort that occurs when a person`s behavior does not match up
with that person`s attitudes. When a person experiences cognitive dissonance, he or she typically changes
the conIlicting behavior to match the attitude, changes the attitude to match his or her behavior, or Iorms
new cognitions to justiIy his or her behavior. Impression formation involves the process oI Iorming the
Iirst knowledge that a person has concerning another person, in other words, the 'Iirst impression. One
component oI impression Iormation involves social categorization, which is the assignment oI a person
to a category based on characteristics the person has in common with other people with whom one has
had experience in the past. Social categorization can oIten result in stereotypes, or a set oI characteristics
that people believe are shared by all members oI a particular social category. People oIten Iorm their own
categories based on implicit personality theories, or sets oI assumptions about how diIIerent types oI
people, personality traits, and actions are all related. Most implicit personality theories are Iormed in
childhood. The Iinal aspect oI social cognition discussed in the textbook is attribution, or the process oI
explaining one`s own behavior and the behavior oI others. Fritz Heider originally described attribution
theory and divided attributions into two categories: situational causes were explanations that relied on
external causes, and dispositional causes assume behavior is the result oI some internal Iactor. The
fundamental attribution error is the most well-known bias oI attribution and is the tendency Ior some
people to almost exclusively use dispositional attributes to explain other people`s behavior.
Social interaction, or the relationship between people, is the third main area oI study in the Iield
oI social psychology. When a person holds an unsupported and oIten negative attitude about the members
oI a particular group it is called a prejudice, and when a person acts diIIerently toward a person based on
that attitude it is called discrimination. The creation oI in-groups and out-groups can oIten intensiIy
discrimination. The realistic conflict theory states that prejudice and discrimination will be increased
between groups that are in conIlict. Jane Elliot used her second-grade classroom to demonstrate the power
oI prejudice and discrimination by dividing her class based on the color oI the students` eyes and
observing the eIIects. ConIlicts between groups tend to increase as pressures and stresses increase. OIten
the prejudice exists because oI the need Ior a scapegoat, a person or group who serves as the target Ior the
Irustrations and negative emotions oI the group with the prejudiced attitude. Several theories have been
proposed to explain the Iormation and persistence oI prejudice. In social cognitive theory, prejudice is
seen as an attitude that is Iormed through direct instruction, modeling, and other social inIluences on
learning. Social identity theory suggests that the three processes oI social categorization, social
identity, and social comparison are involved in the Iormation oI prejudiced attitudes. Stereotype
Social Psychology CHAPTER 12 -156-
vulnerability reIers to the eIIect that a person`s knowledge oI someone else`s stereotyped opinion can
have on that person`s behavior. The resulting Ieeling oI anxiety is reIerred to as stereotype threat. The
negative impact oI stereotype threat on an individual`s perIormance can actually cause a person to act in
the way that the stereotype predicts, thus conIirming an outside observer`s prejudiced attitude. Self-
fulfilling prophecy occurs when a person acts according to his or her existing belieIs and his or her
actions make it more likely that his or her belieIs are conIirmed. A recent study by Rydell and Boucher
Iound that some people can overcome Ieelings oI stereotype threat by identiIying themselves with a
diIIerent social identity. Women were able to overcome stereotype threat by identiIying with 'college
students when taking a math exam rather than with 'Iemales (because Iemales are oIten stereotyped as
being math deIicient). However, this eIIect only held Ior those women with Iairly high selI-esteem. The
best deIense against prejudice is becoming inIormed about people who are diIIerent Irom you. Equal
status contact, in which all individuals involved have the same amount oI power in the situation, is
crucial Ior reducing prejudice. Educators have attempted to create situations oI equal status in the
classroom by setting up jigsaw classrooms, in which students have to work together to reach a speciIic
goal.
Another area oI social interaction discussed in your textbook is interpersonal attraction, or
liking or having the desire Ior a relationship with someone else. Several Iactors are involved in the
attraction oI one person to another including physical attractiveness, proximity (or how close a person is
to you physically), similarity, and reciprocity of liking (or liking someone who likes you). Robert
Sternberg proposed a theory oI love that contains three components: intimacy, passion, and commitment.
He Ielt that seven types oI love could be described by various combinations oI these three components.
Two oI Sternberg`s proposed types oI love are romantic love and companionate love.
A very diIIerent type oI social interaction is that oI violence. Aggression is deIined as any
behavior intended to hurt or destroy another person. Social psychologists have examined the role oI both
biology and the environment on aggression. Twin studies have shown a higher correlation oI aggression
levels in identical twins than in Iraternal twins. Certain areas oI the brain have been Iound to control
aggressive responses, and testosterone levels are related to aggression. However, a large portion oI human
aggression is inIluenced by learning. Several studies have suggested that taking on a particular social role
can lead to an increase in aggressive behavior. One classic study was conducted by the social
psychologist Philip Zimbardo at StanIord University in 1971. In this study about 70 young men
volunteered to participate Ior two weeks. They were told that they would be randomly assigned to the
social role oI either a guard or a prisoner in the experiment. On Day 2, the prisoners staged a revolt (not
planned as part oI the experiment), which was quickly crushed by the guards. The guards then became
increasingly more aggressive, using humiliation to control and punish the prisoners. The staII observing
the experiment had to release Iive oI the prisoners who became so upset that they were physically ill. The
study, which had originally been scheduled to last Ior 2 weeks, had to be cancelled on the IiIth day. This
study highlighted the inIluence that a social role, such as that oI 'guard, can have on perIectly ordinary
people. A number oI studies have also supported the link between exposure to violent media and
aggression.
The Iinal area oI social interaction discussed in your textbook is prosocial behavior, or socially
desirable behavior that beneIits others rather than bringing them harm. Altruism is a speciIic type oI
prosocial behavior in which an individual helps someone else with no expectation oI reward. Sometimes
the presence oI other people can decrease the likelihood oI prosocial behavior as can be seen in the
bystander effect and diffusion of responsibility. Bibb Latane and John Darley conducted a series oI
experiments that Iound that participants were less likely to respond to an emergency situation where other
people were present than when they were alone. Some oI the decisions an individual must make when
deciding whether to oIIer help include noticing the situation, deIining the situation as an emergency,
taking responsibility, planning a course oI action, and taking action.
Technically, a cult reIers to any group oI people with a particular religious or philosophical set oI
belieIs and identities; however, most people associate the term cult with a group oI people whose belieIs
are so diIIerent Irom the mainstream that they are viewed with suspicion.

Social Psychology CHAPTER 12 -157-
STUDY HINTS
22.

The text introduces Iour common methods that are used to gain the compliance oI another
person. In order to better understand the diIIerences among these methods, assume that you are
trying to get your Iriend to come pick you up and then go shopping at the mall with you. In the
space below, come up with an example oI how you might get your Iriend to comply with your
request using each oI the techniques listed.

Technique Example
Foot-in-the-door

Door-in-the-Iace

Lowball

That`s-not-all


Social Psychology CHAPTER 12 -158-

23.

Social psychology contains a large number oI well-known researchers along with the Iamous studies
they carried out. It is important to be able to remember which researcher goes with which study. Next
to the researchers listed here, brieIly describe the experiment they carried out along with the topic they
studied. In the Iinal column, come up with a mnemonic to help you remember the inIormation.

Researcher Experiment Topic Mnemonic
Solomon Asch

Stanley Milgram

Jane Elliot

Latane and Darley

Philip Zimbardo





LEARNING OBJECTIVES
12.1 What factors influence people to conform to
the actions of others?
12.2 How is compliance defined, and what are
four common wavs to gain the compliance
of another?
12.3 What factors make obedience more likelv?
12.4 What are the three components of an
attitude, how are attitudes formed, and how
can attitudes be changed?
12.5 How do people react when attitudes and
behavior are not the same?
12.6 What are social categori:ation and implicit
personalitv theories?

12.7 How do people trv to explain the actions of
others?
12.8 How are prefudice and discrimination different?
12.9 Whv are people prefudiced, and how can
prefudice be stopped?
12.10 What factors govern attraction and love, and
what are some different kinds of love?
12.11 How is aggressive behavior determined bv
biologv and learning?
12.12 What is altruism, and how is deciding to help
someone related to the presence of others?
12.13 Whv do people foin cults?
Social Psychology CHAPTER 12 -159-

PRACTICE EXAM
For the Iollowing multiple choice questions, select the answer you Ieel best answers the question.
1. Vince has always believed children deserve the best prenatal care available. During a class
discussion, he hears the Iirst oI several speakers express negative attitudes toward spending tax
money on prenatal care Ior the poor. When it is his turn to speak, he voices an opinion more in
keeping with the previous speakers. Vince's behavior is an example oI
a) compliance.
b) persuasion.
c) conIormity.
d) obedience.

2. Which oI the Iollowing researchers conducted a series oI studies on conIormity that involved having a
subject judge the length oI three lines aIter a group oI conIederates all reported an obviously incorrect
answer?
a) Jane Elliot
b) Stanley Milgram
c) Philip Zimbardo
d) Solomon Asch

3. occurs when people begin to think that it is more important to maintain a group`s
cohesiveness than to objectively consider the Iacts.
a) Groupthink
b) The lowball technique
c) Obedience
d) Social loaIing

4. All oI the Iollowing are causes Ior groupthink EXCEPT
a) the belieI that the group can do no wrong.
b) the belieI that the group is invulnerable.
c) the belieI that opposition to the group is unsound.
d) openness to diIIering opinions.

5. At the supermarket, a demonstrator gives away Iree samples oI a new pizza. He also gives each
taster a coupon worth $1 oII his or her grocery bill. This manuIacturer is depending on the social
process oI to increase sales.
a) norm oI reciprocity
b) deindividuation
c) group polarization
d) social Iacilitation

6. Selena is trying to get her boyIriend to wash the dishes Ior her. To start with, she asks her boyIriend
to cook dinner Ior her. When her boyIriend reIuses, she asks, 'Well, will you at least wash the
dishes then? To which he readily agrees. Selena has just used the
a) Ioot-in-the-door technique.
b) door-in-the-Iace technique.
c) lowball technique.
d) that`s-not-all technique.

Social Psychology CHAPTER 12 -160-

7. Changing one`s behavior due to a direct order oI an authority Iigure is reIerred to as
a) compliance.
b) obedience.
c) conIormity.
d) persuasion.

8. Imagine 100 individuals are asked to take part in a replication oI Milgram's Iamous study on
obedience. How are these 100 people likely to respond?
a) The majority would administer 450 volts as instructed.
b) The majority would immediately realize the use oI deception and leave.
c) Most oI the women would reIuse to obey, whereas almost all oI the men would obey.
d) Most oI the participants would work together to Iorce the experimenter to
end the experiment.

9. A teacher decides against assigning group projects in which all group members get the same grade.
What social psychological phenomenon might the teacher be concerned about?
a) conIormity
b) social loaIing
c) social inIluence
d) social Iacilitation

10. Ashley has practiced her drum routine over and over. When she gets up to play it at the recital in
Iront oI 100 people, she perIorms it better than she ever has. Her improved perIormance is an
example oI
a) social compliance.
b) persuasion.
c) social Iacilitation.
d) social impairment.

11. Which oI the Iollowing is the best example oI the behavioral component oI an attitude?
a) Bea Ieels recycling is a great concept.
b) Bob is upset when he hears a corporation plans to build a polluting plant near his home.
c) Bill struggles to understand the arguments both sides present in a debate over a new
manuIacturing plant.
d) Betty writes a letter to her senator asking Ior support oI a law making corporations
responsible Ior the pollution they cause.

12. Which oI the Iollowing is not a Iactor that inIluences attitude Iormation?
a) direct contact with an individual
b) DNA inherited Irom your parents
c) instructions Irom your parents
d) observing someone else`s actions

13. Kerry's positive attitude toward China, even though she has never been there, seems to be related to
the Iact that her mother is Chinese and talks about China all the time with Kerry. Which method oI
attitude Iormation is involved in this example?
a) direct contact
b) direct instruction
c) interaction with others
d) classical conditioning

Social Psychology CHAPTER 12 -161-
14. Which communicator would likely be most persuasive?
a) an attractive person who is an expert
b) a moderately attractive person who is an expert
c) an attractive person who has moderate expertise
d) a moderately attractive person who has moderate expertise

15. describes the situation in which people attend to the content oI a message.
a) Central-route processing
b) Cognitive dissonance
c) Social Iacilitation
d) Peripheral-route processing

16. Which oI the Iollowing was a Iinding in the classic study by Festinger and Carlsmith (1959)?
a) Those who got $1 to perIorm a boring task said the task was more interesting than did
those who got $2.
b) Those who got $20 to perIorm a boring task said the task was more interesting than did
those who got $1.
c) Paid groups said the task was less boring than did nonpaid groups.
d) Women perIormed the tasks Ior less money than men.

17. Which oI the Iollowing represents an example oI cognitive dissonance?
a) A boy learns how to ride a bicycle without the training wheels.
b) A Iather tells his daughter that he will really only be proud oI her iI she gets all A`s like
she did last semester.
c) A student stays up all night to study Ior an upcoming exam.
d) A woman argues that it is morally wrong to kill animals Ior Iood becomes upset when she
is asked to explain why she is wearing a leather belt and leather shoes.

18. What is the term Ior the process oI developing an opinion about another person?
a) social interaction
b) stereotyping
c) impression Iormation
d) interpersonal judgment

19. Toni sees a picture oI the new international exchange student and notices that the student looks
happy, so Toni automatically assumes that he is also Iriendly. This automatic assumption about the
student`s personality is an example oI
a) central-route processing.
b) implicit personality theory.
c) cognitive dissonance.
d) discrimination.

20. The process oI explaining one`s own behavior and the behavior oI other people is called
a) stereotyping.
b) attribution.
c) central-route processing.
d) cognitive dissonance .

Social Psychology CHAPTER 12 -162-

21. "Look, OIIicer, I didn't see the stop sign back there because the sun was in my eyes." The police
oIIicer responds, "You were not paying attention." How would a social psychologist describe this
situation?
a) Both individuals were making Iundamental attribution errors.
b) Both individuals were making situational attributions.
c) The driver was making a dispositional attribution; the oIIicer was making a situational
attribution.
d) The driver was making a situational attribution; the oIIicer was making a dispositional
attribution.

22. While watching the TV game show Jeopardv, your roommate says, "The game show host, Alex
Trebek, knows all the answers. He must be a genius." You tell your roommate she probably would
not have said that iI she had attended class the day the instructor discussed the topic oI
a) social Iacilitation.
b) stereotyping illusions.
c) internal attribution biases.
d) Iundamental attribution errors.

23. A bank loan oIIicer thinks people who speak with an accent are lazy; consequently, he reIuses to
grant them loans. The loan oIIicer's belieI is an example oI . His reIusal to grant them loans
is an example oI .
a) discrimination; prejudice
b) stereotyping; attribution
c) attribution; stereotyping
d) prejudice; discrimination

24. The part oI a person's selI-concept that is based on his or her identiIication with a nation, culture, or
ethnic group or with gender or other roles in society is called
a) the Iundamental attribution error.
b) selI-serving bias.
c) ethnocentrism.
d) social identity.

25. Which oI the Iollowing does NOT represent an eIIective method Ior reducing prejudice?
a) establishing a jigsaw classroom
b) bringing diverse groups oI people into contact with each other
c) learning about people who are diIIerent Irom you
d) establishing equal status contact between diIIerent groups oI people

26. We tend to attractive people more than we do less attractive people.
a) like
b) dislike
c) ignore
d) hate

27. When opposites attract it is said that they have characteristics.
a) proximal
b) complementary
c) rewarding
d) reciprocal
Social Psychology CHAPTER 12 -163-

28. Which oI the Iollowing was NOT a component oI Robert Sternberg`s theory oI love?
a) intimacy
b) lust
c) passion
d) commitment

29. Behavior that is intended to hurt or destroy another person is reIerred to as
a) empty love.
b) prejudice.
c) aggression.
d) dissonance.

30. The Iact that a social role can lead to an increase in aggressive behavior points to as a
major contributor to aggression.
a) biology
b) the environment
c) DNA
d) chemical inIluences

31. What term reIers to helping behavior that is perIormed voluntarily Ior the beneIit oI another person,
with no anticipation oI reward?
a) altruism
b) collectivism
c) interdependence
d) humanitarianism

32. In a crowded mall parking lot, dozens oI people hear a Iemale voice yell, "He's killing me!" Yet, no
one calls the police. What is the reason Ior the lack oI action, according to Darley and Latane?
a) People are too busy to respond.
b) Most people 'do not want to become involved.
c) The Iight-or-Ilight response is not activated when others are in danger.
d) There is a diIIusion oI responsibility.

33. In Latane and Darley`s classic 1969 study, they Iound that oI the participants reported
the smoke in the room when the two conIederates in the room noticed the smoke but then ignored it.
a) all
b) three-Iourths
c) one-halI
d) one-tenth

34. All oI the Iollowing are decision points in helping behavior EXCEPT
a) noticing.
b) deIining an emergency.
c) taking responsibility.
d) diIIusion oI responsibility.

PRACTICE EXAM ANSWERS
1. c ConIormity involves going along with the group despite one's real opinion.
Compliance would be the case iI someone had asked him to voice an opinion in
keeping with the previous speakers. In this case, Vince did it on his own as a result
oI internal pressure to conIorm.
Social Psychology CHAPTER 12 -164-
2. d Asch conducted the well-known studies on conIormity. Milgram studied obedience
in his Iamous studies with electrical shock.
3. a Groupthink describes the thought processes that can dominate a group oI
individuals.
4. d Groupthink results in lack oI diIIering opinions. Believing that the group can do no
wrong is actually a cause Ior groupthink.
5. a The norm oI reciprocity involves the tendency oI people to Ieel obligated to give
something in return aIter they have received something. Social Iacilitation is an
increase in perIormance caused by greater arousal.
6. b The door-in-the-Iace technique involves asking Ior a large request that you know
will be reIused Iollowed up by a smaller request, which many people then agree to.
7. b Obedience involves changing your behavior due to an order Irom 'above, while
conIormity involves changing your behavior to better 'Iit in with others around you.
8. a The Milgram experiment has been repeated at various times, in the United States
and in other countries, and the percentage oI participants who went all the way
consistently remained between 61 and 66 percent. In addition, Iew diIIerences
between males and Iemales have been Iound.
9. b The teacher knows that some students will slack oII iI they are not being evaluated
Ior their individual perIormance, due to a phenomenon known as social loaIing.
10. c Social facilitation is the term Ior the positive eIIect on one's perIormance caused by
the perception that others are watching.
11. d Writing is an action, or behavior. The Iact that Bill struggled to understand indicates
that what he is doing is cognitive.
12. b Attitude Iormation is believed to occur solely through the learning process and is
not considered to be something that is inherited biologically.
13. c The Iact that Kerry's mother talks about China all the time with Kerry and is
Chinese indicates that her attitude is the result oI interaction with her mother.
14. a Attractiveness and expertise have been shown to increase persuasiveness.
15. a In central-route processing, an individual pays attention to the content oI the
message, whereas in peripheral-route processing, an individual Iocuses on details
other than the main content oI the message.
16. a The group that got paid less used cognitive dissonance to justiIy their poor pay Ior
telling a lie.
17. d Cognitive dissonance is an emotional disturbance that occurs when a person`s
actions don`t match his or her statements.
18. c Although stereotyping may be a component oI impression Iormation, it is not the
term Ior the process oI developing an opinion about another person.
19. b Implicit personality theory represents the automatic associations a person makes
about personality traits that are assumed to be related.
20. b An attribute is an explanation Ior a person`s behavior. Stereotypes are preconceived
ideas about a group oI people.
21. d The driver attributed his error to something in his situation, the sun; whereas the
oIIicer attributed the driver`s error to something internal to him, his lack oI
attention.
22. d Your roommate attributed something that is situational (Trebek gets the answers
ahead oI time) to an internal characteristic (genius). Although internal attribution
bias sounds correct, it is not a term used in social psychology.
23. d Prejudice is an unsupported, oIten negative belieI about all people in a particular
group, whereas discrimination is an action taken that is based on this belieI. In this
case, the action is the reIusal to grant loans.
Social Psychology CHAPTER 12 -165-

24. d Social identity reIers to a person`s identity with his or her social group.
Ethnocentrism is the process oI viewing the world Irom your own viewpoint and
Iailing to see alternative perspectives.
25. b Simply bringing groups together normally does not reduce prejudice unless all the
members oI the group have equal status and power in the group.
26. a Social psychologists have Iound that we tend to like attractive people more than
unattractive people.
27. b Things that "complement" each other tend to be opposites. The term proximitv
reIers to nearness.
28. b Sternberg`s theory oI love includes the three components oI intimacy, passion, and
commitment.
29. c Aggression describes a type oI behavior, whereas prejudice reIers to a person`s
attitude.
30. b The impact oI the social role points to learning and the inIluence oI the surrounding
environment on an individual`s aggressive behavior.
31. a Altruism is deIined as helping others Ior no personal beneIit. Humanitarianism
means almost the same thing as altruism but is not the term social psychologists use
Ior the helping behavior that is perIormed voluntarily Ior the beneIit oI another
person, with no anticipation oI reward.
32. d According to Latane and Darley most people say they do want to become involved,
however oIten diIIusion oI responsibility occurs. DiIIusion oI responsibility is what
occurs as each person thinks someone else will call Ior help (i.e., take
responsibility).
33. d About one-tenth oI the participants reported smoke when the conIederates in the
room noticed the smoke but did nothing about it. This number was much higher
when the participants were in the room alone.
34. d DiIIusion oI responsibility stops a person Irom helping and is not considered a
decision point.

CHAPTER GLOSSARY
aggression behavior intended to hurt or destroy another person.
altruism prosocial behavior that is done with no expectation oI reward and may
involve the risk oI harm to oneselI.
attitude a tendency to respond positively or negatively toward a certain person,
object, idea, or situation.
attribution the process oI explaining one`s own behavior and the behavior oI others.
attribution theory the theory oI how people make attributions.
bystander effect reIerring to the eIIect that the presence oI other people has on the decision
to help or not help, with help becoming less likely as the number oI
bystanders increases.
central-route processing type oI inIormation processing that involves attending to the content oI the
message itselI.
cognitive dissonance sense oI discomIort or distress that occurs when a person`s behavior does
not correspond to that person`s attitudes.
companionate love type oI love consisting oI intimacy and commitment.
compliance changing one`s behavior as a result oI other people directing or asking Ior
the change.
conformity changing one`s own behavior to match that oI other people.
consumer psychology branch oI psychology that studies the habits oI people in the marketplace.
Social Psychology CHAPTER 12 -166-

cult any group oI people with a particular religious or philosophical set oI
belieIs and identity.
diffusion of responsibility occurring when a person Iails to take responsibility Ior actions or Ior
inaction because oI the presence oI other people who are seen to share the
responsibility.
discrimination treating people diIIerently because oI prejudice toward the social group to
which they belong.
dispositional cause cause oI behavior attributed to internal Iactors such as personality or
character.
door-in-the-face
technique
asking Ior a large commitment and being reIused, and then asking Ior a
smaller commitment.
elaboration likelihood
model
model oI persuasion stating that people will either elaborate on the
persuasive message or Iail to elaborate on it, and that the Iuture actions oI
those who do elaborate are more predictable than those who do not.
equal status contact contact between groups in which the groups have equal status, with neither
group having power over the other.
foot-in-the-door
technique
asking Ior a small commitment and, aIter gaining compliance, asking Ior a
bigger commitment.
fundamental attribution
error
the tendency to overestimate the inIluence oI internal Iactors in determining
behavior while underestimating situational Iactors.
groupthink kind oI thinking that occurs when people place more importance on
maintaining group cohesiveness than on assessing the Iacts oI the problem
with which the group is concerned
group polarization the tendency Ior members involved in a group discussion to take somewhat
more extreme positions and suggest riskier actions when compared to
individuals who have not participated in a group discussion.
implicit personality
theory
sets oI assumptions about how diIIerent types oI people, personality traits,
and actions are related to each other.
impression formation the Iorming oI the Iirst knowledge that a person has concerning another
person.
in-groups social groups with whom a person identiIies; 'us.
interpersonal attraction liking or having the desire Ior a relationship with another person.
jigsaw classroom educational technique in which each individual is given only part oI the
inIormation needed to solve a problem, causing the separate individuals to
be Iorced to work together to Iind the solution.
lowball technique getting a commitment Irom a person and then raising the cost oI that
commitment.
norm of reciprocity assumption that iI someone does something Ior a person, that person should
do something Ior the other in return.
obedience changing one`s behavior at the command oI an authority Iigure.
out-groups social groups with whom a person does not identiIy; 'them.
peripheral-route
processing
type oI inIormation processing that involves attending to Iactors not
involved in the message, such as the appearance oI the source oI the
message, the length oI the message, and other noncontent Iactors.
persuasion the process by which one person tries to change the belieI, opinion,
position, or course oI action oI another person through argument, pleading,
or explanation.
prejudice negative attitude held by a person about the members oI a particular social
group.
prosocial behavior socially desirable behavior that beneIits others.
Social Psychology CHAPTER 12 -167-
Social Psychology CHAPTER 12 -168-
proximity physical or geographical nearness.
realistic conflict theory theory stating that prejudice and discrimination will be increased between
groups that are in conIlict over a limited resource.
reciprocity of liking tendency oI people to like other people who like them in return.
romantic love type oI love consisting oI intimacy and passion.
self-fulfilling prophecy the tendency oI one`s expectations to aIIect one`s behavior in such a way as
to make the expectation more likely to be occur.
situational cause cause oI behavior attributed to external Iactors, such as delays, the action oI
others, or some other aspect oI the situation.
social categorization the assignment oI a person one has just met to a category based on
characteristics the new person has in common with other people with whom
one has had experience in the past.
social cognition the mental processes that people use to make sense oI the social world
around them.
social cognitive theory reIerring to the use oI cognitive processes in relation to understanding the
social world.
social comparison the comparison oI oneselI to others in ways that raise one`s selI-esteem.
social facilitation the tendency Ior the presence oI other people to have a positive impact on
the perIormance oI an easy task.
social identity the part oI the selI-concept including one`s view oI selI as a member oI a
particular social category.
social identity theory theory in which the Iormation oI a person`s identity within a particular
social group is explained by social categorization, social identity, and social
comparison.
social impairment the tendency Ior the presence oI other people to have a negative impact on
the perIormance oI a diIIicult task.
social influence the process through which the real or implied presence oI others can
directly or indirectly inIluence the thoughts, Ieelings, and behavior oI an
individual
social loafing the tendency Ior people to put less eIIort into a simple task when working
with others on that task.
social psychology the scientiIic study oI how a person`s thoughts, Ieelings, and behavior are
inIluenced by the real, imagined, or implied presence oI others
social role the pattern oI behavior that is expected oI a person who is in a particular
social position.
stereotype vulnerability the eIIect that people`s awareness oI the stereotypes associated with their
social group has on their behavior.
stereotype a set oI characteristics that people believe is shared by all members oI a
particular social category.
that`s-not-all technique a sales technique in which the persuader makes an oIIer and then adds
something extra to make the oIIer look better beIore the target person can
make a decision.

CHAPTER 13 - THEORIES OF PERSONALITY


YOU KNOW YOU ARE READY FOR THE TEST IF YOU ARE ABLE TO.
DeIine personality according to the various perspectives in psychology.
Discuss Freud`s psychoanalytical perspective on personality and modiIications oI his theory by
the neo-Freudians.
Describe the behaviorists` perspective on personality and the social cognitive theory including
Albert Bandura`s model.
Introduce the humanistic perspective oI personality including Carl Rogers`s view oI the selI and
concept oI unconditional positive regard.
Discuss trait theory with regard to the description oI personality.
Explain what is known about the role oI biology and heredity in personality development.
Describe major methods oI personality assessment including interviews, projective tests,
behavioral assessment, and personality inventories.

RAPID REVIEW
Personality is the unique way in which each individual thinks, acts, and Ieels throughout liIe.
Two components oI personality are character, which reIers to value judgments made about a person`s
morals or ethical behavior and temperament, or the enduring characteristics a person is born with. Four
perspectives regarding personality include the psychoanalytic, behaviorist, humanistic, and trait
perspectives.
The psychoanalytic perspective originated with the theories oI Sigmund Freud and Iocuses on
the role oI unconscious thoughts and desires in the development oI personality. It is important to take into
account the sexually repressed Victorian era in which Freud grew up when evaluating his theory or
personality. Freud believed the mind was divided into three parts: the conscious mind contains all the
things a person is aware oI at any given moment; the preconscious mind contains all the memories and
Iacts that can be recalled with only minimal eIIort; and the unconscious mind remains hidden at all times.
Freud believed the unconscious mind was the most important Iactor in directing behavior and personality.
In addition to the divisions oI the mind, Freud also believed that personality could be divided into three
components: the id, ego, and superego. The id resides completely in the unconscious mind and represents
the most primitive part oI the personality containing the basic biological drives such as hunger, thirst, and
sex. According to Freud, the id operates on the pleasure principle, which attempts to seek immediate
gratiIication oI needs with no regard Ior consequences. Freud reIerred to the psychological tension
created by a person`s unconscious desires as the libido. The ego represents the mostly conscious and
rational aspect oI personality, which operates on the reality principle, attempting to satisIy the desires oI
the id in a way that will minimize negative consequences. The superego is the last part oI the personality
to develop according to Freud`s theory and represents the moral center oI personality. The superego
contains the conscience, or the part oI personality that makes a person Ieel good or bad, depending on
whether they do the right or wrong thing. According to Freud, the id demands immediate satisIaction,
while the superego places restrictions on which behaviors are morally acceptable, and the ego is leIt in the
middle to come up with a compromise.
The psychological defense mechanisms are ways oI dealing with stress through unconsciously
distorting one`s perception oI reality. These deIense mechanisms were mainly outlined and studied by
Freud`s daughter, Anna Freud, who was a psychoanalyst. In order Ior the three parts oI the personality to
Iunction, the constant conIlict among them must be managed, and Freud assumed that the deIense
mechanisms were one oI the most important tools Ior dealing with the anxiety caused by this conIlict.
These mechanisms include denial, repression, rationalization, projection, reaction formation,
displacement, regression, identification, compensation (substitution), and sublimation.
For Sigmund Freud, the three components oI personality develop in a series oI psychosexual
stages with each stage Iocused on a diIIerent erogenous zone, or area oI the body that produces
Theories of Personality CHAPTER 13 -169-
pleasurable Ieelings. Unresolved conIlicts at any oI the stages oI development can lead to fixation and
subsequent emotional or psychological problems as an adult. The Iirst stage is called the oral stage
because the erogenous zone is the mouth. Fixation can occur in this stage iI the baby is weaned Irom the
mother`s breast too soon or too late. The second stage in Freud`s theory is the anal stage, during which
time period the anus serves as the erogenous zone and the conIlict centers around toilet training. The third
stage is the phallic stage and Iocuses on the child`s own genitals. During this stage the child develops a
sexual attraction to the opposite-sex parent, becomes jealous oI the same-sex parent, develops anxiety due
to the attraction and the jealousy, and resolves the anxiety through sexual repression and identiIication
with the same-sex parent. Freud reIerred to this process in boys as the Oedipus complex and suggested
that girls go through a similar process with their Iathers as the target oI their aIIection. The process oI
identification leads to the development oI the superego so that by the end oI Freud`s third stage oI
development, all three components oI personality are in place. The Iourth stage, known as the latency
stage, consists oI repressed sexual Ieelings during which children Iocus on intellectual, physical, and
social development but not sexual development. The Iinal stage occurs around the start oI puberty when
sexual Ieelings can no longer be repressed and is reIerred to as the genital stage.
A number oI psychologists, reIerred to as neo-Freudians, agreed with parts oI Freud`s theories
but not all aspects. Carl Gustav Jung believed that there were two parts oI the unconscious, a personal
unconscious similar to the unconscious described by Freud and a collective unconscious, which
contained universal human memories that Jung called archetypes. AlIred Adler Ielt that the motivating
Iactor oI behavior was not the pleasure-seeking drive oI the libido suggested by Freud, but rather the
seeking oI superiority through deIense mechanisms such as compensation. Karen Horney disagreed with
Freud`s emphasis on sexuality and thought personalities were shaped more by a child`s sense oI basic
anxiety, which iI unattended to could lead to the development oI neurotic personalities. Erik Erikson
developed eight psychosocial stages oI development that Iocused on the role oI social relationships in the
development oI personality.
Although Freud`s theory has had a signiIicant impact on the culture oI modern Western societies,
his theory has been criticized on the scientiIic grounds due to the Iact that it was not developed based on
scientiIic experiments but rather on Freud`s personal observations in his private practice as a psychiatrist,
and that Freud`s personal observations were limited to a speciIic group oI wealthy Austrian women living
in the sexually repressed Victorian era.
According to the behaviorists` perspective, personality consists oI a set oI learned responses or
habits. A variation on the behaviorist perspective is that oI the social cognitive learning theorists, who
emphasize the role oI conditioning along with an individual`s thought processes in the development oI
personality. A strong proponent oI the social cognitive view, Albert Bandura, suggested that the
environment, behavior, and personal/cognitive Iactors all act together to determine an individual`s actions
in a process Bandura reIerred to as reciprocal determinism. An important component oI the cognitive
Iactors is the person`s sense oI self-efficacy, or perception oI how eIIective a behavior will be in a
particular context. Julian Rotter proposed that individuals develop a relatively set way oI responding and
this behavior represented 'personality. An important determinant oI the individual`s response was his or
her sense oI locus of control. According to Rotter, the individual`s expectancy and the response`s
reinIorcement value were the two key Iactors that determined how an individual would react.
The humanistic perspective oI personality Iocuses more on qualities that are considered
uniquely human such as Iree will and subjective emotions. Carl Rogers proposed that humans are always
striving to IulIill their innate capacities in a process known as the self-actualizing tendency. Rogers
deIined positive regard as warmth, aIIection, love, and respect that comes Irom signiIicant others. In
order Ior an individual to work toward selI-actualization, they need to be exposed to a certain level oI
unconditional positive regard Irom the signiIicant others in their lives. Rogers Ielt that conditional
positive regard would restrict a person`s ability to become a Iully Iunctioning person. Rogers believed an
individual`s image oI oneselI, or self-concept, also played a role in becoming fully functional. The selI-
concept was based on what an individual is told by others and also his or her own sense oI self. According
to Rogers, selI-concept could be divided into a real self and an ideal self. II the real selI and ideal selI
concept were too Iar apart, anxiety and neurotic behavior would result.
Theories of Personality CHAPTER 13 -170-
Trait theories oI personality have Iocused on describing personality and predicting behavior
based on that description. A trait is a consistent, enduring way oI thinking, Ieeling, or behaving. Gordon
Allport identiIied approximately 200 traits in the English language that he Ielt were 'wired into each
person`s nervous system. Raymond Cattell narrowed the number oI traits down Iurther by dividing traits
into surface traits, such as the 200 traits described by Allport and source traits, or the more basic traits
that underlie the surIace traits and Iorm the core oI personality. Introversion is an example oI a source
trait. Cattell identiIied 16 basic or source traits. Later researchers narrowed this list to Iive source traits
and developed the personality model known as the five-factor model, or the Big Five. The Iive trait
dimensions are openness, conscientiousness, extraversion, agreeableness, and neuroticism. Critics oI
the Iive-Iactor model have argued that the situation plays a more signiIicant role in determining an
individual`s behavior than is suggested by trait theory and have proposed a theory that includes a trait-
situation interaction.
The Iield oI behavioral genetics studies the role oI inherited traits in personality. Twin studies
have Iound that identical twins are more similar than Iraternal twins or unrelated people in certain aspects
oI personality such as intelligence, leadership, tendency to Iollow rules, assertiveness, and aggressiveness.
Adoption studies have supported some oI these Iindings and have suggested a biological basis Ior shyness
and aggressiveness.
In an attempt to describe 'national personalities, Geert HoIstede conducted a cross-cultural
study Ior IBM that resulted in a description oI each country along Iour basic dimensions. The dimensions
HoIstede observed were individualism/collectivism, power distance, masculinity/Iemininity, and
uncertainty avoidance.
Methods Ior assessing personality have been developed based on speciIic theories oI personality as well
as the various goals oI classiIication, selI-insight, and the diagnosis oI psychological disorders. An
interview is a method oI personality assessment in which the proIessional asks questions oI the client and
allows the client to answer in either a structured or unstructured manner. Interviews are limited by the Iact
that clients can lie, intentionally or unintentionally, and the interviewers can bring their own biases into
their interpretations including the halo effect, which is the tendency oI a person`s Iirst impression to
inIluence later assessments. Psychoanalysts have developed projective tests in an attempt to assess a
person`s unconscious conIlicts or desires by having them projected onto an ambiguous visual stimulus.
Two oI the most commonly used projective tests are the Rorschach inkblot test and the Thematic
Apperception Test, or TAT. Projective tests are highly subjective and have been Iound to have low
reliability and validity. A behaviorist would be more likely to measure personality by directly observing
an individual`s actions. In direct observation, the psychologist would observe an individual in a speciIic
setting and record his or her behaviors through the use oI a rating scale or a frequency count. Critics oI
this approach have pointed out the possibility Ior both the observer eIIect and observer bias. Trait
theorists would be most likely to use a personality inventory, which consists oI a questionnaire that has
a standard list oI questions that require speciIic answers such as 'yes or 'no. Examples oI commonly
used personality inventories include Cattell`s 16 PF, the Neuroticism/Extraversion/Openness Personality
Inventory (NEO-PI), the Myers-Briggs Type Indicator (MPTI), and the Minnesota Multiphasic
Personality Inventory, Version II (MMPI-2). The advantage oI personality inventories is that they are
scored objectively, which eliminates the possibility oI observer bias, and they have been Iound to have
high reliability and validity scores. However, the inventories are still based on selI-report. Questions may
be interpreted in diIIerent ways by diIIerent individuals and are likely to be subject to cultural inIluences.
A large number oI personality tests are accessible over the Internet; however, the results oI such tests
should be interpreted with an appropriate level oI skepticism.

Theories of Personality CHAPTER 13 -171-
STUDY HINTS
24.

Students oIten conIuse the levels oI awareness suggested by Freud with his three components oI
personality. The next two exercises should help you keep them straight. To start with let`s think
about your levels oI awareness. For each oI the levels listed, list at least three examples oI the
inIormation or memories that would be Iound there. Start with the conscious level.


My conscious level oI awareness might contain the Iollowing:






My preconscious level oI awareness might contain the Iollowing:








My unconscious level oI awareness might contain the Iollowing:








Theories of Personality CHAPTER 13 -172-

25.

Now think about the three components that Freud suggested make up an individual`s personality:
the id, the ego, and the superego. For each oI the situations listed below, describe how a person`s
id, ego, and superego might respond. The Iirst example has been completed Ior you. Notice how
the ego always represents the compromise between the two extremes.
Situation Id Ego Superego

Someone cuts you
oII in traIIic as you
are driving down the
Ireeway.
Speed up, cut in front
of them, and then
slow wav down.
Ill vell a few words
at the driver from mv
own car but remain
driving at the speed
limit.
Its wrong to break
the law, and we dont
know what is
happening with that
personmavbe thev
have an emergencv.

Your alarm goes oII
Ior school but you
still Ieel completely
exhausted.



Your co-worker asks
you to work her shiIt
Ior you so that she
can have the night oII
to go to a concert.




Your roommate just
made a batch oI
chocolate chip
cookies and said he is
going to take most oI
them to work with
him tomorrow.



You just Iinished
watching two hours
oI TV and still have a
lot oI homework to
do Ior tomorrow but
you don`t Ieel like
doing it.




Theories of Personality CHAPTER 13 -173-
LEARNING OBJECTIVES
12.1 What is personalitv, and how do the various
perspectives in psvchologv view
personalitv?
12.2 How did Freuds historical view of the mind
and personalitv form a basis for
psvchodvnamic theorv?
12.3 How did Jung, Adler, Hornev, and Erikson
modifv Freuds theorv?
12.4 How does modern psvchoanalvtic theorv
differ from that of Freud?
12.5 How do behaviorist and social cognitive
theorists explain personalitv?

12.6 How do humanists such as Carl Rogers
explain personalitv?
12.7 What are the historv and current views of the
trait perspective?
12.8 What part do biologv, hereditv, and culture
plav in personalitv?
12.9 What are the advantages and disadvantages
of the following measures of personalitv.
interviews, profective tests, behavioral
assessment, personalitv inventories, and
online personalitv tests?


PRACTICE EXAM
For the Iollowing multiple choice questions, select the answer you Ieel best answers the question.

1. The unique way in which each individual thinks, acts, and Ieels throughout liIe is called
a) character.
b) personality.
c) temperament.
d) the unconscious.

2. One limitation oI the trait perspective compared to the other perspectives is there is not much
a) description.
b) research.
c) material.
d) explanation.

3. Many have compared Freud's idea oI the mind to an iceberg. II that were the case and you were
standing on the deck oI a ship in Alaska, what part oI the mind would you see above the water?
a) ego
b) superego
c) id
d) preconscious

4. InIormation that cannot be recalled even when a person makes a determined eIIort to retrieve it
would be said by Freud to be residing in the
a) conscious.
b) preconscious.
c) unconscious.
d) superego.

5. In Sigmund Freud's theory, the operates according to the pleasure principle.
a) id
b) ego
c) thanatos
d) superego

Theories of Personality CHAPTER 13 -174-

6. According to Freud, the last component oI an individual`s personality to develop is the
a) ego.
b) superego.
c) id.
d) libido.

7. What is Freud's term Ior the executive oI the personality that has a realistic plan Ior obtaining
gratiIication oI an individual's desires?
a) id
b) ego
c) superego
d) preconscious

8. Freud called the developmental stage in which the Oedipus complex occurs the
a) oral stage.
b) anal stage.
c) phallic stage.
d) latency stage.

9. Freud believed that the personality characteristics oI overeating, gum chewing, being too dependent
or overly optimistic developed due to Iixation during the
a) oral stage.
b) anal stage.
c) phallic stage.
d) latency stage.

10. Which neo-Freudian viewed personality disturbances as resulting Irom the Ieelings oI inIeriority all
people share?
a) Carl Jung
b) AlIred Adler
c) Carl Rogers
d) Karen Horney

11. Karen Horney disagreed with Freud about the unconscious Iorce that inIluences behavior. She
believed the Iorce was not sexual desire, but rather
a) Ieelings oI inIeriority.
b) basic anxiety.
c) the collective unconscious.
d) selI-regard.

12. Which oI the Iollowing is not a current criticism oI Freud`s psychoanalytic theory?
a) the signiIicant impact it has had on culture
b) the lack oI empirical evidence
c) observations based on Freud`s personal clients
d) role oI women in Freud`s theory

Theories of Personality CHAPTER 13 -175-

13. Albert Bandura's notion that people are aIIected by their environment but can also inIluence that
environment is known as
a) selI-eIIicacy.
b) locus oI control.
c) phenomenology.
d) reciprocal determinism.

14. A baseball player's son is quite talented; he has received lots oI awards over the years. When he gets
up to bat he expects to get a hit, and when he is in the Iield he expects to make every catch.
According to Bandura, what characteristic does this young man seem to have?
a) selI-regard
b) selI-centeredness
c) selI-eIIicacy
d) selI-actualization

15. theory is called the third Iorce in personality theory.
a) Psychoanalytic.
b) Behaviorist
c) Cognitive
d) Humanistic

16. In Carl Rogers's theory, our perception oI our abilities, behaviors, and characteristics is known as
a) personality.
b) selI-regard.
c) selI-esteem.
d) selI-concept.

17. Which oI the Iollowing represents an example oI unconditional positive regard?
a) a mother telling her son that she hopes he becomes an engineer like his Iather
b) a Iather telling his daughter that he will really only be proud oI her iI she gets all As like
she did last semester
c) an owner only pays attention to her dog when he is well-behaved
d) a parent telling his son he loves him even though he just wrecked the Iamily car

18. What did Gordon Allport think about traits?
a) He thought they were like stages.
b) He thought they were wired into the nervous system.
c) He thought they were learned.
d) He thought they were the result oI cognitive modeling.

19. How many source traits did Raymond Cattell discover through the process oI Iactor analysis?
a) 5
b) 16
c) 200
d) 4,500

Theories of Personality CHAPTER 13 -176-

20. What psychoanalytic theorist most notably inIluenced the Big Five theory oI personality?
a) Freud
b) Jung
c) Erikson
d) Horney

21. The Iact that an outgoing extravert might be very talkative at a party but very quiet at a Iuneral is an
example oI
a) trait-situation interaction.
b) cross-cultural similarities.
c) source trait reliability.
d) neuroticism.

22. What major conclusion about personality traits emerged Irom the Minnesota twin study?
a) Identical twins are more similar than any other type oI sibling.
b) Siblings reared apart were much more similar than identical twins.
c) Fraternal twins reared together were much more similar than identical twins.
d) Personality scores Ior twins were not related in either case.

23. Which oI the Iollowing countries would not be considered a collectivist country according to the
studies by Geert HoIstede?
a) Japan
b) United States
c) Mexico
d) Korea

24. Which oI the Iollowing terms describes the cultural personality oI the United States according to
HoIstede's dimensions oI cultural personality?
a) individualistic
b) high in power distance
c) low in individualism
d) high in uncertainty avoidance

25. Which oI the Iollowing is considered an advantage in the use oI interviews Ior personality
assessment?
a) halo eIIect
b) answers are based on selI-report
c) bias oI the interviewer
d) natural Ilow oI the questions

26. Which personality test relies on the interpretation oI inkblots to understand personality?
a) MMPI
b) 16PF
c) TAT
d) Rorschach

27. Which oI the Iollowing is not a criticism oI projective tests?
a) They are a projection oI the person`s unconscious concern.
b) They are low in reliability.
c) Their interpretation is more an art than a science.
d) They lack validity.
Theories of Personality CHAPTER 13 -177-
28. The most commonly used personality inventory is the
a) MMPI-2.
b) MBTI.
c) TAT.
d) CPI.

29. Which oI the Iollowing is an advantage to using personality inventories?
a) observer bias
b) their standardization
c) biases oI interpretation
d) their reliance on selI-report

30. Darla tries to save money by bringing her lunch to work. However, on Iour out oI Iive work days
last week, she threw away her sandwich Irom home and instead went out to lunch with her work
colleagues. To justiIy the added expense oI the restraunt meals, Darla tells herselI that the time
spent "networking" with her colleagues will help to improve her chances Ior promotion at work.
Which oI the Iollowing psychological deIense mechanisms best describes Darla's behavior?
a) Projection
b) Displacement
c) Rationalization
d) Repression

PRACTICE EXAM ANSWERS
1. b Temperament and character are both part oI personality. Character reIers to value
judgments made about a person`s morals, and temperament reIers to the enduring
characteristics that a person is born with.
2. a Trait theories are descriptive and deal with the actual end result oI personality.
3. a The ego is the part oI the mind that is conscious and in view.
4. c Freud thought that inIormation sometimes seeped out oI the unconscious through
our dreams or slips oI the tongue, but Ior the most part, the inIormation was not
readily available to our conscious awareness.
5. a According to Freud, the id represents the most basic part oI the personality and
operates on the pleasure principle. The ego operates on the reality principle.
6. b Freud`s theory states that the superego develops during the phallic stage or when an
individual is 56 years old.
7. b The ego is in charge oI reality and decisions and the superego is there Ior moral
judgments, but the ego makes the decisions.
8. c The Oedipus complex leads to the development oI the superego and occurs during
the phallic stage.
9. a Freud described those personality traits as resulting Irom Iixation during the oral
stage oI development.
10. b Adler viewed personality disturbances as resulting Irom the Ieelings oI inIeriority
all people share. Jung Iocused on archetypes in the collective unconscious.
11. b Horney believed that basic anxiety was the unconscious driving Iorce behind many
oI the behaviors people exhibited.
12. a The impact oI Freud`s theory on culture is not considered a criticism.
13. d SelI-eIIicacy reIers to one's perception oI how eIIective a behavior will be in any
particular circumstance, whereas reciprocal determinism is Bandura's notion that
people are aIIected by their environment but can also inIluence that environment.
14. c SelI-eIIicacy reIers to one's perception oI how eIIective a behavior will be in any
particular circumstance. SelI-actualization has to do with selI-IulIillment and
reaching one's Iull potential.
Theories of Personality CHAPTER 13 -178-
15. d Humanistic theory is called the third Iorce in personality theory; the Iirst two are
psychoanalytic theory and behaviorist theory.
16. d SelI-esteem has more to do with one's sense oI worth.
17. d Rogers deIined unconditional positive regard as being love, aIIection, and respect
with no strings attached.
18. b Allport thought traits were not learned, but rather were wired into the nervous system.
19. b Cattell proposed that there were 16 source traits oI personality.
20. b Freud's views are not involved in trait theory, but Jung's theory mentioned
extroversion, which is one oI the Big Five traits.
21. a The trait-situation interaction Iocuses on the interaction oI source traits with the
speciIic environment or situation that a person is in.
22. a Identical twins, who share the same genes, are more similar in personality than are
any other type oI siblings.
23. b The HoIstede study Iound that the United States could be described as more oI an
individualistic culture.
24. a Americans expect power to be well-distributed rather than held by an elite Iew;
democracies are typically low in power distance.
25. d The natural Ilow oI the interview process is one oI the advantages oI this method.
26. d The Rorschach is a projective test that relies on the use oI inkblot interpretation.
27. a The reason a psychologist would use a projective test is to get a 'projection oI that
individual`s unconscious concerns.
28. a The MMPI-2 is used more than any other inventory.
29. b The Iact that personality inventories are standardized represents one oI the greatest
advantages to using this assessment technique.
30. c The psychological deIense mechanism oI rationalization is deIined as making up
acceptable excuses Ior unacceptable behavior.

CHAPTER GLOSSARY
agreeableness the emotional style oI a person which may range Irom easy-going,
Iriendly, and likeable to grumpy, crabby, and unpleasant.
anal stage second stage occurring Irom about 1 or 1.5 years oI age, in which the
anus is the erogenous zone and toilet training is the source oI conIlict.
archetypes Jung`s collective, universal human memories.
basic anxiety anxiety created when a child is born into the bigger and more powerIul
world oI older children and adults.
behavioral genetics Iield oI study devoted to discovering the genetic bases Ior personality
characteristics.
character value judgments oI a person`s moral and ethical behavior.
collective unconscious Jung`s name Ior the memories shared by all members oI the human
species.
compensation
(substitution)
deIense mechanism in which a person makes up Ior inIeriorities in one
area by becoming superior in another area.
conditional positive regard positive regard that is given only when the person is doing what the
providers oI positive regard wish.
conscience Part oI the superego that produces guilt, depending on how acceptable
behavior is.
conscientiousness the care a person gives to organization and thoughtIulness oI others,
dependability.
denial psychological deIense mechanism in which a person reIuses to
acknowledge or recognize a threatening situation.
direct observation assessment in which the proIessional observes the client engaged in
ordinary, day-to-day behavior in either a clinical or natural setting.
Theories of Personality CHAPTER 13 -179-
displacement redirecting Ieelings Irom one threatening target to a less threatening one.
ego part oI the personality that develops out oI a need to deal with reality,
mostly conscious, rational and logical.
expectancy a person`s subjective Ieeling that a particular behavior will lead to a
reinIorcing consequence.
extraversion dimension oI personality reIerring to one`s need to be with other people.
extraverts people who are outgoing and sociable.
five-factor model (Big Five) model oI personality traits that describes Iive basic trait dimensions.
fixation disorder in which the person does not Iully resolve the conIlict in a
particular psychosexual stage, resulting in personality traits and behavior
associated with that earlier stage.
frequency count assessment in which the Irequency oI a particular behavior is counted.
fully functioning person a person who is in touch with and trusting oI the deepest, innermost urges
and Ieelings.
habits in behaviorism, sets oI well-learned responses that have become
automatic.
halo effect tendency oI an interviewer to allow positive characteristics oI a client to
inIluence the assessments oI the client`s behavior and statements.
humanistic perspective the 'third Iorce in psychology that Iocuses on those aspects oI
personality that make people uniquely human, such as subjective Ieelings
and Ireedom oI choice.
id part oI the personality present at birth and completely unconscious.
ideal self one`s perception oI who one should be or would like to be.
identification deIense mechanism in which a person tries to become like someone else
to deal with anxiety.
interview method oI personality assessment in which the proIessional asks
questions oI the client and allows the client to answer, either in a
structured or unstructured Iashion.
introversion dimension oI personality in which people tend to withdraw Irom
excessive stimulation.
introverts people who preIer solitude and dislike being the center oI attention.
latency Iourth stage occurring during the school years, in which the sexual
Ieelings oI the child are repressed while the child develops in other ways.
locus of control the tendency Ior people to assume that they either have control or do not
have control over events and consequences in their lives.
neo-Freudians Iollowers oI Freud who developed their own, competing psychodynamic
theories.
neurotic personalities personalities typiIied by maladaptive ways oI dealing with relationships
in Horney`s theory.
neuroticism degree oI emotional instability or stability.
Oedipus complex/Electra
complex
situation occurring in the phallic stage in which a child develops a sexual
attraction to the opposite-sex parent and jealousy oI the same sex-parent.
Males develop an Oedipus complex and Iemales develop an Electra
complex.
openness one oI the Iive Iactors, willingness to try new things and be open to new
experiences.
oral stage Iirst stage occurring in the Iirst year oI liIe, and in which the mouth is the
erogenous zone and weaning is the primary conIlict.
personal unconscious Jung`s name Ior the unconscious mind as described by Freud.
personality the unique and relatively stable ways in which people think, Ieel, and
behave.
Theories of Personality CHAPTER 13 -180-
personality inventory paper and pencil or computerized test that consists oI statements that
require a speciIic, standardized response Irom the person taking the test.
phallic stage third stage occurring Irom about 3 to 6 years oI age, in which the child
discovers sexual Ieelings.
pleasure principle principle by which the id Iunctions; the immediate satisIaction oI needs
without regard Ior the consequences.
positive regard warmth, aIIection, love, and respect that come Irom signiIicant others in
one`s liIe.
projection psychological deIense mechanism in which unacceptable or threatening
impulses or Ieelings are seen as originating with someone else, usually
the target oI the impulse or Ieelings. Also deIined as a deIense
mechanism involving placing, or 'projecting, one`s own unacceptable
thoughts onto others, as iI the thoughts actually belonged to those others
and not to oneselI.
projective tests personality assessments that present ambiguous visual stimuli to the client
and ask the client to respond with whatever comes to mind.
psychoanalysis Freud`s term Ior both the theory oI personality and the therapy based on
it.
psychological defense
mechanisms
unconscious distortions oI a person`s perception oI reality that reduce
stress and anxiety.
psychosexual stages Iive stages oI personality development proposed by Freud and tied to the
sexual development oI the child.
rating scale assessment in which a numerical value is assigned to speciIic behavior
that is listed in the scale.
reaction formation psychological deIense mechanism in which a person Iorms an opposite
emotional or behavioral reaction to the way he or she really Ieels to keep
those true Ieelings hidden Irom selI and others.
real self one`s perception oI actual characteristics, traits, and abilities.
reality principle principle by which the ego Iunctions; the satisIaction oI the demands oI
the id only when negative consequences will not result.
reciprocal determinism Bandura`s explanation oI how the Iactors oI environment, personal
characteristics, and behavior can interact to determine Iuture behavior.
regression psychological deIense mechanism in which a person Ialls back on
childlike patterns oI responding in reaction to stressIul situations.
repression psychological deIense mechanism in which the person reIuses to
consciously remember a threatening or unacceptable event, instead
pushing those events into the unconscious mind.
Rorschach inkblot test projective test that uses 10 inkblots as the ambiguous stimuli.
self an individual`s awareness oI his or her own personal characteristics and
level oI Iunctioning.
self-actualizing tendency the striving to IulIill one`s innate capacities and capabilities.
self-concept the image oI oneselI that develops Irom interactions with important,
signiIicant people in one`s liIe.
self-efficacy individual`s perception oI how eIIective his or her eIIorts to accomplish a
goal will be in any particular circumstance.
social cognitive learning
theorists
theorists who emphasize the importance oI both the inIluences oI other
people`s behavior and oI a person`s own expectancies oI learning.
social cognitive view learning theory that includes cognitive processes such as anticipating,
judging, memory, and imitation oI models.
source traits the more basic traits that underlie the surIace traits, Iorming the core oI
personality.
Theories of Personality CHAPTER 13 -181-
Theories of Personality CHAPTER 13 -182-

subjective reIerring to concepts and impressions that are only valid within a
particular person`s perception and may be inIluenced by biases, prejudice,
and personal experiences.
sublimation channeling socially unacceptable impulses and urges into socially
acceptable behavior.
superego part oI the personality that acts as a moral center.
surface traits aspects oI personality that can easily be seen by other people in the
outward actions oI a person.
temperament the enduring characteristics with which each person is born.
Thematic Apperception
Test (TAT)
projective test that uses twenty pictures oI people in ambiguous situations
as the visual stimuli.
trait a consistent, enduring way oI thinking, Ieeling, or behaving.
trait theories theories that endeavor to describe the characteristics that make up human
personality in an eIIort to predict Iuture behavior.
trait-situation interaction the assumption that the particular circumstances oI any given situation
will inIluence the way in which a trait is expressed.
unconditional positive
regard
positive regard that is given without conditions or strings attached.
unconscious mind level oI the mind in which thoughts, Ieelings, memories, and other
inIormation are kept that are not easily or voluntarily brought into
consciousness.

CHAPTER 14 - PSYCHOLOGICAL DISORDERS


YOU KNOW YOU ARE READY FOR THE TEST IF YOU ARE ABLE TO.
DeIine abnormality and brieIly discuss the historical and cultural impact on deIining
psychological disorders.
Present the biological and psychological models oI psychopathology.
Discuss the diagnosis and prevalence rates oI psychological disorders in the United States.
Describe speciIic categories oI psychological disorders including anxiety disorders, mood
disorders, eating disorders, dissociative disorders, schizophrenia, and personality disorders.

RAPID REVIEW
The study oI abnormal behavior, or psychopathology, can be traced to at least as early as 3000
B.C. Irom evidence oI trepanning, or the drilling oI holes in the skull. Today, abnormal behavior is
considered to be any behavior that is rare, deviates Irom the social norm within the situational context,
causes subjective discomfort, or is maladaptive. Psychological disorders are deIined as a pattern oI
behavior that causes people signiIicant distress, causes them to harm themselves or others, or interIeres
with their ability to Iunction in daily liIe. In the sociocultural perspective oI abnormality, abnormal
behavior (as well as normal behavior) is seen as the product oI behavioral shaping within the context oI
Iamily inIluences, the social group to which one belongs, and the culture within which the Iamily and
social group exist. This is an important issue Ior psychological proIessionals who are attempting to assess
and treat members oI a culture diIIerent Irom their own. Cultural relativity reIers to the need to consider
the unique characteristics oI the culture in which the person with a disorder was nurtured to be able to
correctly diagnose and treat the disorder. For example, a recent research study by Mejia and McCarthy
revealed that college students oI Mexican heritage with migrant Iarming backgrounds reported more
symptoms oI anxiety and depression as compared to nonmigrant college students oI Mexican heritage,
suggesting that the nature oI migrant Iarming poses stressors diIIerent Irom those Iaced by nonmigrant
Iamilies. Culture-bound syndromes are certain psychological disorders that are only Iound in particular
cultures. For example, anorexia nervosa and bulimia nervosa are most oIten Iound in Western societies.
Three perspectives on abnormality are highlighted in the chapter. First, the biological model oI
psychopathology proposes that psychological disorders are caused by biological changes in the chemical,
structural, or genetic systems oI the body. Second, the psychological models propose that disordered
behavior is the result oI various Iorms oI emotional, behavioral, or thought-related malIunctioning. This
perspective includes cognitive, behavioral and psychodynamic theories. Cognitive psychologists study
the way that people think, remember, and mentally organize inIormation. These psychologists have
proposed the cognitive model oI psychopathology, which describes psychological disorders as resulting
Irom Iaulty thinking patterns. Third, the biopsychosocial model proposes that abnormal behavior is the
result oI the combined and interacting Iorces oI biological, psychological, social, and cultural inIluences.
Currently in the United States, psychological disorders are assessed by reIerring to the
Diagnostics and Statistical Manual of Mental Disorders, Jersion 4, Text Revision (DSM-IV-TR), which
provides inIormation Ior about 250 diIIerent disorders, including common symptoms, prevalence rates,
and criteria Ior diagnosis. The individual is assessed in Iive diIIerent categories, or axes. Axis 1 contains
all the psychological disorders except personality disorders. Axis II includes personality disorders and
mental retardation. Axis III includes an assessment oI any physical disorders that aIIect a person
psychologically. Axis IV consists oI problems in a person`s environment that may be aIIecting his or her
psychological Iunctioning, and Axis V is an assessment oI a person`s overall (or global) level oI
Iunctioning ranging Irom 0 to 100. In a given year, about 26 percent oI adults over 18 years oI age in the
United States could be diagnosed with a mental disorder.
Labeling psychological disorders provides a common language Ior the mental health community
to use. However, labels can also be dangerous, as shown by researcher David Rosenhan`s classic study in
which healthy participants ('pseudo-patients) were asked to enter psychiatric hospitals and complain that
Psychological Disorders CHAPTER 14 -183-
they were hearing voices. All oI the pseudo-patients were admitted to the hospitals and diagnosed with
either schizophrenia or manic depression. Once admitted, the pseudo-patients stopped pretending to be ill
and acted as they normally would, but the hospital staII`s interpretation oI this normal behavior was
skewed by the label oI mental illness. The labels stuck, even when actual symptoms oI mental illness
disappeared. Rosenhan concluded that psychological labels are long lasting and powerIul, aIIecting not
only how other people see mental patients but also how patients see themselves.
Anxiety disorders include all disorders characterized by excessive or unrealistic anxiety. Free-
floating anxiety is the term given to anxiety that seems to be unrelated to any realistic, known Iactor.
Phobias are a speciIic Iorm oI anxiety disorder deIined as an irrational and persistent Iear oI something
and include social phobias; specific phobias such as claustrophobia and acrophobia; and
agoraphobia, or Iear oI being in a place that would be diIIicult to escape Irom iI something happened to
go wrong. Panic disorder is characterized by Irequent occurrences oI panic attacks or sudden onsets oI
extreme panic. II a Iear oI having panic attacks prevents an individual Irom going to public places it is
called panic disorder with agoraphobia. Obsessive-compulsive disorder involves a reoccurring
thought (or obsession) that causes extreme anxiety and leads to some repetitive or ritualistic behavior (or
compulsion). Acute stress disorder (ASD) results Irom exposure to a major stressor with symptoms oI
anxiety, dissociation, recurring nightmares, sleep disturbances, problems in concentration, and moments
in which people seem to 'relive the event in dreams and Ilashbacks Ior as long as 1 month Iollowing the
event. Posttraumatic stress disorder (PTSD) also results Irom exposure to a major stressor and has
similar symptoms (anxiety, dissociation, nightmares, poor sleep, reliving the event, and concentration
problems), but diIIers Irom ASD in that it lasts Ior more than 1 month. Individuals diagnosed with
generalized anxiety disorder display excessive anxiety and worries with no real source that can be
pinpointed as leading to the anxiety. Cognitive psychologists believe that anxiety disorders are caused by
illogical thinking that includes maladaptive thinking process such as magnification, all-or-nothing
thinking, overgeneralization, and minimization. Evidence also supports biological Iactors, such as an
imbalance in neurotransmitter levels, particularly serotonin and GABA, as playing a role in anxiety
disorders. Other research suggests a possible deIect in the way serotonin binds to its receptors in the
nervous system. More recently, studies with mice have indicated that an area oI the hippocampus known
as the ventral hippocampus may help control anxiety by communicating with the medial preIrontal cortex,
the area oI the brain important in processing emotional awareness.
Mood disorders, also reIerred to as affective disorders, represent a disturbance in emotion. Two
mild Iorms oI mood disorders include dysthymia, a chronic depression that lasts Ior at least two years or
more, and cyclothymia, a cycle oI sadness and happiness that also persists Ior two or more years. The
most common mood disorder is major depression, which is characterized by prolonged Ieelings oI
extreme sadness. Bipolar disorder involves all the symptoms oI major depression in addition to brieI
periods oI extreme mania, or excessive excitement, energy, and Ieelings oI happiness. Learning theorists
attribute depression to learned helplessness. In the social cognitive view, depressed people continually
have negative, selI-deIeating thoughts about themselves, which depress them Iurther in a downward spiral
oI despair. A recent study by Strunk and colleagues revealed that when therapists Iocus on helping clients
to change their way oI thinking, depression improves signiIicantly when compared to therapy that Iocuses
only on changing behavior. Biological explanations have Iocused on the role oI brain chemicals such as
serotonin, norepinephrine, and dopamine. Some people Iind that they only get depressed at certain times
oI the year, particularly during the winter months. These individuals may be experiencing seasonal
affective disorder (SAD), which is a mood disorder that is caused by the body`s reaction to low levels oI
light present in the winter months
The two primary types oI eating disorders are anorexia and bulimia. Anorexia nervosa, oIten
called anorexia, is a condition in which a person reduces eating to the point that a weight loss oI 15
percent below expected body weight or more is the result. At a weight loss oI 40 percent below expected
body weight, hospitalization is necessary. Compared to other demographic groups, individuals who are
young and Iemales are most likely to develop anorexia. The causes oI anorexia are not Iully understood
but are thought to involve biological issues and/or psychological Iactors such as sexual abuse, Iamily
dysIunction, and perIectionism with a desire to control as many aspects oI one`s liIe as possible. Bulimia
Psychological Disorders CHAPTER 14 -184-
nervosa, oIten called bulimia, is a condition in which a person develops a cycle oI 'binging or
overeating enormous amounts oI Iood at one sitting, and then using inappropriate methods Ior avoiding
weight gain. Some oI these methods include 'purging behaviors such as deliberate vomiting aIter the
binge or misuse oI laxatives. Other methods include Iasting the day or two aIter the binge or engaging in
excessive exercise. Bulimia is similar to anorexia in that the victims are usually Iemale, are obsessed with
their appearance, diet excessively, and believe themselves to be Iat even though they are not. Bulimia
diIIers Irom anorexia in that victims are typically a little older than those with anorexia at the onset oI the
disorder (early 20s rather than early puberty), and they oIten maintain a normal weight, making the
disorder more diIIicult to detect. The most obvious diIIerence between the two disorders is that bulimic
individuals will eat; in Iact, they will binge to excess. In a typical binge, a bulimic may consume 3,500 to
50,000 calories in one sitting.
Dissociative disorders involve a break, or dissociation, in a person`s sense oI identity. In
dissociative amnesia, an individual cannot remember inIormation contained in long-term memory such
as her own name or where she lives. A dissociative fugue occurs when a person suddenly travels away
Irom his home and aIterwards cannot remember the trip or even his own identity. In dissociative identity
disorder, Iormerly reIerred to as multiple personality disorder, a person seems to experience at least two
or more distinct personalities. Within the psychological perspective, behaviorists believe that 'not
thinking about certain events can be negatively reinIorced by reducing anxiety and unpleasant Ieelings,
while cognitive psychologists Iocus on the Ieelings oI guilt, shame, or anxiety that may be avoided
through 'thought avoidance. Biological explanations Ior dissociative disorders also exist. Researchers
have Iound that individuals with depersonalization disorder also have lower brain activity in areas oI
the brain responsible Ior our sense oI body awareness.
Schizophrenia is a severe psychotic disorder in which the person is not able to distinguish
Iantasy Irom reality and experiences disturbances in thinking, emotions, behavior, and perception. Many
people with schizophrenia experience delusions (Ialse belieIs about the world), hallucinations (seeing or
hearing things that are not really there), and flat affect (the display oI little or no emotion). II an
individual experiences delusions alone they would more likely be diagnosed with a type oI delusional
disorder. Schizophrenia can be divided into Iive basic categories: disorganized, characterized by
conIused speech along with Irequent and vivid hallucinations; catatonic, in which the individual may sit
without moving Ior hours or may move about wildly; paranoid, identiIied by hallucinations and
delusions; undiIIerentiated, in which the individual does not Iit in one oI the three categories already
mentioned; and residual, in which a person is in a state oI recovery Irom the symptoms oI schizophrenia.
Schizophrenia can also be classiIied according to the kind oI symptoms displayed. Positive symptoms
reIlect an excess or distortion oI normal Iunctions, such as hallucinations, whereas negative symptoms
reIlect a decrease oI normal Iunctions. Medication appears to be more eIIective in treating the positive
symptoms oI schizophrenia. The causes oI schizophrenia have been attempted to be explained with the
biological model. Increased levels oI dopamine and brain structural deIects are currently the two
explanations with the strongest support. In addition, the stress-vulnerability model proposes that
individuals may have a biological sensitivity that is then made worse by environmental stress. Recent
imaging research has indicated that two areas oI the brain, the cingulum bundle (CB, consisting oI Iibers
underlying the cingulate gyrus linking parts oI the limbic system) and the uncinate Iasciculus (UF, neural
Iibers linking the Irontal lobe to the temporal lobe), have signiIicantly less myelin coating on their axons.
This makes these areas oI the brain less eIIicient in sending neural messages to other cells, resulting in
decreased memory and decision-making ability. The CB may be associated with attention problems in
schizophrenia; whereas lower white matter integrity in the areas oI the Irontal lobe might be associated
with genetic predisposition to schizophrenia.
Disorders that aIIect a person`s entire liIe adjustment are reIerred to as personality disorders.
The DSM-IJ-TR recognizes ten diIIerent personality disorders. An individual with antisocial personality
disorder typically Ieels no remorse and oIten behaves in an impulsive manner with no regard Ior the
consequences. Borderline personality disorder is deIined by moody, unstable behaviors in which the
individual lacks a clear sense oI identity. As an explanation Ior personality disorders, cognitive and
learning theorists Iocus on how speciIic behaviors are learned and reinIorced over time.
Psychological Disorders CHAPTER 14 -185-
Although it is not yet recognized as a clinical disorder in the DSM-IJ-TR, test anxiety is a
problem Ior countless students. Test anxiety is the personal experience oI possible negative consequences
or poor outcomes on an exam or evaluation accompanied by a cluster oI cognitive, aIIective, and
behavioral symptoms. To decrease the impact oI test anxiety, one should try to Iind some internal
motivation to do well on the exam rather than simply relying on extrinsic reasons. Next, one should
develop some type oI strategy Ior controlling cognitive state and behavior beIore and during the exam.
Recent research suggests that competence-priming, or imagining a person who is successIul at a related
task, lowers the relationship between test anxiety and test perIormance. Finally, instead oI Iocusing on the
whole exam, take control and address one question at a time, Iirst answering the questions you know.

STUDY HINTS
26.

Six diIIerent categories oI psychological disorders are presented in this chapter. In order to help
organize the new terms, try creating a table oI the diIIerent disorders that includes a general
description oI each category and the speciIic disorders within the category. The Iirst category has
been completed Ior you as an example.
Disorder Type General Description Specific Examples

Anxiety disorders

a psvchological disorder in
which the main svmptom is
an intense fear or anxietv
social phobias,
specific phobias,
agoraphobia,
obsessive-compulsive
disorder
generali:ed anxietv disorder,
panic disorder

Dissociative disorders


Mood disorders


Schizophrenia


Personality disorders


Psychological Disorders CHAPTER 14 -186-


27.


In addition to understanding the disorders themselves, it is important to understand the diIIerent
theories as to the causes oI each disorder. Your textbook discusses three theories oI abnormality:
the biological, psychological (including cognitive, behavioral and psychodynamic perspectives),
and biopsychosocial models. In order to enhance your understanding oI these models, brieIly
describe how each oI them would explain the disorders listed below.

Model Depression Schizophrenia
Dissociative Identity
Disorder

Biological


Psychological

Biopsychosocial

Suggested answers for Question 2

Model Depression Schizophrenia
Dissociative Identity
Disorder
Biological brain chemical
imbalance (in
neurotransmitters
such as serotonin
and dopamine)
chemical imbalance
and brain structure
abnormalities
variation in brain
activitv between
different
'personalities`

Psychological
negative and self-
defeating thoughts
severe form of
illogical thinking
thought avoidance
Biopsychosocial
genetic susceptibilitv
made worse bv a
stressful environment
unstable familv
environment triggers
biological sensitivitv
traumatic childhood
event causes changes
in neural activitv
Psychological Disorders CHAPTER 14 -187-


LEARNING OBJECTIVES
14.1 How has mental illness been explained in
the past, how is abnormal behavior defined
todav, and what is the impact of cultural
differences in defining abnormalitv?
14.2 How can psvchological disorders be
explained within the biological and
psvchological models?
14.3 What are the different tvpes of
psvchological disorders, and how common
are thev?
14.4 What are the different tvpes of anxietv
disorders, their svmptoms, and causes?

14.5 What are the different kinds of mood
disorders and their causes?
14.6 What are the two primarv tvpes of eating
disorders, how do thev differ, and who are
thev most likelv to affect?
14.7 What are the main svmptoms, tvpes, and
causes of schi:ophrenia?
14.8 How do the various personalitv disorders
differ, and what is thought to be the cause of
personalitv disorders?
14.9 What are some of the future directions in
psvchopathologv?


PRACTICE EXAM
For the Iollowing multiple choice questions, select the answer you Ieel best answers the question.

1. It is probably accurate to assume that in ancient times signs oI mental illness were believed to be
caused by
a) imbalance oI body Iluids.
b) demons.
c) improper diet.
d) social Iorces.

2. What is the primary diIIiculty with applying the criterion oI "social norm deviance" to deIine
abnormal behavior?
a) Norms are diIIicult to enumerate.
b) Cultures accept and view all behaviors as normal.
c) Behavior that is considered disordered in one culture may be acceptable in another.
d) Norms do not guide behavior except in rare instances.

3. Which oI the Iollowing is not a criterion used to decide whether a pattern oI behavior should be
considered a psychological disorder?
a) The behavior is physically exhausting.
b) The behavior causes subjective distress.
c) The behavior goes against the norms oI the society.
d) The behavior is maladaptive.

4. The biological model views psychological disorders as resulting
Irom
a) distorted thought patterns.
b) repressed memories.
c) underlying behavioral issues.
d) physiological causes.

Psychological Disorders CHAPTER 14 -188-

5. Jennie is in her early twenties. Lately, when she looks in the mirror, she sees an obese woman. In
reality Jennie is actually oI normal weight. Sometimes Jennie eats large quantities oI Iood in one
sitting and then makes herselI vomit in order to avoid gaining weight. Taken together, Jennie`s
pattern oI symptoms suggests that she may have
a) schizophrenia.
b) bulimia nervosa.
c) anorexia nervosa.
d) agoraphobia.

6. Alan went to see a psychologist to get some help overcoming his anxiety in public. The psychologist
spent a lot oI time discussing the speciIic thoughts Alan has when he is in public and trying to help
him change those thought patterns. The psychologist could be best described as adhering to the
a) psychological perspective.
b) biological model.
c) psychoanalytical perspective.
d) sociocultural model.

7. Anorexia and bulimia may be considered , as they are most oIten Iound in Western
societies.
a) restricted syndromes
b) naturalistic syndromes
c) sociocultural disorders
d) culture-bound syndromes

8. is used to help psychological proIessionals diagnose psychological disorders.
a) The Diagnostic and Statistical Manual of Mental Disorders
b) The Phvsicians Desk Reference
c) The Textbook of Psvchological Disorders
d) The Textbook of Phvsiological Disorders

9. When a psychologist or psychiatrist is using the DSM-IJ-TR as a guide to evaluating a client, he or
she would assess the client on each oI Iive
a) axes.
b) stages.
c) phases.
d) steps.

10. In any given year in the United States, approximately how many adults over age 18 experience a
mental disorder?
a) 5 percent
b) 26 percent
c) 52 percent
d) 76 percent

11. Which oI the Iollowing statements is true about anxiety?
a) It is never considered realistic or normal.
b) Some anxiety is realistic when its source is obvious and understandable.
c) It always maniIests itselI as a disorder.
d) It is unusual Ior a mentally healthy person to experience anxiety.

Psychological Disorders CHAPTER 14 -189-

12. Over the past Iew years, Sam has become extremely IearIul oI going to any public place such as a
restaurant, concert, or even the grocery store. On many days, Sam does not even leave his house Ior
Iear that he might be caught somewhere that would not be easy to escape Irom. With which anxiety
disorder would Sam most likely be diagnosed?
a) a speciIic phobia
b) obsessive-compulsive disorder
c) generalized anxiety disorder
d) agoraphobia

13. Liza has an anxiety disorder. She is currently seeing a therapist who believes that anxiety disorders
are a result oI illogical, irrational thought processes. Liza is probably seeking treatment Irom a
a) behavioral psychologist.
b) cognitive psychologist.
c) psychoanalyst.
d) psychologist with a biological perspective.

14. The cognitive process oI magniIication could be described as
a) interpreting a single negative event as a never-ending pattern oI deIeat.
b) making mountains out oI molehills.
c) giving little or no emphasis to one`s successes or positive events.
d) throwing the baby out with the bath water.

15. Disorders characterized by a break in conscious awareness, memory, the sense oI identity, or some
combination are called
a) paraphilias.
b) anxiety disorders.
c) somatoIorm disorders.
d) dissociative disorders.

16. Dissociative identity disorder is a psychological disorder more commonly known as
a) amnesia.
b) Iugue or Ilight disorder.
c) schizophrenia.
d) multiple personality disorder.

17. Which oI the Iollowing perspectives claims that shaping may play a big role in the development oI
some cases oI dissociative identity disorder?
a) psychological
b) humanistic
c) biological
d) psychoanalytic

18. Disorders characterized by disturbances in emotion are known as disorders.
a) conversion
b) somatoIorm
c) mood
d) dissociative

Psychological Disorders CHAPTER 14 -190-

19. An individual diagnosed with dysthymia would most likely exhibit which oI the Iollowing
symptoms?
a) cycles oI being sad then happy then sad
b) mild depression over a period oI several years
c) severe depression that appears very rapidly without any apparent reason
d) periods oI excessive excitement Iollowed by days or weeks oI severe depression

20. Which oI the Iollowing is the biological explanation Ior mood disorders?
a) They are a result oI learned helplessness.
b) They are a result oI anger turned inward on oneselI.
c) They are a result oI distortions in thinking.
d) They are a result oI an imbalance oI brain chemicals.

21. A person suIIering Irom disordered thinking, bizarre behavior, and hallucinations, who is unable to
distinguish between Iantasy and reality, is likely suIIering Irom
a) schizophrenia.
b) bipolar disorder.
c) a dissociative disorder.
d) passive-aggressive personality.

22. The condition in which a person shows little or no emotion is reIerred to as
a) Ilat aIIect.
b) hallucinations.
c) delusions.
d) disorganization.

23. The primary Ieature oI schizophrenia is severe disturbance oI motor behavior.
a) disorganized
b) catatonic
c) residual
d) paranoid

24. Which oI the Iollowing symptoms would not be considered a negative symptom oI schizophrenia?
a) lack oI aIIect
b) poor attention
c) social withdrawal
d) hallucinations

25. Sal has decreased levels oI the neurotransmitter dopamine in his preIrontal cortex. Which disorder
might he be at risk oI experiencing?
a) antisocial personality disorder
b) agoraphobia
c) schizophrenia
d) dissociative Iugue

26. Disorders that aIIect the entire liIe adjustment oI a person are reIerred to as
a) somatoIorm disorders.
b) dissociative disorders.
c) mood disorders.
d) personality disorders.

Psychological Disorders CHAPTER 14 -191-
27. A person with antisocial personality disorder would be likely to engage in which oI the Iollowing
behaviors?
a) lying to other people without worrying about the consequences
b) display excessive and inappropriate emotions
c) report hallucinations
d) completely withdraw Irom society

28. Which oI the Iollowing statements represents the biological view oI personality disorders?
a) They are due to an inadequate resolution oI the Oedipus complex.
b) They are a type oI learned behavior.
c) They have physiological causes.
d) They are due to disturbances in Iamily relationships.

29. is a mood disorder that is caused by the body's reaction to low levels oI light present in
the winter months.
a) Panic disorder
b) Bipolar disorder
c) Dysthymic disorder
d) Seasonal aIIective disorder

PRACTICE EXAM ANSWERS
1. b People oI ancient times perceived signs oI mental illness as caused by demons.
Hippocrates, a Greek physician, viewed the imbalance oI body Iluids as the cause oI
mental illness, but Hippocrates` time period is not considered 'ancient times.
2. c Behavior that is considered disordered in one culture may be acceptable in another.
Most people do allow social norms to guide much oI their behavior.
3. a The three main criteria Ior a behavior to be considered a psychological disorder are
that it deviates Irom social norms, is maladaptive, and causes the individual
personal distress or discomIort.
4. d The biological model emphasizes physiological or physical causes Ior
psychological disorders. The other three choices represent the psychological models
oI cognitive, psychoanalytical, and behavioral.
5. b Bulimia nervosa is similar to anorexia in that victims see themselves as being
overweight when they are not. However, bulimia diIIers Irom anorexia in that it
typically begins in the victim`s early twenties rather than during puberty. Also,
bulimics eat and then purge; whereas, anorexics avoid eating altogether.
6. a Within the psychological model, cognitive psychologists tend to treat disorders by
attempting to change the person`s thought patterns.
7. d Anorexia and bulimia may be considered culture-bound syndromes, as they tend to
occur primarily in Western cultures.
8. a The DSM helps psychological proIessionals diagnose psychological disorders,
while the Phvsicians Desk Reference is used by medical proIessionals to diagnose
physiological problems.
9. a The DSM-IV-TR uses a system oI Iive diIIerent axes Ior evaluations.
10. b According to recent studies, approximately 26 percent oI the U.S. adult population
experiences a mental disorder in a given year.
11. b This statement is true because some types oI anxiety are normal.
12. d Agoraphobia is an anxiety disorder characterized by an extreme Iear oI going in
public places that would be diIIicult to escape Irom iI necessary.
13. b Cognitive psychologists view anxiety disorders as a result oI distorted thought
processes.
Psychological Disorders CHAPTER 14 -192-

14. b MagniIication is the tendency to interpret a situation as being Iar more harmIul,
dangerous, or embarrassing than it actually is, or in other words, making a big deal
out oI something that is actually very small.
15. d Dissociative disorders are characterized by a break in conscious awareness,
memory, the sense oI identity, or some combination.
16. d Multiple personality disorder is a term no longer used by psychologists but is still
very common in the general public.
17. a Within the psychological model oI abnormality, behavioral psychologists
emphasize shaping through positive and negative reinIorcement as a Iactor in the
development oI some cases oI dissociate identity disorder.
18. c Mood disorders are characterized by disturbances in emotion, while somatoIorm
disorders take the Iorm oI bodily ailments that have no physical cause
19. b Dysthymia can be thought oI as a mild version oI depression, while cyclothymia
more closely resembles a mild version oI bipolar disorder.
20. d The biological explanation emphasizes an imbalance oI brain chemicals.
21. a Disordered thinking, bizarre behavior, hallucinations, and inability to distinguish
between Iantasy and reality are all symptoms oI schizophrenia. Bipolar disorder is
characterized by mood swings between depression and mania and does not involve
hallucinations or inability to distinguish between Iantasy and reality.
22. a The word affect is used to mean emotion or mood.
23. b Severe motor disturbance is a Ieature oI catatonic schizophrenia. Symptoms oI
residual schizophrenia include negative belieIs, poor language skills, unusual ideas
and perceptions.
24. d Negative symptoms oI schizophrenia reIlect a decrease in normal Iunction (such as
lack oI social interactions or displays oI emotions). Hallucinations represent an
excess or addition in normal Iunction and would be classiIied as a positive symptom
oI schizophrenia.
25. c Schizophrenia is associated with an imbalance oI dopamine.
26. d Personality disorders do not just aIIect a single aspect oI a person`s liIe but rather
aIIect the person`s entire liIe adjustment.
27. a Antisocial personality disorder is characterized by an individual who acts 'against
society. For example, an individual might commit a crime without Ieeling any
remorse.
28. c The biological perspective Iocuses on physiological causes Ior psychological
disorders, and cognitive-learning theorists do believe that the behavior displayed by
people with personality disorders is learned through reinIorcement, shaping, and
modeling.
29. d Seasonal aIIective disorder occurs primarily during the winter months. Dysthymic
disorder consists oI similar symptoms but is not seasonal in nature.

CHAPTER GLOSSARY
acrophobia Iear oI heights.
acute stress disorder (ASD) a disorder resulting Irom exposure to a major stressor with symptoms
oI anxiety, dissociation, recurring nightmares, sleep disturbances,
problems in concentration, and moments in which people seem to
'relive the event in dreams and Ilashbacks Ior as long as 1 month
Iollowing the event.
affect in psychology, a term indicating emotion or mood.
agoraphobia Iear oI being in a place or situation Irom which escape is diIIicult or
impossible.
Psychological Disorders CHAPTER 14 -193-

all-or-nothing thinking the tendency to believe that one`s perIormance must be perIect or the
result will be a total Iailure.
anorexia nervosa (anorexia) a condition in which a person reduces eating to the point that a weight
loss oI 15 percent below the ideal body weight or more occurs.
antisocial personality disorder disorder in which a person has no morals or conscience and oIten
behaves in an impulsive manner without regard Ior the consequences
oI that behavior.
anxiety disorders disorders in which the main symptom is excessive or unrealistic
anxiety and IearIulness.
biological model model oI explaining behavior as caused by biological changes in the
chemical, structural, or genetic systems oI the body.
biopsychosocial model perspective in which abnormal behavior is seen as the result oI the
combined and interacting Iorces oI biological, psychological, social,
and cultural inIluences.
bipolar disorder severe mood swings between major depressive episodes and manic
episodes.
borderline personality
disorder
maladaptive personality pattern in which the person is moody,
unstable, lacks a clear sense oI identity, and oIten clings to others.
bulimia nervosa (bulimia) a condition in which a person develops a cycle oI 'binging or
overeating enormous amounts oI Iood at one sitting, and then using
unhealthy methods to avoid weight gain.
catatonic type oI schizophrenia in which the person experiences periods oI
statue-like immobility mixed with occasional bursts oI energetic,
Irantic movement and talking.
claustrophobia Iear oI being in a small enclosed space.
cognitive psychologists psychologists who study the way people think, remember and
mentally organize inIormation.
cultural relativity the need to consider the unique characteristics oI the culture in which
behavior takes place.
culture-bound syndromes disorders Iound only in particular cultures.
delusional disorder a psychotic disorder in which the primary symptom is one or more
delusions.
delusions Ialse belieIs held by a person who reIuses to accept evidence oI their
Ialseness.
depersonalization disorder dissociative disorder in which individuals Ieel detached and
disconnected Irom themselves, their bodies, and their surroundings.
disorganized type oI schizophrenia in which behavior is bizarre and childish and
thinking, speech, and motor actions are very disordered.
dissociative amnesia loss oI memory Ior personal inIormation, either partial or complete.
dissociative disorders disorders in which there is a break in conscious awareness, memory,
the sense oI identity, or some combination.
dissociative fugue traveling away Irom Iamiliar surroundings with amnesia Ior the trip
and possible amnesia Ior personal inIormation.
dissociative identity disorder disorder occurring when a person seems to have two or more distinct
personalities within one body.
flat affect a lack oI emotional responsiveness.
free-floating anxiety anxiety that is unrelated to any realistic, known source.
Psychological Disorders CHAPTER 14 -194-

generalized anxiety disorder disorder in which a person has Ieelings oI dread and impending doom
along with physical symptoms oI stress, and which lasts six months or
more.
hallucinations Ialse sensory perceptions, such as hearing voices that do not really
exist.
magnification the tendency to interpret situations as Iar more dangerous, harmIul, or
important than they actually are.
major depression severe depression that comes on suddenly and seems to have no
external cause, or is too severe Ior current circumstances.
maladaptive anything that does not allow a person to Iunction within or adapt to
the stresses and everyday demands on liIe.
manic having the quality oI excessive excitement, energy, and elation or
irritability.
minimization the tendency to give little or no importance to one`s successes or
positive events and traits.
mood disorders disorders in which mood is severely disturbed.
negative symptoms symptoms oI schizophrenia that are less than normal behavior or an
absence oI normal behavior; poor attention, Ilat aIIect, and poor
speech production.
obsessive-compulsive disorder disorder in which intruding, recurring thoughts or obsessions create
anxiety that is relieved by perIorming a repetitive, ritualistic behavior
or mental act (compulsion).
overgeneralization the tendency to interpret a single negative event as a never-ending
pattern oI deIeat and Iailure.
panic attack sudden onset oI intense panic in which multiple physical symptoms oI
stress occur, oIten with Ieelings that one is dying.
panic disorder disorder in which panic attacks occur Irequently enough to cause the
person diIIiculty in adjusting to daily liIe.
panic disorder with
agoraphobia
Iear oI leaving one`s Iamiliar surroundings because one might have a
panic attack in public.
paranoid type oI schizophrenia in which the person suIIers Irom delusions oI
persecution, grandeur, and jealousy, together with hallucinations.
personality disorders disorders in which a person adopts a persistent, rigid, and maladaptive
pattern oI behavior that interIeres with normal social interactions.
phobia an irrational, persistent Iear oI an object, situation, or social activity.
positive symptoms symptoms oI schizophrenia that are excesses oI behavior or occur in
addition to normal behavior; hallucinations, delusions, and distorted
thinking.
posttraumatic stress disorder
(PTSD)
A disorder resulting Irom exposure to a major stressor, with
symptoms oI anxiety, dissociation, nightmares, poor sleep, reliving
the event, and concentration problems lasting Ior more than 1 month.
psychological disorders any pattern oI behavior that causes people signiIicant distress, causes
them to harm others, or harms their ability to Iunction in daily liIe.
psychopathology the study oI abnormal behavior.
psychotic term applied to a person who is no longer able to distinguish what is
real and what is Iantasy.
schizophrenia severe disorder in which the person suIIers Irom disordered thinking,
bizarre behavior, hallucinations, and is unable to distinguish between
Iantasy and reality.
Psychological Disorders CHAPTER 14 -195-
Psychological Disorders CHAPTER 14 -196-

seasonal affective disorder
(SAD)
a mood disorder caused by the body`s reaction to low levels oI
sunlight in the winter months.
situational context the social or environmental setting oI a person`s behavior.
social phobia Iear oI interacting with others or being in social situations that might
lead to a negative evaluation.
sociocultural perspective perspective in which abnormal behavior (as well as normal behavior)
is seen as the product oI the learning and shaping oI behavior within
the context oI the Iamily, the social group to which one belongs, and
the culture within which the Iamily and social group exist.
specific phobias Iear oI objects or speciIic situations or events.
stress-vulnerability model explanation oI disorder that assumes a biological sensitivity, or
vulnerability, to a certain disorder will result in the development oI
that disorder under the right conditions oI environmental or emotional
stress.
subjective discomfort emotional distress or emotional pain.


CHAPTER 15 - PSYCHOLOGICAL THERAPIES


YOU KNOW YOU ARE READY FOR THE TEST IF YOU ARE ABLE TO.
DeIine two main types oI modern treatment Ior psychological disorders and brieIly discuss the
history oI treatment oI the mentally ill.
Introduce the major types oI psychotherapy including psychoanalysis, humanistic, behavior,
cognitive, and group therapy.
Discuss the assessment and eIIectiveness oI the psychotherapy treatments.
Describe the biomedical approaches oI treating psychological disorders including the use oI
drugs, electroconvulsive therapy, and psychosurgery.

RAPID REVIEW
Therapy Ior psychological disorders consists oI treatment methods aimed at making people Ieel
better and Iunction more eIIectively. The two modern ways in which psychological disorders can be
treated are psychotherapy, which consists oI talking things out with a proIessional and biomedical
therapy, which consists oI using biological methods such as medication to treat a psychological disorder.
Psychotherapy techniques can be roughly divided into insight therapies, which have the goal oI selI-
understanding and action therapies, which Iocus on changing an individual`s behaviors. Biomedical
therapies consist mainly oI the use oI drugs, surgical techniques, or electroconvulsive therapy. It is
important to note that biomedical therapy oIten eliminates or alleviates the symptoms oI a disorder while
psychotherapy addresses issues associated with the disorder. Recent research suggests that, when used
together, these two types oI therapy Iacilitate each other. Early treatment oI the mentally ill oIten
consisted oI Iatal attempts to 'rid the individual oI the physical impurities causing the abnormal
behavior. It was not until 1793 that Philippe Pinel began the movement oI humane treatment oI the
mentally ill.
Psychoanalysis is an insight therapy developed by Sigmund Freud with the goal oI revealing the
unconscious conIlicts, urges, and desires that Freud assumed were the cause oI the psychological
disorder. Freud utilized a number oI techniques in his attempt to reveal the unconscious. Dream
interpretation involved an analysis oI the actual or manifest content oI a dream as well as the hidden or
latent content. Freud Ielt the latent content oI dreams could reveal unconscious conIlict. In addition,
Freud used free association, or allowing the patients to Ireely say whatever came to their mind, to
uncover the repressed material; resistance, in which the patient became unwilling to discuss a topic any
Iurther; and transference, in which the therapist became a symbol oI a parental authority Iigure. Today,
psychoanalytic therapy is oIten reIerred to as psychodynamic therapy and is directive, places more
emphasis on transIerence, and is usually much shorter than traditional psychoanalysis. Individuals with
anxiety, somatoIorm, or dissociative disorders are more likely to beneIit Irom psychodynamic therapy
than individuals with other types oI disorders. Interpersonal psychotherapy (IPT) is a psychotherapy
developed to address depression. It is an insight therapy Iocusing on the client`s relationships and the
interplay between mood and the events oI everyday liIe.
Humanistic therapy is also an insight therapy, but unlike psychoanalysis, humanistic therapy
Iocuses on conscious experiences oI emotion and an individual`s sense oI selI. The two most common
humanistic therapies are person-centered therapy and Gestalt therapy. Carl Rogers developed person-
centered therapy, which has the goal oI helping an individual get his or her real and ideal selves to more
closely match up. According to Rogers, the role oI the therapist is to provide the unconditional positive
regard that was missing in the individual`s liIe. He Ielt the therapy should be nondirective with the
individual doing most oI the work and believed the Iour key elements oI reflection, unconditional
positive regard, empathy, and authenticity were crucial Ior a successIul person-therapist relationship.
Motivational interviewing (MI) is a variation oI person-centered therapy that has speciIic goals to reduce
the ambivalence about change and to increase intrinsic motivation to make the change happen. The Iour
goals oI a therapist administering MI therapy are as Iollows: express empathy, develop discrepancy
Psychological Therapies CHAPTER 15 -197-
between the client`s present behaviors and values, roll with resistance, and support the client`s selI-
eIIicacy. Fritz Perls believed that people`s problems arose Irom hiding important parts oI their Ieelings
Irom themselves and developed another humanistic therapy called Gestalt therapy, a directive Iorm oI
insight therapy. Gestalt therapy Iocuses on the client`s Ieelings and subjective experiences and uses
leading questions and planned experiences such as role-playing to help the person reveal the Ieelings they
may be hiding Irom themselves. Humanistic therapies have been Iound to be more successIul with
individuals who are able to express their thoughts and Ieelings in a logical manner and are not necessarily
the best choice Ior individuals with more severe psychological disorders.
Behavior therapies use action-based therapy to change behavior based on basic principles oI
classical and operant conditioning. The abnormal behavior is not seen as a symptom, but rather seen as
the problem itselI. Behavior modification or applied behavior analysis reIers to the use oI conditioning
techniques to modiIy behavior. Behavior therapies that rely on classical conditioning include systematic
desensitization, aversion therapy, and Ilooding. Systematic desensitization consists oI a three-step
process that utilizes counter-conditioning in order to reduce Iear and anxiety. First the client learns deep
muscle relaxation techniques, then the client creates a list oI anxiety-producing events called a hierarchy
oI Iear, and Iinally the client conIronts the anxiety-producing event while remaining in a relaxed state.
Aversion therapy uses classical conditioning to decrease a behavior by pairing an aversive (unpleasant)
stimulus with the stimulus that normally produces the unwanted behavior. Behavioral techniques that
introduce the client to situations, under careIully controlled conditions, which are related to their anxieties
or Iears are called exposure therapies. Flooding involves rapid and intense exposure to an anxiety-
producing object in order to produce extinction oI the conditioned Iear response.
Francine Shapiro developed a therapy technique called eye-movement desensitization reprocessing
(EMDR) in which clients attempt to decrease their Iears, anxieties, and disturbing thoughts by moving
their eyes rapidly back and Iorth. Although EMDR has been a popular treatment Ior post-traumatic stress
disorder, the eIIectiveness oI the treatment has yet to be Iirmly established and has not been Iound to be
any more eIIective than other more traditional techniques such as simple muscle relaxation or exposure
therapy.
Behavior therapies that utilize operant conditioning include participant modeling, token
economies, contingency contracts, and extinction techniques such as the use oI a time-out. Participant
modeling has been used to successIully treat phobias and obsessive-compulsive disorders by having the
client watch and mimic a model demonstrating the desired behaviors. In a token economy, clients are
reinforced with tokens Ior behaving correctly and can later exchange the tokens Ior things they want such
as Iood, candy, or special privileges. A contingency contract is a written statement oI speciIic required
behaviors, contingent penalties, and subsequent rewards. Extinction techniques such as time-outs work
by removing the reinIorcement Ior a behavior. In adults, simply reIusing to acknowledge a person`s
behavior is oIten successIul in reducing the Irequency oI that behavior. Behavior therapies have been
eIIective in the treatment oI disorders including overeating, drug addictions, and phobias.
Cognitive therapy is an action therapy that Iocuses on helping people change the distorted
thinking and unrealistic belieIs that lead to maladaptive behaviors. Common distortions in thought
include arbitrary inference (or 'jumping to conclusions), selective thinking, overgeneralization,
magnification and minimization, and personalization. Cognitive behavioral therapy (CBT) is a type
oI cognitive therapy in which the goal is to help clients overcome problems by learning to think more
rationally and logically. Albert Ellis developed a version oI CBT called rational-emotive behavioral
therapy in which clients are taught to replace their own irrational belieIs with more rational, helpIul
statements. Cognitive therapies have considerable success in treating disorders such as depression, stress
disorders, anxiety disorders, and some types oI schizophrenia.
An alternative to individual therapy is group therapy, in which a group oI clients with similar
problems gather together and discuss their problems under the guidance oI a single therapist. Types oI
group therapies include family counseling and self-help (or support) groups. The advantages oI group
therapy are the lower cost, exposure to the ways other people handle the same kinds oI problems, the
opportunity Ior the therapist to see how that person interacts with others, and the social and emotional
support Irom the people in the group. Research indicates that an extremely shy person may initially have
Psychological Therapies CHAPTER 15 -198-
great diIIiculty speaking up in a group setting but cognitivebehavioral group therapy can be eIIective Ior
social phobia. The disadvantages are that the person may not Ieel as Iree to reveal embarrassing or
personal inIormation, the therapist`s time must be shared during the session, and people with severe
disorders such as schizophrenia may not tolerate a group setting. Group therapy seems to be most
successIul as a long-term treatment intended to promote the development oI skilled social interactions.
The eIIectiveness oI the various psychotherapy techniques is diIIicult to determine due to various
time Irames required Ior the diIIerent therapies, alternate explanations oI 'eIIectiveness, the lack oI
adequate control groups, experimenter bias, and the inaccuracies oI selI-report inIormation. Most
psychological proIessionals today take an eclectic approach to psychotherapy, which involves using a
combination oI methods to Iit the particular client`s needs. The most important aspect oI successIul
psychotherapy appears to be the relationship between the client and the therapist, also reIerred to as the
therapeutic alliance. Therapy should also oIIer clients a protected setting in which to release emotions
and reveal private thoughts and concerns. Other common Iactors in therapy eIIectiveness are opportunity
Ior catharsis (relieving pent-up emotions), learning and practice oI new behaviors, and positive
experiences Ior the client. DiIIerences in culture between the therapist and the client can make it diIIicult
Ior the therapist to understand the exact nature oI the client`s problems. Several studies have Iound that
members oI minority racial or ethnic groups drop out oI therapy at signiIicantly higher rates than the
majority group clients. Barriers to eIIective psychotherapy include diIIerence in language, culture-bound
values, class-bound values, and nonverbal communication. A new Iorm oI therapy that is delivered via
the Internet, called cybertherapy, is now available.
Biomedical therapies directly aIIect the biological Iunction oI the body and include the three
categories oI drug therapy, shock therapy, and surgical treatments. Psychopharmacology reIers to the
use oI drugs to control or relieve the symptoms oI a psychological disorder and is oIten combined with
psychotherapy Ior a more eIIective outcome. Psychopharmacological drugs can be divided according to
the disorders they treat including drugs Ior psychotic disorders, anxiety disorders, manic symptoms oI
mood disorders, and depression. Drugs used to treat psychotic symptoms such as hallucinations,
delusions, and bizarre behaviors are called antipsychotic drugs and include typical neuroleptics, atypical
neuroleptics, and partial dopamine agonists. In general, these drugs work to decrease dopamine levels in
the brain. The newer drugs tend to have Iewer negative side eIIects than the older typical neuroleptics.
The two kinds oI drugs currently used to treat anxiety disorders include the traditional antianxiety drugs
such as the minor tranquilizers, or benzodiazepines, including Xanax, Ativan, and Valium and
antidepressant drugs to be discussed in more detail shortly. The most common treatment Ior the manic
symptoms oI bipolar disorder is the antimanic drug oI lithium. The exact mechanism oI lithium is still not
clearly understood. Antidepressant drugs can be divided into three separate categories: the monamine
oxidase inhibitors (MAOIs) such as Marplan and Nardil; tricyclic antidepressants such as ToIranil and
Elavil; and the selective serotonin reuptake inhibitors (SSRIs) such as Prozac and ZoloIt.
Electroconvulsive therapy (ECT), also known as shock therapy, is still in use today to treat severe cases
oI depression, schizophrenia, and mania. The treatment involves delivery oI an electric shock to one or
both sides oI a person`s head, causing a release oI neurotransmitters and almost immediate improvement
in the individual`s mood. One oI the main side eIIects oI ECT is at least a short-term loss oI memory.
Psychosurgery involves operating on an individual`s brain to remove or destroy brain tissue Ior the
purpose oI relieving symptoms oI psychological disorders. One oI the earliest psychosurgery techniques
is the prefrontal lobotomy, which is no longer perIormed today. The main psychosurgery technique in
use today is the bilateral cingulotomy which destroys the cingulated gyrus and has been shown to be
eIIective in about one-third oI cases oI depression, bipolar disorder, and obsessive-compulsive disorder.
This procedure is only perIormed with the patient`s Iull and inIormed consent aIter all other treatment
options have been exhausted.
New noninvasive techniques Ior eIIecting changes in the brain include repetitive transcranial
magnetic stimulation (rTMS), where magnetic pulses are applied to the cortex, and transcranial direct
current stimulation (tCDS), which uses scalp electrodes to pass very low amplitude direct currents to the
brain. These techniques are being evaluated as possible treatment options Ior PTSD and depression. Deep
Psychological Therapies CHAPTER 15 -199-
brain stimulation is another technique that is being investigated as a possible treatment Ior both
depression and OCD.

STUDY HINTS
28.

An important task in this chapter is to understand the diIIerences among the multiple types oI
therapy. Listed below are several oI the psychotherapies discussed in the chapter. For each
therapy, indicate the type oI therapy (insight or action), the role oI the therapist (directive or
nondirective), the school oI thought most likely to use this technique, and the overall goal oI the
therapy. The Iirst psychotherapy has been Iilled in as an example.

Therapy
Type of
Therapy
Role of
Therapist
School of
Thought
Goal of
Therapy
Traditional
psychoanalysis
Insight Nondirective Psvchoanalvsis
Uncover
unconscious
conflicts
Person-centered
therapy

Gestalt therapy

Rational-
emotive
behavioral
therapy (REBT)

Systematic
desensitization












Psychological Therapies CHAPTER 15 -200-
Suggested Answers for Question 1

Therapy
Type of
therapy
Role of
therapist
School of
thought
Goal of therapy
Traditional
Psychoanalysis
Insight Nondirective Psvchoanalvsis
Uncover
unconscious
conflicts
Person-centered
therapy
Insight Nondirective Humanistic
Bring ideal self
and real self
into congruence
Gestalt therapy Insight Directive Humanistic
Increase self-
awareness
Rational-
emotive
behavioral
therapy (REBT)
Action Directive
Cognitive-
Behavioral
Replace
irrational beliefs
with more
rational, helpful
statements
Systematic
desensitization
Action Directive Behaviorist
Reduce fear and
anxietv

Which oI the therapies listed above would you Iind most helpIul?

Why?





Psychological Therapies CHAPTER 15 -201-

29.

Rational-emotive behavioral therapy is commonly used Ior individuals with depression and
anxiety. The therapy is based on the idea that an individual has adopted irrational belieIs that
have in turn led to their condition oI anxiety and depression. The goal oI the therapy is to
identiIy the irrational belieIs and teach the individual how to respond with more rational thought
processes. In order to better understand the process, assume you are a therapist using the REBT
technique and your client makes the Iollowing irrational statements. List a suggestion Ior a
rational belieI the client could adopt instead. The Iirst one has already been completed.

Irrational BelieI Rational BelieI
a. I must be loved, or at least liked, and
approved by every signiIicant person I
meet.
I want to be loved or liked bv some of the
people in mv life, and I know I mav feel
disappointed or lonelv when that doesnt
happen, but I can cope with those feelings.
b. I must be completely competent, make
no mistakes, and achieve in every
possible way iI I am to be worthwhile.

c. It is dreadIul, nearly the end oI the
world, when things aren`t how I would
like them to be.

d. Human unhappiness, including mine, is
caused by Iactors outside oI my control,
so little can be done about it.

e. II something might be dangerous,
unpleasant, or Irightening, I should
worry about it a great deal.

I. My problem(s) were caused by event(s)
in my past, and that`s why I have my
problem(s) now.

g. I should be very upset by other people`s
problems and diIIiculties



Psychological Therapies CHAPTER 15 -202-
LEARNING OBJECTIVES
15.1 What are the two modern wavs in which
psvchological disorders can be treated, and
how have thev been treated in the past?
15.2 What were the basic elements of Freuds
psvchoanalvsis, and how does
psvchoanalvsis differ todav?
15.3 What are the basic elements of the
humanistic therapies known as person-
centered therapv and Gestalt therapv?
15.4 How do behavior therapists use classical
and operant conditioning to treat
disordered behavior?
15.5 How successful are behavior therapies?
15.6 What are the goals and basic elements of
cognitive therapies such as cognitive-
behavioral therapv and rational-emotive
behavioral therapv?

15.7 What are the various tvpes of group
therapies and the advantages and
disadvantages of group therapv?
15.8 How effective is psvchotherapv, and how is
the effectiveness of psvchotherapv influenced
bv cultural, ethnic, and gender differences?
15.9 What are the various tvpes of drugs used to
treat psvchological disorders?
15.10 How are electroconvulsive therapv and
psvchosurgerv used to treat psvchological
disorders todav?
15.11 How might computers be used in
psvchotherapv?


PRACTICE EXAM
For the Iollowing multiple choice questions, select the answer you Ieel best answers the question.

1. Therapies directed at changing disordered behavior are reIerred to as
a) action therapies.
b) insight therapies.
c) biomedical therapies.
d) relationship therapies.

2. Which oI the Iollowing is the best example oI biomedical therapy?
a) use oI antidepressants to treat depression
b) use oI insight therapy Ior social phobia
c) psychoanalysis to help treat an anxiety disorder
d) Ilooding treatment Ior an individual with obsessive-compulsive disorder

3. Approximately how long ago were the Iirst eIIorts made to treat the mentally ill with kindness,
rather than subjecting them to harsh physical treatment?
a) 20 years ago
b) 100 years ago
c) 200 years ago
d) 500 years ago

4. Psychoanalysis was a therapy technique designed by
a) AlIred Adler.
b) Carl Rogers.
c) Fritz Perls.
d) Sigmund Freud.

5. Freud believed one oI the indications that he was close to discovering an unconscious conIlict was
when a patient became unwilling to talk about a topic. He reIerred to this response in the patient as
a) transIerence.
b) latent content.
c) dream analysis.
d) resistance.

Psychological Therapies CHAPTER 15 -203-

6. Which oI the Iollowing individuals would be least likely to beneIit Irom psychoanalysis?
a) Mary, who has a somatoIorm disorder
b) Kaleem, who suIIers Irom a severe psychotic disorder
c) Pasha, who has panic attacks
d) Lou, who suIIers Irom anxiety

7. The modern psychoanalyst provides guidance to the patient, asks questions, suggests helpIul
behaviors, and gives opinions and interpretations. This type oI role Ior the therapist is described as a
approach.
a) Iree association
b) directive
c) biomedical
d) nondirective

8. What did Carl Rogers view as a cause oI most personal problems and unhappiness?
a) reinIorcement oI maladaptive behavior patterns
b) unrealistic modes oI thought employed by many people
c) mismatch between an individual's ideal selI and real selI
d) unresolved unconscious conIlicts that occur between the id and superego

9. Which oI the Iollowing was not one oI the Iour key elements Rogers viewed as necessary Ior a
successIul person-therapist relationship?
a) reIlection
b) unconditional positive regard
c) authenticity
d) resistance

10. What is a major goal oI the Gestalt therapist?
a) to Iacilitate transIerence
b) to eliminate the client's undesirable behaviors
c) to provide unconditional positive regard
d) to help clients become more aware oI their own Ieelings

11. Which oI the Iollowing is a limitation oI humanistic therapy?
a) Clients do not need to be verbal.
b) There is not enough empirical research to support its basic ideas.
c) It cannot be used in a variety oI contexts.
d) The therapist runs the risk oI having his or her words misinterpreted by the client.

12. In the aversion therapy technique known as rapid smoking the client takes a puII on a cigarette
every Iive or six seconds so that the nicotine now produces unpleasant responses such as nausea and
dizziness, so that eventually the cigarette itselI produces a sensation oI nausea in the client. In the
terms oI classical conditioning, the cigarette Iunctions as the and the nicotine is the
.
a) UCS; CS
b) CS; UCS
c) CR; UCS
d) CS; UCR

Psychological Therapies CHAPTER 15 -204-

13. Which method oI treating phobias involves progressive relaxation and exposure to the Ieared
object?
a) extinction
b) punishment
c) token economy
d) systematic desensitization

14. In a token economy, what role does the token play in shaping behavior?
a) The tokens are used as punishment to decrease the maladaptive behavior.
b) The tokens are used to reinIorce the desired behavior.
c) The token is the actual behavior itselI.
d) The token represents the written contract between the client and therapist.

15. What is an advantage oI using operant conditioning in treating undesirable behaviors?
a) The results are usually quickly obtained.
b) Clients can get an understanding oI the underlying cause oI the problem.
c) Unconscious urges are revealed.
d) Clients can change distorted thought patterns that aIIect behavior.

16. Which oI the Iollowing is one oI the criticisms oI behavior therapy?
a) It Iocuses on the underlying cause oI behavior and not the symptoms.
b) Therapy typically lasts Ior several years and is very expensive.
c) It Iocuses too much on the past.
d) It only relieves some symptoms oI schizophrenia but does not treat the overall disorder.

17. What is the goal oI cognitive therapy?
a) to help clients gain insight into their unconscious
b) to help people change their ways oI thinking
c) to change a person`s behavior through shaping and reinIorcement
d) to provide unconditional positive regard Ior the client

18. Which oI these clients is the most likely candidate Ior Aaron Beck's Iorm oI cognitive therapy?
a) Albert, who suIIers Irom mania
b) Barbara, who suIIers Irom depression
c) Robert, who suIIers Irom schizophrenia
d) Virginia, who has been diagnosed with dissociative identity disorder

19. Which approach assumes that disorders come Irom illogical, irrational cognitions and that changing
the thinking patterns to more rational, logical ones will relieve the symptoms oI the disorder?
a) cognitive-behavioral
b) person-centered
c) psychoanalytic
d) Gestalt

Psychological Therapies CHAPTER 15 -205-

20. According to Albert Ellis, we become unhappy and depressed about events because oI
a) our behaviors.
b) our irrational belieIs.
c) the events that happen to us.
d) other people's irrational belieIs.

21. Which oI the Iollowing is the best example oI an irrational belieI that a therapist using rational-
emotive behavioral therapy would challenge you to change?
a) It is disappointing when things don`t go my way.
b) II I Iail this test, it will hurt my grade in this class but I will try to make it up on the next
exam.
c) There must be something wrong with Bob since he turned down my invitation Ior a date.
d) Everyone should love and approve oI me and iI they don`t, there must be something
wrong with me.

22. Which oI the Iollowing is an advantage oI cognitive and cognitive-behavioral therapies?
a) Clients do not need to be verbal.
b) They treat the underlying cause oI the problem.
c) They are less expensive and short-term than typical insight therapies.
d) The therapist decides which oI the client's belieIs are rational and which are irrational.

23. An advantage to group therapy is that groups
a) are a source oI social support.
b) allow countertransIerence to occur.
c) provide unconditional approval to the group members.
d) allow antisocial individuals to dominate group discussions.

24. In Iamily therapy, the therapist would most likely
a) Iocus on one individual who has been identiIied as the source oI the problem.
b) have each Iamily member come in Ior therapy individually.
c) provide unconditional approval to all the Iamily members.
d) Iocus on the entire Iamily system to understand the problem.

25. Which oI the Iollowing is not true about selI-help support groups?
a) SelI-help groups do not have leaders.
b) Currently, only a limited number oI selI-help groups operate in the United States.
c) SelI-help groups are typically not directed by a licensed therapist.
d) SelI-help groups are usually Iree to attend.

26. is a controversial Iorm oI therapy in which the client is directed to move the eyes
rapidly back and Iorth while thinking oI a disturbing memory.
a) Eye-movement desensitization reprocessing
b) Systematic desensitization
c) Eye-memory therapy
d) Eye therapy

27. Most psychological proIessionals today take a(n) view oI psychotherapy.
a) group treatment
b) humanistic
c) eclectic
d) behavioral
Psychological Therapies CHAPTER 15 -206-

28. The most important aspect oI a successIul psychotherapy treatment is
a) the length oI the session.
b) the speciIic approach oI the therapist.
c) the relationship between the client and the therapist.
d) the severity oI the disorder.

29. Studies that have examined cultural and ethnic Iactors in the therapeutic relationship have Iound that
a) members oI minority racial or ethnic groups are more likely to continue treatment until
the problem has been resolved.
b) members oI the majority racial or ethnic group usually have lower prevalence rates oI
disorders.
c) members oI minority racial or ethnic groups drop out oI therapy at a higher rate than
members oI the majority group.
d) members oI minority racial or ethnic groups rarely or never seek therapy.

30. Which oI the Iollowing has not been Iound to be a barrier to eIIective psychotherapy when the
cultural backgrounds oI client and therapist are diIIerent?
a) language diIIerences
b) diIIering cultural values
c) nonverbal communication
d) severity oI the disorder

31. Antipsychotic drugs treat symptoms such as
a) hopelessness, sadness, and suicide ideations.
b) excessive worry, repetitive thoughts, and compulsive behavior.
c) hallucinations, delusions, and bizarre behavior.
d) manipulation, lying, and cheating.

32. In what way is the new class oI antidepressants known as the SSRIs an improvement over the older
types oI antidepressants?
a) They work Iaster.
b) They are more eIIective.
c) They target a larger number oI diIIerent neurotransmitters.
d) They have Iewer side eIIects.

33. For which disorder was electroconvulsive therapy originally developed as a treatment?
a) panic
b) schizophrenia
c) bipolar disorder
d) cyclothymia

34. Which oI the Iollowing is the appropriate deIinition oI psychosurgery?
a) inIormation given to a patient about a surgical procedure beIore the surgery in order to
prevent anxiety
b) surgery that is perIormed on brain tissue to relieve or control severe psychological disorders
c) surgery that severs the spinal cord oI the patient
d) a procedure in which a brieI current oI electricity is used to trigger a seizure that
typically lasts one minute, causing the body to convulse

Psychological Therapies CHAPTER 15 -207-
PRACTICE EXAM ANSWERS
1. a Action therapy emphasizes changing behavior, whereas insight therapy emphasizes
understanding one's motives and actions.
2. a Any medical treatment that is directed at changing the physiological Iunctioning oI
an individual is classiIied as a biomedical therapy. All oI the remaining choices are
types oI psychotherapy treatments.
3. c In 1793 Philippe Pinel unchained the mentally ill inmates at an asylum in Paris,
France, and began the movement oI humane treatment Ior the mentally ill.
4. d Freud was the Iounder oI psychoanalysis, while Rogers developed person-centered
therapy.
5. d Resistance occurred when a patient became unwilling to discuss a concept. In
transIerence the patient would transIer positive and negative Ieelings Ior an
authority Iigure in their past onto the therapist.
6. b People with severe psychotic disorders are less likely to beneIit Irom
psychoanalysis than are people who suIIer Irom somatoIorm or anxiety disorders.
7. b A directive approach involves asking questions and suggesting behaviors. The more
traditional psychoanalyst typically takes a more nondirective approach in which the
therapist remains neutral and does not interpret or take direct actions with regard to
the client.
8. c Rogers believed the closer the match between a person's ideal and real selves, the
happier the person. It was Freud, not Rogers, who viewed unresolved unconscious
conIlicts between the id and superego as the cause oI personal problems.
9. d Rogers Ielt a therapist must provide the Iour elements oI reIlection, unconditional
positive regard, empathy, and authenticity in order Ior successIul treatment.
10. d The major goal oI Gestalt therapists is to help clients become more aware oI their
Ieelings. Providing unconditional positive regard is the primary goal oI person-
centered therapy, not Gestalt.
11. b The humanistic therapist does not run the risk oI having his or her words
misinterpreted by the client because the therapist uses reIlection as the main means
oI communication. However, unIortunately at this point there is not enough
empirical evidence to support or reIute the basic ideas oI humanistic therapy.
12. b Both the cigarette and nicotine are stimuli. In rapid smoking, the cigarette serves as
the conditioned stimulus and the nicotine serves as the unconditioned stimulus.
13. d Systematic desensitization involves progressive relaxation and exposure to the
Ieared object, while extinction involves the removal oI a reinIorcer to reduce the
Irequency oI a particular response.
14. b In a token economy, the tokens are the reinIorcers used to shape and strengthen the
desired behaviors.
15. a Operant conditioning is not concerned with the cause oI the problems, rather it is
concerned with changing behavior. However, operant conditioning does provide
rapid change in behavior in comparison to other therapies.
16. d Behavior therapy may help relieve some symptoms but does not treat the overall
disorder oI schizophrenia.
17. b Cognitive therapy Iocuses on changing an individual`s cognitions or thought
processes.
18. b Beck's cognitive therapy is especially eIIective in treating distortions related to
depression.
19. a Cognitive behavioral therapists are concerned with helping clients change their
irrational thoughts to more rational and positive thoughts. A person-centered
therapist believes disorders come Irom a mismatch between the ideal selI and the
real selI and a lack oI unconditional positive regard.
20. b Ellis believes irrational belieIs cause dissatisIaction and depression.
Psychological Therapies CHAPTER 15 -208-
21. d Irrational belieIs typically have one thing in common; they are all-or-none types oI
statements.
22. c Cognitive and cognitive-behavioral therapies are relatively inexpensive and are
short-term.
23. a Group therapy provides social support Ior people who have similar problems.
24. d Family therapy Iocuses on the entire Iamily as a part oI the problem.
25. b Currently an extremely large number oI selI-help groups operate in the United
States.
26. a EMDR is a Iorm oI therapy in which the client is directed to move the eyes rapidly
back and Iorth while thinking oI a disturbing memory. Systematic desensitization
gradually exposes the client to the Ieared object while using relaxation techniques to
reduce anxiety.
27. c An eclectic view is one that combines a number oI diIIerent approaches to best Iit
the needs oI the client.
28. c A number oI studies have Iound that the client-therapist relationship (also called the
therapeutic alliance) is the best predictor oI successIul treatment.
29. c Members oI minority groups are much more likely to drop out oI therapy when
compared to members oI majority racial and ethnic groups.
30. d The severity oI the disorder has not been Iound to be a cultural barrier Ior treatment.
31. c Hallucinations, delusions, and bizarre behaviors are deIined as psychotic behaviors
and are treated with antipsychotic drugs. Antidepressant drugs, not antipsychotic
drugs, treat Ieelings oI hopelessness, sadness, and suicide ideations.
32. d The speed oI action and eIIectiveness is similar among the three classes oI
antidepressants but the main diIIerence is the number oI negative side eIIects. The
SSRIs actually target only one neurotransmitter: serotonin.
33. b ECT was originally designed to induce seizures in schizophrenics.
34. b Severing the spinal cord would lead to the very negative side eIIect oI paralysis oI
the body. Psychosurgery is perIormed on brain tissue.

CHAPTER GLOSSARY
action therapies therapies in which the main goal is to change disordered or inappropriate
behavior directly.
antianxiety drugs drugs used to treat and calm anxiety reactions, typically minor tranquilizers.
antidepressant drugs drugs used to treat depression and anxiety.
antipsychotic drugs drugs used to treat psychotic symptoms such as delusions, hallucinations, and
other bizarre behavior.
arbitrary inference distortion oI thinking in which a person draws a conclusion that is not based
on any evidence.
authenticity the genuine, open, and honest response oI the therapist to the client.
aversion therapy Iorm oI behavioral therapy in which an undesirable behavior is paired with an
aversive stimulus to reduce the Irequency oI the behavior.
behavior modification
or applied behavior
analysis
the use oI learning techniques to modiIy or change undesirable behavior and
increase desirable behavior.
behavior therapies action therapies based on the principles oI classical and operant conditioning
and aimed at changing disordered behavior without concern Ior the original
causes oI such behavior.
bilateral cingulotomy surgical technique in which an electrode wire is inserted into the anterior
cingulate gyrus with the guidance oI a magnetic resonance imaging machine
Ior the purpose oI destroying that area oI brain tissue with an electric current.
Psychological Therapies CHAPTER 15 -209-

biomedical therapy therapy Ior mental disorders in which a person with a problem is treated with
biological or medical methods to relieve symptoms; also deIined as therapies
that directly aIIect the Iunctioning oI the body and brain.
cognitive behavioral
therapy (CBT)
action therapy in which the goal is to help clients overcome problems by
learning to think more rationally and logically.
cognitive therapy therapy in which the Iocus is on helping clients recognize distortions in their
thinking and replace distorted, unrealistic belieIs with more realistic, helpIul
thoughts.
contingency contract a Iormal, written agreement between the therapist and client (or teacher and
student) in which goals Ior behavioral change, reinIorcements, and penalties
are clearly stated.
cybertherapy psychotherapy that is oIIered on the Internet. Also called online, Internet, or
Web therapy or counseling.
directive therapy in which the therapist actively gives interpretations oI a client`s
statements and may suggest certain behavior or actions.
eclectic approach to therapy that results Irom combining elements oI several diIIerent
approaches or techniques.
electroconvulsive
therapy (ECT)
Iorm oI biomedical therapy to treat severe depression in which electrodes are
placed on either one or both sides oI a person`s head and an electric current is
passed through the electrodes that is strong enough to cause a seizure or
convulsion.
empathy the ability oI the therapist to understand the Ieelings oI the client.
exposure therapies behavioral techniques that expose individuals to anxiety- or Iear-related
stimuli, under careIully controlled conditions, to promote new learning.
extinction the removal oI a reinIorcer to reduce the Irequency oI a behavior.
family counseling
(family therapy)
a Iorm oI group therapy in which Iamily members meet together with a
counselor or therapist to resolve problems that aIIect the entire Iamily.
flooding technique Ior treating phobias and other stress disorders in which the person
is rapidly and intensely exposed to the Iear-provoking situation or object and
prevented Irom making the usual avoidance or escape response.
free association psychoanalytic technique in which a patient was encouraged to talk about
anything that came to mind without Iear oI negative evaluations.
Gestalt therapy Iorm oI directive insight therapy in which the therapist helps the client to
accept all parts oI his or her Ieelings and subjective experiences, using
leading questions and planned experiences such as role-playing.
insight therapies therapies in which the main goal is helping people to gain insight with respect
to their behavior, thoughts, and Ieelings.
interpersonal therapy
(IPT)
Iorm oI therapy Ior depression which incorporates multiple approaches and
Iocuses on interpersonal problems.
latent content the symbolic or hidden meaning oI dreams.
magnification and
minimization
distortions oI thinking in which a person blows a negative event out oI
proportion to its importance (magniIication) while ignoring relevant positive
events (minimization).
manifest content the actual content oI one`s dream.
modeling learning through the observation and imitation oI others.
nondirective therapy style in which the therapist remains relatively neutral and does not
interpret or take direct actions with regard to the client, instead remaining a
calm, nonjudgmental listener while the client talks.
Psychological Therapies CHAPTER 15 -210-

overgeneralization distortion oI thinking in which a person draws sweeping conclusions based on
only one incident or event and applies those conclusions to events that are
unrelated to the original.
participant modeling technique in which a model demonstrates the desired behavior in a step-by-
step, gradual process while the client is encouraged to imitate the model.
personalization distortion oI thinking in which a person takes responsibility or blame Ior
events that are unconnected to the person.
person-centered
therapy
a nondirective insight therapy based on the work oI Carl Rogers in which the
client does all the talking and the therapist listens.
prefrontal lobotomy psychosurgery in which the connections oI the preIrontal lobes oI the brain to
the rear portions are severed.
psychoanalysis an insight therapy based on the theory oI Freud, emphasizing the revealing oI
unconscious conIlicts.
psychodynamic
therapy
a newer and more general term Ior therapies based on psychoanalysis, with an
emphasis on transIerence, shorter treatment times and a more direct
therapeutic approach.
psychopharmacology the use oI drugs to control or relieve the symptoms oI psychological
disorders.
psychosurgery surgery perIormed on brain tissue to relieve or control severe psychological
disorders.
psychotherapy therapy Ior mental disorders in which a person with a problem talks with a
psychological proIessional.
rational-emotive
behavioral therapy
(REBT)
cognitive-behavioral therapy in which clients are directly challenged in their
irrational belieIs and helped to restructure their thinking into more rational
belieI statements.
reflection therapy technique in which the therapist restates what the client says rather
than interpreting those statements.
reinforcement the strengthening oI a response by Iollowing it with a pleasurable
consequence or the removal oI an unpleasant stimulus.
resistance occurring when a patient becomes reluctant to talk about a certain topic,
either changing the subject or becoming silent.
selective thinking distortion oI thinking in which a person Iocuses on only one aspect oI a
situation while ignoring all other relevant aspects.
self-help groups
(support groups)
a group composed oI people who have similar problems and who meet
together without a therapist or counselor Ior the purpose oI discussion,
problem solving, and social and emotional support.
systematic
desensitization
behavior technique used to treat phobias, in which a client is asked to make a
list oI ordered Iears and taught to relax while concentrating on those Iears.
therapeutic alliance the relationship between therapist and client that develops as a warm, caring,
accepting relationship characterized by empathy, mutual respect, and
understanding.
therapy treatment methods aimed at making people Ieel better and Iunction more
eIIectively.
time-out an extinction process in which a person is removed Irom the situation that
provides reinIorcement Ior undesirable behavior, usually by being placed in a
quiet corner or room away Irom possible attention and reinIorcement
opportunities.
token economy the use oI objects called tokens to reinIorce behavior in which the tokens can
be accumulated and exchanged Ior desired items or privileges.
Psychological Therapies CHAPTER 15 -211-
Psychological Therapies CHAPTER 15 -212-

transference in psychoanalysis, the tendency Ior a patient or client to project positive or
negative Ieelings Ior important people Irom the past onto the therapist.
unconditional positive
regard
reIerring to the warmth, respect, and accepting atmosphere created by the
therapist Ior the client in person-centered therapy.

Vous aimerez peut-être aussi